Download as pdf or txt
Download as pdf or txt
You are on page 1of 109

12/31/2017 Adda247 Store | Adda247 Store

Purchased by roshnipanwar025@gmail.com
Q1.
Read the following information carefully and answer the questions given below:
Eight persons P, Q, R, S, T, U, V and X. All of them are sitting around a circular table. Some of them
are facing inside while some are facing outside of the center. Each of the person is going to give
di erent exam viz. IBPS PO, IBPS Clerk, IBPS SO, Rrb PO, Rrb Clerk, UIIC Assistant, RBI Assistant,
OICL AO. Also each of them has di erent examination date in the month of November viz. 5, 9, 12,
15, 17, 21, 25 and 28, but not necessarily in the same order.
T sits third to the left of the one who gives exam of IBPS Clerk. Both S and V sit next to each other
and face same direction. R sits second to the right of X and has exam of UIIC Assistant. R has exam
after more than one person. U has exam on the date which is an odd one and multiple of 5. The
one who has exam of OICL AO has exam on date which is prime number but not 5. Only one
person sits between the who has exam of RBI Assistant and the one who has exam of Rrb Clerk
and both of them faces opposite direction to each other. The one who has exam of IBPS PO has
exam date which is a perfect square. T has exam on even date. P’s immediate neighbour has exam
date which is a square of the exam date of P. Q is facing outside of the center and gives exam after
maximum number of persons. X faces T, who has exam of Rrb Clerk. V sits to the immediate right
of the one who gives exam of RRB PO. Q sits between the one who gives exam of UIIC Assistant
and IBPS PO. P is an immediate neighbour of T and sits to right of T. Neither S nor V gives the
exam of IBPS clerk. The immediate left of the one, whose exam date is multiple of six, gives exam
of IBPS SO. Both U and R faces same direction but opposite to S.
Who gives exam of OICL AO? (Marks 2)
A. T
B. S
C. V
D. R
E. Q
Your Answer: Not Attempted

Correct Answer: B. S <br>

Solution for more join


X faces T who has exam of Rrb Clerk. T sits third to the left of the one who gives exam of IBPS
Clerk. Only one person sits between thehttps://t.me/currentAffairscLuB
who has exam of RBI Assistant and the one who has exam
of Rrb Clerk So, there can be two cases:

R sits second to the right of X and has exam of UIIC Assistant. So, it is clear that the one who has
exam of RBI Assistant is an immediate neighbour of the one who has exam of IBPS Clerk. Q sits
between the one who gives exam of UIIC Assistant and IBPS PO. P is an immediate neighbour of T
and sits to right of T. Both S and V sit next to each other and face same direction. So, it is clear that
https://store.adda247.com/#!/myTestAnalysis/printsolution/mappingId=25273/packageId=359/lang=ENGLISH 1/109
12/31/2017 Adda247 Store | Adda247 Store
U will sit in between T and R.

T has exam on even date. P’s immediate neighbour has exam date which is a square of the exam
date of P. So, only possibility is P’s exam will be on 5 and his immediate neighbour has exam on
25. So, either S or V gives exam on 25. Q is facing outside of the center and gives exam after
maximum number of persons. So, Q gives exam on 28. The one who has exam of IBPS PO has
exam date which is a perfect square. So, X gives exam on 9. T has exam on even date. So, T gives
exam on 12.

Neither S nor V gives the exam of IBPS clerk. So, case-1 will be eliminated. Now from case-2: The
one who has exam of OICL AO has exam on date which is prime number but not 5. So, either S or
V gives exam of OICL on 17. U has exam on the date which is an odd one and multiple of 5. The
who has exam of RBI Assistant and the one who has exam of Rrb Clerk and both of them faces
opposite direction to each other. V sits to the immediate right of the one who gives exam of RRB
PO. The immediate left of the one, whose exam date is multiple of six, gives exam of IBPS SO. So,
U gives exam of IBPS SO. R has exam after more than one person. Only one person sits between
the who has exam of RBI Assistant and the one who has exam of Rrb Clerk and both of them faces
opposite direction to each other. Both S and V sit next to each other and face same direction. Both
U and R faces same direction but opposite to S. So, nal solution is----

Purchased by roshnipanwar025@gmail.com
Q2.
Read the following information carefully and answer the questions given below:
Eight persons P, Q, R, S, T, U, V and X. All of them are sitting around a circular table. Some of them
are facing inside while some are facing outside of the center. Each of the person is going to give
di erent exam viz. IBPS PO, IBPS Clerk, IBPS SO, Rrb PO, Rrb Clerk, UIIC Assistant, RBI Assistant,
OICL AO. Also each of them has di erent examination date in the month of November viz. 5, 9, 12,
15, 17, 21, 25 and 28, but not necessarily in the same order.
T sits third to the left of the one who gives exam of IBPS Clerk. Both S and V sit next to each other
and face same direction. R sits second to the right of X and has exam of UIIC Assistant. R has exam
after more than one person. U has exam on the date which is an odd one and multiple of 5. The
one who has exam of OICL AO has exam on date which is prime number but not 5. Only one
for more join
person sits between the who has exam of RBI Assistant and the one who has exam of Rrb Clerk
https://t.me/currentAffairscLuB
and both of them faces opposite direction to each other. The one who has exam of IBPS PO has
exam date which is a perfect square. T has exam on even date. P’s immediate neighbour has exam
date which is a square of the exam date of P. Q is facing outside of the center and gives exam after
maximum number of persons. X faces T, who has exam of Rrb Clerk. V sits to the immediate right
of the one who gives exam of RRB PO. Q sits between the one who gives exam of UIIC Assistant
and IBPS PO. P is an immediate neighbour of T and sits to right of T. Neither S nor V gives the

https://store.adda247.com/#!/myTestAnalysis/printsolution/mappingId=25273/packageId=359/lang=ENGLISH 2/109
12/31/2017 Adda247 Store | Adda247 Store
exam of IBPS clerk. The immediate left of the one, whose exam date is multiple of six, gives exam
of IBPS SO. Both U and R faces same direction but opposite to S.
The one who gives exam of IBPS SO gives exam on which date? (Marks 2)
A. 12
B. 28
C. 21
D. 15
E. 9
Your Answer: Not Attempted

Correct Answer: D. 15 <br>

Solution
X faces T who has exam of Rrb Clerk. T sits third to the left of the one who gives exam of IBPS
Clerk. Only one person sits between the who has exam of RBI Assistant and the one who has exam
of Rrb Clerk So, there can be two cases:

R sits second to the right of X and has exam of UIIC Assistant. So, it is clear that the one who has
exam of RBI Assistant is an immediate neighbour of the one who has exam of IBPS Clerk. Q sits
between the one who gives exam of UIIC Assistant and IBPS PO. P is an immediate neighbour of T
and sits to right of T. Both S and V sit next to each other and face same direction. So, it is clear that
U will sit in between T and R.

T has exam on even date. P’s immediate neighbour has exam date which is a square of the exam
date of P. So, only possibility is P’s exam will be on 5 and his immediate neighbour has exam on
25. So, either S or V gives exam on 25. Q is facing outside of the center and gives exam after
maximum number of persons. So, Q gives exam on 28. The one who has exam of IBPS PO has
exam date which is a perfect square. So, X gives exam on 9. T has exam on even date. So, T gives
exam on 12.

Neither S nor V gives the exam of IBPS clerk. So, case-1 will be eliminated. Now from case-2: The
for more join
one who has exam of OICL AO has exam on date which is prime number but not 5. So, either S or
V gives exam of OICL on 17. U has exam on the date which is an odd one and multiple of 5. The
https://t.me/currentAffairscLuB
who has exam of RBI Assistant and the one who has exam of Rrb Clerk and both of them faces
opposite direction to each other. V sits to the immediate right of the one who gives exam of RRB
PO. The immediate left of the one, whose exam date is multiple of six, gives exam of IBPS SO. So,
U gives exam of IBPS SO. R has exam after more than one person. Only one person sits between
the who has exam of RBI Assistant and the one who has exam of Rrb Clerk and both of them faces

https://store.adda247.com/#!/myTestAnalysis/printsolution/mappingId=25273/packageId=359/lang=ENGLISH 3/109
12/31/2017 Adda247 Store | Adda247 Store
opposite direction to each other. Both S and V sit next to each other and face same direction. Both
U and R faces same direction but opposite to S. So, nal solution is----

Purchased by roshnipanwar025@gmail.com
Q3.
Read the following information carefully and answer the questions given below:
Eight persons P, Q, R, S, T, U, V and X. All of them are sitting around a circular table. Some of them
are facing inside while some are facing outside of the center. Each of the person is going to give
di erent exam viz. IBPS PO, IBPS Clerk, IBPS SO, Rrb PO, Rrb Clerk, UIIC Assistant, RBI Assistant,
OICL AO. Also each of them has di erent examination date in the month of November viz. 5, 9, 12,
15, 17, 21, 25 and 28, but not necessarily in the same order.
T sits third to the left of the one who gives exam of IBPS Clerk. Both S and V sit next to each other
and face same direction. R sits second to the right of X and has exam of UIIC Assistant. R has exam
after more than one person. U has exam on the date which is an odd one and multiple of 5. The
one who has exam of OICL AO has exam on date which is prime number but not 5. Only one
person sits between the who has exam of RBI Assistant and the one who has exam of Rrb Clerk
and both of them faces opposite direction to each other. The one who has exam of IBPS PO has
exam date which is a perfect square. T has exam on even date. P’s immediate neighbour has exam
date which is a square of the exam date of P. Q is facing outside of the center and gives exam after
maximum number of persons. X faces T, who has exam of Rrb Clerk. V sits to the immediate right
of the one who gives exam of RRB PO. Q sits between the one who gives exam of UIIC Assistant
and IBPS PO. P is an immediate neighbour of T and sits to right of T. Neither S nor V gives the
exam of IBPS clerk. The immediate left of the one, whose exam date is multiple of six, gives exam
of IBPS SO. Both U and R faces same direction but opposite to S.
Who among the following sits third to the right of S? (Marks 2)
A. The one who gives exam of IBPS Clerk
B. The one who gives exam of Rrb Clerk
C. The one who gives exam on 5
D. The one who gives exam of RBI Assistant
E. The one who gives exam of 21
Your Answer: Not Attempted
for more join
Correct Answer: B. The one who gives exam of Rrb Clerk <br>
https://t.me/currentAffairscLuB
Solution
X faces T who has exam of Rrb Clerk. T sits third to the left of the one who gives exam of IBPS
Clerk. Only one person sits between the who has exam of RBI Assistant and the one who has exam
of Rrb Clerk So, there can be two cases:

R sits second to the right of X and has exam of UIIC Assistant. So, it is clear that the one who has
https://store.adda247.com/#!/myTestAnalysis/printsolution/mappingId=25273/packageId=359/lang=ENGLISH 4/109
12/31/2017 Adda247 Store | Adda247 Store
exam of RBI Assistant is an immediate neighbour of the one who has exam of IBPS Clerk. Q sits
between the one who gives exam of UIIC Assistant and IBPS PO. P is an immediate neighbour of T
and sits to right of T. Both S and V sit next to each other and face same direction. So, it is clear that
U will sit in between T and R.

T has exam on even date. P’s immediate neighbour has exam date which is a square of the exam
date of P. So, only possibility is P’s exam will be on 5 and his immediate neighbour has exam on
25. So, either S or V gives exam on 25. Q is facing outside of the center and gives exam after
maximum number of persons. So, Q gives exam on 28. The one who has exam of IBPS PO has
exam date which is a perfect square. So, X gives exam on 9. T has exam on even date. So, T gives
exam on 12.

Neither S nor V gives the exam of IBPS clerk. So, case-1 will be eliminated. Now from case-2: The
one who has exam of OICL AO has exam on date which is prime number but not 5. So, either S or
V gives exam of OICL on 17. U has exam on the date which is an odd one and multiple of 5. The
who has exam of RBI Assistant and the one who has exam of Rrb Clerk and both of them faces
opposite direction to each other. V sits to the immediate right of the one who gives exam of RRB
PO. The immediate left of the one, whose exam date is multiple of six, gives exam of IBPS SO. So,
U gives exam of IBPS SO. R has exam after more than one person. Only one person sits between
the who has exam of RBI Assistant and the one who has exam of Rrb Clerk and both of them faces
opposite direction to each other. Both S and V sit next to each other and face same direction. Both
U and R faces same direction but opposite to S. So, nal solution is----

Purchased by roshnipanwar025@gmail.com
Q4.
Read the following information carefully and answer the questions given below:
Eight persons P, Q, R, S, T, U, V and X. All of them are sitting around a circular table. Some of them
are facing inside while some are facing outside of the center. Each of the person is going to give
di erent exam viz. IBPS PO, IBPS Clerk, IBPS SO, Rrb PO, Rrb Clerk, UIIC Assistant, RBI Assistant,
OICL AO. Also each of them has di erent examination date in the month of November viz. 5, 9, 12,
15, 17, 21, 25 and 28, but not necessarily in the same order.
T sits third to the left of the one who gives exam of IBPS Clerk. Both S and V sit next to each other
for more join
and face same direction. R sits second to the right of X and has exam of UIIC Assistant. R has exam
after more than one person. U has exam https://t.me/currentAffairscLuB
on the date which is an odd one and multiple of 5. The
one who has exam of OICL AO has exam on date which is prime number but not 5. Only one
person sits between the who has exam of RBI Assistant and the one who has exam of Rrb Clerk
and both of them faces opposite direction to each other. The one who has exam of IBPS PO has
exam date which is a perfect square. T has exam on even date. P’s immediate neighbour has exam
date which is a square of the exam date of P. Q is facing outside of the center and gives exam after
maximum number of persons. X faces T, who has exam of Rrb Clerk. V sits to the immediate right
https://store.adda247.com/#!/myTestAnalysis/printsolution/mappingId=25273/packageId=359/lang=ENGLISH 5/109
12/31/2017 Adda247 Store | Adda247 Store
of the one who gives exam of RRB PO. Q sits between the one who gives exam of UIIC Assistant
and IBPS PO. P is an immediate neighbour of T and sits to right of T. Neither S nor V gives the
exam of IBPS clerk. The immediate left of the one, whose exam date is multiple of six, gives exam
of IBPS SO. Both U and R faces same direction but opposite to S.
Which of the following combination is true regarding V? (Marks 2)
A. IBPS PO-9
B. RRB PO-5
C. RRB Clerk-12
D. OICL AO-17
E. RBI Assistant-25
Your Answer: Not Attempted

Correct Answer: E. RBI Assistant-25 <br>

Solution
X faces T who has exam of Rrb Clerk. T sits third to the left of the one who gives exam of IBPS
Clerk. Only one person sits between the who has exam of RBI Assistant and the one who has exam
of Rrb Clerk So, there can be two cases:

R sits second to the right of X and has exam of UIIC Assistant. So, it is clear that the one who has
exam of RBI Assistant is an immediate neighbour of the one who has exam of IBPS Clerk. Q sits
between the one who gives exam of UIIC Assistant and IBPS PO. P is an immediate neighbour of T
and sits to right of T. Both S and V sit next to each other and face same direction. So, it is clear that
U will sit in between T and R.

T has exam on even date. P’s immediate neighbour has exam date which is a square of the exam
date of P. So, only possibility is P’s exam will be on 5 and his immediate neighbour has exam on
25. So, either S or V gives exam on 25. Q is facing outside of the center and gives exam after
maximum number of persons. So, Q gives exam on 28. The one who has exam of IBPS PO has
exam date which is a perfect square. So, X gives exam on 9. T has exam on even date. So, T gives
exam on 12.
for more join
Neither S nor V gives the exam of IBPS clerk. So, case-1 will be eliminated. Now from case-2: The
https://t.me/currentAffairscLuB
one who has exam of OICL AO has exam on date which is prime number but not 5. So, either S or
V gives exam of OICL on 17. U has exam on the date which is an odd one and multiple of 5. The
who has exam of RBI Assistant and the one who has exam of Rrb Clerk and both of them faces
opposite direction to each other. V sits to the immediate right of the one who gives exam of RRB
PO. The immediate left of the one, whose exam date is multiple of six, gives exam of IBPS SO. So,
U gives exam of IBPS SO. R has exam after more than one person. Only one person sits between

https://store.adda247.com/#!/myTestAnalysis/printsolution/mappingId=25273/packageId=359/lang=ENGLISH 6/109
12/31/2017 Adda247 Store | Adda247 Store
the who has exam of RBI Assistant and the one who has exam of Rrb Clerk and both of them faces
opposite direction to each other. Both S and V sit next to each other and face same direction. Both
U and R faces same direction but opposite to S. So, nal solution is----

Purchased by roshnipanwar025@gmail.com
Q5.
Read the following information carefully and answer the questions given below:
Eight persons P, Q, R, S, T, U, V and X. All of them are sitting around a circular table. Some of them
are facing inside while some are facing outside of the center. Each of the person is going to give
di erent exam viz. IBPS PO, IBPS Clerk, IBPS SO, Rrb PO, Rrb Clerk, UIIC Assistant, RBI Assistant,
OICL AO. Also each of them has di erent examination date in the month of November viz. 5, 9, 12,
15, 17, 21, 25 and 28, but not necessarily in the same order.
T sits third to the left of the one who gives exam of IBPS Clerk. Both S and V sit next to each other
and face same direction. R sits second to the right of X and has exam of UIIC Assistant. R has exam
after more than one person. U has exam on the date which is an odd one and multiple of 5. The
one who has exam of OICL AO has exam on date which is prime number but not 5. Only one
person sits between the who has exam of RBI Assistant and the one who has exam of Rrb Clerk
and both of them faces opposite direction to each other. The one who has exam of IBPS PO has
exam date which is a perfect square. T has exam on even date. P’s immediate neighbour has exam
date which is a square of the exam date of P. Q is facing outside of the center and gives exam after
maximum number of persons. X faces T, who has exam of Rrb Clerk. V sits to the immediate right
of the one who gives exam of RRB PO. Q sits between the one who gives exam of UIIC Assistant
and IBPS PO. P is an immediate neighbour of T and sits to right of T. Neither S nor V gives the
exam of IBPS clerk. The immediate left of the one, whose exam date is multiple of six, gives exam
of IBPS SO. Both U and R faces same direction but opposite to S.
R gives exam on which date? (Marks 2)
A. 15
B. 17
C. 21
D. 5
E. 9
Your Answer: Not Attempted for more join
Correct Answer: C. 21 <br> https://t.me/currentAffairscLuB
Solution
X faces T who has exam of Rrb Clerk. T sits third to the left of the one who gives exam of IBPS
Clerk. Only one person sits between the who has exam of RBI Assistant and the one who has exam
of Rrb Clerk So, there can be two cases:

https://store.adda247.com/#!/myTestAnalysis/printsolution/mappingId=25273/packageId=359/lang=ENGLISH 7/109
12/31/2017 Adda247 Store | Adda247 Store
R sits second to the right of X and has exam of UIIC Assistant. So, it is clear that the one who has
exam of RBI Assistant is an immediate neighbour of the one who has exam of IBPS Clerk. Q sits
between the one who gives exam of UIIC Assistant and IBPS PO. P is an immediate neighbour of T
and sits to right of T. Both S and V sit next to each other and face same direction. So, it is clear that
U will sit in between T and R.

T has exam on even date. P’s immediate neighbour has exam date which is a square of the exam
date of P. So, only possibility is P’s exam will be on 5 and his immediate neighbour has exam on
25. So, either S or V gives exam on 25. Q is facing outside of the center and gives exam after
maximum number of persons. So, Q gives exam on 28. The one who has exam of IBPS PO has
exam date which is a perfect square. So, X gives exam on 9. T has exam on even date. So, T gives
exam on 12.

Neither S nor V gives the exam of IBPS clerk. So, case-1 will be eliminated. Now from case-2: The
one who has exam of OICL AO has exam on date which is prime number but not 5. So, either S or
V gives exam of OICL on 17. U has exam on the date which is an odd one and multiple of 5. The
who has exam of RBI Assistant and the one who has exam of Rrb Clerk and both of them faces
opposite direction to each other. V sits to the immediate right of the one who gives exam of RRB
PO. The immediate left of the one, whose exam date is multiple of six, gives exam of IBPS SO. So,
U gives exam of IBPS SO. R has exam after more than one person. Only one person sits between
the who has exam of RBI Assistant and the one who has exam of Rrb Clerk and both of them faces
opposite direction to each other. Both S and V sit next to each other and face same direction. Both
U and R faces same direction but opposite to S. So, nal solution is----

Purchased by roshnipanwar025@gmail.com
Q6.
Law made on eradicating superstition from society is an important step towards this reform. But
banning ‘evil’ practices by law is not enough: social reform must be more broad-based.
Which of the following strengthen the given statement?
(I)Mere legislation is not enough to eradicate superstition from society, but laws do have the utility
value of curbing the prevalence of inhuman rituals and practices.
(II)The Karnataka Prevention and Eradication of Inhuman Evil Practices and Black Magic Bill, 2017
for more join
has been approved by the State Cabinet and is likely to be introduced soon in the Assembly.
https://t.me/currentAffairscLuB
(III)Ultimately, it is education and awareness that can truly liberate a society from superstition,
blind faith and abominable practices in the name of faith. (Marks 2)
A. Only (II)
B. Only (II) and (I)
C. Only (I)
D. Both (I) and (III)
E. None of these
https://store.adda247.com/#!/myTestAnalysis/printsolution/mappingId=25273/packageId=359/lang=ENGLISH 8/109
12/31/2017 Adda247 Store | Adda247 Store
Your Answer: Not Attempted

Correct Answer: D. Both (I) and (III) <br>

Solution
For I-This statement supports the given statement as it describes that only laws are not enough to
end superstition from society but the law will help in curbing the prevalence of inhuman rituals
and practices which is also mentioned in the given statement.
For II- This statement neither strengthens nor negates the given statement but provides the
information regarding The Karnataka Prevention and Eradication of Inhuman Evil Practices and
Black Magic Bill, 2017 which has been approved by the assembly.
For III- This statement supports the given statement as it describes that ultimately education and
awareness that can truly liberate a society from superstition which is also mentioned in the given
statement that social reform must be more broad-based to eradicate superstition from society.
Purchased by roshnipanwar025@gmail.com
Q7.
Physical conditions necessary for life on earth such as availability of oxygen, required heat, light
etc are not present on any planet in the solar system other than the Earth. Thus it can be
concluded that planets other than Earth cannot be the habitats of any life.
Which of the following is an assumption in arriving at the above conclusion? (Marks 1)
A. Planets other than Earth do not have enough oxygen.
B. Planets belonging to systems other than our solar system may sustain life.
C. Earth is the most conducive place for any life.
D. Physical conditions necessary for life on Earth are the same as those required for organisms of
any kind.
E. None of these
Your Answer: Not Attempted

Correct Answer: D. Physical conditions necessary for life on Earth are the same as those required
for organisms of any kind. <br>

Solution for more join


(a), (b) and (c) are not the assumptions underlying the conclusions.(d) is the correct assumption as
https://t.me/currentAffairscLuB
it is assumed by the passage that the conditions which are necessary for life on earth are also
necessary for life on other planet.
Purchased by roshnipanwar025@gmail.com
Q8.
Read the following information carefully and answer the questions given below:
Ten persons of a family are E, J, K, L, N, O, S, T, U and W of a three generation. L and O are cousins
of each other. W is the aunt of L. E is the sister-in-law of U. J is the son of K, who is the son-in-law of
https://store.adda247.com/#!/myTestAnalysis/printsolution/mappingId=25273/packageId=359/lang=ENGLISH 9/109
12/31/2017 Adda247 Store | Adda247 Store
the wife of U. S is the spouse of J. N is married to the son of T, who is the uncle of O. W is the
mother of the grand mother-in-law of S and has one sibling. E is the mother-in-law of N.
How is T related to W? (Marks 2)
A. Mother
B. Father-in-law
C. Sister
D. Brother-in-law
E. Can’t be determine
Your Answer: Not Attempted

Correct Answer: D. Brother-in-law <br>

Solution

Purchased by roshnipanwar025@gmail.com
Q9.
Read the following information carefully and answer the questions given below:
Ten persons of a family are E, J, K, L, N, O, S, T, U and W of a three generation. L and O are cousins
of each other. W is the aunt of L. E is the sister-in-law of U. J is the son of K, who is the son-in-law of
the wife of U. S is the spouse of J. N is married to the son of T, who is the uncle of O. W is the
mother of the grand mother-in-law of S and has one sibling. E is the mother-in-law of N.
How is O related to E? (Marks 2)
A. Cousin
B. Father-in-law
C. Sister-in-law
D. Niece
E. Can’t be determine
Your Answer: Not Attempted

Correct Answer: D. Niece <br>

Solution for more join


https://t.me/currentAffairscLuB
Purchased by roshnipanwar025@gmail.com
Q10.
What is a popular program for creating documents that can be read on both a PC and a Macintosh
computer? (Marks 1)
A. Microsoft Word
B. Adobe In Design
C. Adobe Acrobat
https://store.adda247.com/#!/myTestAnalysis/printsolution/mappingId=25273/packageId=359/lang=ENGLISH 10/109
12/31/2017 Adda247 Store | Adda247 Store
D. QuarkXPress
E. None of these
Your Answer: Not Attempted

Correct Answer: C. Adobe Acrobat <br>

Solution
Purchased by roshnipanwar025@gmail.com
Q11.
Study the following information carefully and answer the questions given below:
P, Q, R, S, T, U and V are seven persons live in two di erent coaches’ homes. In Sunil gawasker’s
home they have some di erent sports items i.e. Net, Ball, Badminton, Shuttlecock, Football, Bat
and Helmet and in Anil kumble’s home they have some di erent sports items i.e. Bat, Ball, Net,
Badminton, Hockey stick, Football and Stump. They all are not necessarily in the same order.
The person, who have sports item in Sunil gawasker’s home and Anil Kumble’s home are not
necessarily in the same order. Q has Ball in Sunil gawasker’s home and does not have Hockey stick
and Football in Anil kumble’s home. Ball and Bat is not the combination of sports item of any of
the person has in Sunil gawasker’s home and Anil kumble’s home. S has Shuttlecock in Sunil
gawasker’s home and has Stump in Anil kumble’s home. Sports item of P in Sunil gawasker’s home
and sports item of U in Anil kumble’s home is same. Sports item of T in Sunil gawasker’s home and
sports item of R in Anil kumble’s home is same. None of the person have same sports item in Anil
Kumble’s home and Sunil gawasker’s home. V has Bat in Sunil gawasker’s home and T has Bat in
Anil kumble’s home. U does not have any Football.
Which among the following sports item does P have in Anil kumble’s home? (Marks 1)
A. Net
B. Hockey stick
C. Bat
D. None of these
E. Data inadequate
Your Answer: Not Attempted

Correct Answer: D. None of these <br> for more join


Solution https://t.me/currentAffairscLuB
Q has Ball in Sunil gawasker’s home and has not Hockey stick and Football in Anil kumble’s home.
S has Shuttlecock in Sunil gawasker’s home and has Stump in Anil kumble’s home. V has Bat in
Sunil gawasker’s home and T has Bat in Anil kumble’s home. U does not have any Football.

Ball and Bat is not the combination of sports item of any of the person has in Sunil gawasker’s
home and Anil kumble’s home. None of the person have same sports item in Anil Kumble’s home
https://store.adda247.com/#!/myTestAnalysis/printsolution/mappingId=25273/packageId=359/lang=ENGLISH 11/109
12/31/2017 Adda247 Store | Adda247 Store
and Sunil gawasker’s home. Sports item of T in Sunil gawasker’s home and sports item of R in Anil
kumble’s home is same. As U cannot have football and Sports item of P in Sunil gawasker’s home
and sports item of U in Anil kumble’s home is same so Sports item of P in Sunil gawasker’s home
and sports item of U in Anil kumble’s home can be either Net or Badminton. And the rest Sports
item of T in Sunil gawasker’s home and sports item of R in Anil kumble’s home will be Football.
Remaining P in Anil kumble’s home will have Ball and V in Anil kumble’s home will have Hockey. Q
will have either Net or Badminton. Both U and R can have in Sunil gawasker’s home
Helmet/Net/Badminton.

Purchased by roshnipanwar025@gmail.com
Q12.
Study the following information carefully and answer the questions given below:
P, Q, R, S, T, U and V are seven persons live in two di erent coaches’ homes. In Sunil gawasker’s
home they have some di erent sports items i.e. Net, Ball, Badminton, Shuttlecock, Football, Bat
and Helmet and in Anil kumble’s home they have some di erent sports items i.e. Bat, Ball, Net,
Badminton, Hockey stick, Football and Stump. They all are not necessarily in the same order.
The person, who have sports item in Sunil gawasker’s home and Anil Kumble’s home are not
necessarily in the same order. Q has Ball in Sunil gawasker’s home and does not have Hockey stick
and Football in Anil kumble’s home. Ball and Bat is not the combination of sports item of any of
the person has in Sunil gawasker’s home and Anil kumble’s home. S has Shuttlecock in Sunil
gawasker’s home and has Stump in Anil kumble’s home. Sports item of P in Sunil gawasker’s home
and sports item of U in Anil kumble’s home is same. Sports item of T in Sunil gawasker’s home and
sports item of R in Anil kumble’s home is same. None of the person have same sports item in Anil
Kumble’s home and Sunil gawasker’s home. V has Bat in Sunil gawasker’s home and T has Bat in
Anil kumble’s home. U does not have any Football.
Which among the following sports item does V have in Anil Kumble’s home? (Marks 1)
A. Ball
B. Stump
C. Net
D. Hockey stick
E. None of these
Your Answer: Not Attempted for more join
Correct Answer: D. Hockey stick <br> https://t.me/currentAffairscLuB
Solution
Q has Ball in Sunil gawasker’s home and has not Hockey stick and Football in Anil kumble’s home.
S has Shuttlecock in Sunil gawasker’s home and has Stump in Anil kumble’s home. V has Bat in
Sunil gawasker’s home and T has Bat in Anil kumble’s home. U does not have any Football.

https://store.adda247.com/#!/myTestAnalysis/printsolution/mappingId=25273/packageId=359/lang=ENGLISH 12/109
12/31/2017 Adda247 Store | Adda247 Store
Ball and Bat is not the combination of sports item of any of the person has in Sunil gawasker’s
home and Anil kumble’s home. None of the person have same sports item in Anil Kumble’s home
and Sunil gawasker’s home. Sports item of T in Sunil gawasker’s home and sports item of R in Anil
kumble’s home is same. As U cannot have football and Sports item of P in Sunil gawasker’s home
and sports item of U in Anil kumble’s home is same so Sports item of P in Sunil gawasker’s home
and sports item of U in Anil kumble’s home can be either Net or Badminton. And the rest Sports
item of T in Sunil gawasker’s home and sports item of R in Anil kumble’s home will be Football.
Remaining P in Anil kumble’s home will have Ball and V in Anil kumble’s home will have Hockey. Q
will have either Net or Badminton. Both U and R can have in Sunil gawasker’s home
Helmet/Net/Badminton.

Purchased by roshnipanwar025@gmail.com
Q13.
Study the following information carefully and answer the questions given below:
P, Q, R, S, T, U and V are seven persons live in two di erent coaches’ homes. In Sunil gawasker’s
home they have some di erent sports items i.e. Net, Ball, Badminton, Shuttlecock, Football, Bat
and Helmet and in Anil kumble’s home they have some di erent sports items i.e. Bat, Ball, Net,
Badminton, Hockey stick, Football and Stump. They all are not necessarily in the same order.
The person, who have sports item in Sunil gawasker’s home and Anil Kumble’s home are not
necessarily in the same order. Q has Ball in Sunil gawasker’s home and does not have Hockey stick
and Football in Anil kumble’s home. Ball and Bat is not the combination of sports item of any of
the person has in Sunil gawasker’s home and Anil kumble’s home. S has Shuttlecock in Sunil
gawasker’s home and has Stump in Anil kumble’s home. Sports item of P in Sunil gawasker’s home
and sports item of U in Anil kumble’s home is same. Sports item of T in Sunil gawasker’s home and
sports item of R in Anil kumble’s home is same. None of the person have same sports item in Anil
Kumble’s home and Sunil gawasker’s home. V has Bat in Sunil gawasker’s home and T has Bat in
Anil kumble’s home. U does not have any Football.
Who among the following person has Helmet in Sunil gawasker’s home? (Marks 1)
A. R
B. P
C. None of these
D. U for more join
E. R or U
Your Answer: Not Attempted https://t.me/currentAffairscLuB
Correct Answer: E. R or U <br>

Solution
Q has Ball in Sunil gawasker’s home and has not Hockey stick and Football in Anil kumble’s home.
S has Shuttlecock in Sunil gawasker’s home and has Stump in Anil kumble’s home. V has Bat in
https://store.adda247.com/#!/myTestAnalysis/printsolution/mappingId=25273/packageId=359/lang=ENGLISH 13/109
12/31/2017 Adda247 Store | Adda247 Store
Sunil gawasker’s home and T has Bat in Anil kumble’s home. U does not have any Football.

Ball and Bat is not the combination of sports item of any of the person has in Sunil gawasker’s
home and Anil kumble’s home. None of the person have same sports item in Anil Kumble’s home
and Sunil gawasker’s home. Sports item of T in Sunil gawasker’s home and sports item of R in Anil
kumble’s home is same. As U cannot have football and Sports item of P in Sunil gawasker’s home
and sports item of U in Anil kumble’s home is same so Sports item of P in Sunil gawasker’s home
and sports item of U in Anil kumble’s home can be either Net or Badminton. And the rest Sports
item of T in Sunil gawasker’s home and sports item of R in Anil kumble’s home will be Football.
Remaining P in Anil kumble’s home will have Ball and V in Anil kumble’s home will have Hockey. Q
will have either Net or Badminton. Both U and R can have in Sunil gawasker’s home
Helmet/Net/Badminton.

Purchased by roshnipanwar025@gmail.com
Q14.
Study the following information carefully and answer the questions given below:
P, Q, R, S, T, U and V are seven persons live in two di erent coaches’ homes. In Sunil gawasker’s
home they have some di erent sports items i.e. Net, Ball, Badminton, Shuttlecock, Football, Bat
and Helmet and in Anil kumble’s home they have some di erent sports items i.e. Bat, Ball, Net,
Badminton, Hockey stick, Football and Stump. They all are not necessarily in the same order.
The person, who have sports item in Sunil gawasker’s home and Anil Kumble’s home are not
necessarily in the same order. Q has Ball in Sunil gawasker’s home and does not have Hockey stick
and Football in Anil kumble’s home. Ball and Bat is not the combination of sports item of any of
the person has in Sunil gawasker’s home and Anil kumble’s home. S has Shuttlecock in Sunil
gawasker’s home and has Stump in Anil kumble’s home. Sports item of P in Sunil gawasker’s home
and sports item of U in Anil kumble’s home is same. Sports item of T in Sunil gawasker’s home and
sports item of R in Anil kumble’s home is same. None of the person have same sports item in Anil
Kumble’s home and Sunil gawasker’s home. V has Bat in Sunil gawasker’s home and T has Bat in
Anil kumble’s home. U does not have any Football.
Which among the following sports item does U have in Sunil gawasker’s home? (Marks 1)
A. Bat
B. Data inadequate for more join
C. Shuttlecock
D. Helmet https://t.me/currentAffairscLuB
E. None of these

Your Answer: Not Attempted

Correct Answer: B. Data inadequate <br>

https://store.adda247.com/#!/myTestAnalysis/printsolution/mappingId=25273/packageId=359/lang=ENGLISH 14/109
12/31/2017 Adda247 Store | Adda247 Store
Solution
Q has Ball in Sunil gawasker’s home and has not Hockey stick and Football in Anil kumble’s home.
S has Shuttlecock in Sunil gawasker’s home and has Stump in Anil kumble’s home. V has Bat in
Sunil gawasker’s home and T has Bat in Anil kumble’s home. U does not have any Football.

Ball and Bat is not the combination of sports item of any of the person has in Sunil gawasker’s
home and Anil kumble’s home. None of the person have same sports item in Anil Kumble’s home
and Sunil gawasker’s home. Sports item of T in Sunil gawasker’s home and sports item of R in Anil
kumble’s home is same. As U cannot have football and Sports item of P in Sunil gawasker’s home
and sports item of U in Anil kumble’s home is same so Sports item of P in Sunil gawasker’s home
and sports item of U in Anil kumble’s home can be either Net or Badminton. And the rest Sports
item of T in Sunil gawasker’s home and sports item of R in Anil kumble’s home will be Football.
Remaining P in Anil kumble’s home will have Ball and V in Anil kumble’s home will have Hockey. Q
will have either Net or Badminton. Both U and R can have in Sunil gawasker’s home
Helmet/Net/Badminton.

Purchased by roshnipanwar025@gmail.com
Q15.
Study the following information carefully and answer the questions given below:
P, Q, R, S, T, U and V are seven persons live in two di erent coaches’ homes. In Sunil gawasker’s
home they have some di erent sports items i.e. Net, Ball, Badminton, Shuttlecock, Football, Bat
and Helmet and in Anil kumble’s home they have some di erent sports items i.e. Bat, Ball, Net,
Badminton, Hockey stick, Football and Stump. They all are not necessarily in the same order.
The person, who have sports item in Sunil gawasker’s home and Anil Kumble’s home are not
necessarily in the same order. Q has Ball in Sunil gawasker’s home and does not have Hockey stick
and Football in Anil kumble’s home. Ball and Bat is not the combination of sports item of any of
the person has in Sunil gawasker’s home and Anil kumble’s home. S has Shuttlecock in Sunil
gawasker’s home and has Stump in Anil kumble’s home. Sports item of P in Sunil gawasker’s home
and sports item of U in Anil kumble’s home is same. Sports item of T in Sunil gawasker’s home and
sports item of R in Anil kumble’s home is same. None of the person have same sports item in Anil
Kumble’s home and Sunil gawasker’s home. V has Bat in Sunil gawasker’s home and T has Bat in
Anil kumble’s home. U does not have any Football.
for more join
Which among the following sports item does Q have in Anil kumble’s home? (Marks 1)
A. Ball https://t.me/currentAffairscLuB
B. Net
C. Stump
D. Data inadequate
E. None of these
Your Answer: Not Attempted

https://store.adda247.com/#!/myTestAnalysis/printsolution/mappingId=25273/packageId=359/lang=ENGLISH 15/109
12/31/2017 Adda247 Store | Adda247 Store
Correct Answer: D. Data inadequate <br>

Solution
Q has Ball in Sunil gawasker’s home and has not Hockey stick and Football in Anil kumble’s home.
S has Shuttlecock in Sunil gawasker’s home and has Stump in Anil kumble’s home. V has Bat in
Sunil gawasker’s home and T has Bat in Anil kumble’s home. U does not have any Football.

Ball and Bat is not the combination of sports item of any of the person has in Sunil gawasker’s
home and Anil kumble’s home. None of the person have same sports item in Anil Kumble’s home
and Sunil gawasker’s home. Sports item of T in Sunil gawasker’s home and sports item of R in Anil
kumble’s home is same. As U cannot have football and Sports item of P in Sunil gawasker’s home
and sports item of U in Anil kumble’s home is same so Sports item of P in Sunil gawasker’s home
and sports item of U in Anil kumble’s home can be either Net or Badminton. And the rest Sports
item of T in Sunil gawasker’s home and sports item of R in Anil kumble’s home will be Football.
Remaining P in Anil kumble’s home will have Ball and V in Anil kumble’s home will have Hockey. Q
will have either Net or Badminton. Both U and R can have in Sunil gawasker’s home
Helmet/Net/Badminton.

Purchased by roshnipanwar025@gmail.com
Q16.
Read the following information carefully and answer the questions given below:
Shreya starts walking towards east direction. After walking 15 km he turns towards north-west and
walks 8km and then he turns towards his right and walks 10km. Now he turns to the west and
walks 18km. From there he turns to his left and walks 5 km. Now, again turns to his left and walks
3km. From there he turns to his right and walks 2km. Now he turns 45° in anticlockwise direction
and then turns 135° in clockwise direction. Further moves 8km and then turns to his right and
walks 3km. Finally turns to his right and walks 4km and reached the nal position.
What is the direction of Shreya at the nal position with respect to his initial position? (Marks 1)
A. North-west
B. West
C. East
D. South-east for more join
E. South-west
Your Answer: Not Attempted https://t.me/currentAffairscLuB
Correct Answer: A. North-west <br>

Solution

Purchased by roshnipanwar025@gmail.com
https://store.adda247.com/#!/myTestAnalysis/printsolution/mappingId=25273/packageId=359/lang=ENGLISH 16/109
12/31/2017 Adda247 Store | Adda247 Store
Q17.
Read the following information carefully and answer the questions given below:
Shreya starts walking towards east direction. After walking 15 km he turns towards north-west and
walks 8km and then he turns towards his right and walks 10km. Now he turns to the west and
walks 18km. From there he turns to his left and walks 5 km. Now, again turns to his left and walks
3km. From there he turns to his right and walks 2km. Now he turns 45° in anticlockwise direction
and then turns 135° in clockwise direction. Further moves 8km and then turns to his right and
walks 3km. Finally turns to his right and walks 4km and reached the nal position.
How much total distance traveled by Shreya, before taking his rst left turn? (Marks 1)
A. 28km
B. 36km
C. 51km
D. 43km
E. None of these
Your Answer: Not Attempted

Correct Answer: C. 51km <br>

Solution

Purchased by roshnipanwar025@gmail.com
Q18.
Read the following passage carefully to answer the given questions:
Prime Minister Narendra Modi on Sunday inaugurated the rst phase of the Rs 615 crore ‘roll-on-
roll-o ’ (Ro-Ro) ferry service between Ghogha in Saurashtra and Dahej in south Gujarat. “This is
the rst of its kind project not only in India but also in South-East Asia,” Mr. Modi, a veteran leader
of BJP said at the rally which comes ahead of the Assembly polls in the State. “Imagine how much
time and petrol this ferry service will save. This ferry service connects Saurashtra and South
Gujarat… people from these regions frequently travels to and fro,” Mr. Modi said. He further
reiterated "This ferry service will not be restricted to this one route. We are planning to link other
places also through ferries.
Which of the following can be hypothesized from above mentioned details?
for more join
(I) Project has been launched by keeping state election in mind.
https://t.me/currentAffairscLuB
(II) People of Gujrat will welcome the gift of PM Modi and will happy to use this convenience.
(III) Ferry service will be started in entire nation very soon. (Marks 1)
A. Only II and III
B. Only I and III
C. Only II
D. Only I
E. None of these
https://store.adda247.com/#!/myTestAnalysis/printsolution/mappingId=25273/packageId=359/lang=ENGLISH 17/109
12/31/2017 Adda247 Store | Adda247 Store
Your Answer: Not Attempted

Correct Answer: E. None of these <br>

Solution
Description: We have to nd from the given point, which point can be assumed from the given
passage. For I- As mentioned in above given paragraph that this was project was launched in a
rally before Assembly polls in the state. So it may be launched as keeping state election in mind.
For II- As it is mentioned by PM Modi this project will save time and petrol. So, it can be assumed
as this project is for the convenience of people.
For III-There is no hint from PM Modi about implementation of Ferry service in all over India. So, it
can’t be assumed.
Purchased by roshnipanwar025@gmail.com
Q19.
Read the following passage carefully to answer the given questions:
Prime Minister Narendra Modi on Sunday inaugurated the rst phase of the Rs 615 crore ‘roll-on-
roll-o ’ (Ro-Ro) ferry service between Ghogha in Saurashtra and Dahej in south Gujarat. “This is
the rst of its kind project not only in India but also in South-East Asia,” Mr. Modi, a veteran leader
of BJP said at the rally which comes ahead of the Assembly polls in the State. “Imagine how much
time and petrol this ferry service will save. This ferry service connects Saurashtra and South
Gujarat… people from these regions frequently travels to and fro,” Mr. Modi said. He further
reiterated "This ferry service will not be restricted to this one route. We are planning to link other
places also through ferries.
Which of the following abrogate the intention of PM Modi towards development prospect of
Gujrat?
(I) According to a survey, before state election in Uttar Pradesh which was later won by BJP govt.,
some major project was launched but there was no progress in project after election.
(II) Many of the news channels have predicted, Congress will win the state assembly election.
(III) 5 MLA of BJP has joined Congress before the rally of PM Modi in Gujrat. (Marks 1)
A. Only II and III
B. Only I and III
C. Only III for more join
D. Only I
E. All of the above https://t.me/currentAffairscLuB
Your Answer: Not Attempted

Correct Answer: D. Only I <br>

https://store.adda247.com/#!/myTestAnalysis/printsolution/mappingId=25273/packageId=359/lang=ENGLISH 18/109
12/31/2017 Adda247 Store | Adda247 Store
Solution
Description: We have to choose that point which nulli es the intention of PM Modi for
development in Gujrat.
For I- It nulli es because it shows a negative side of BJP govt. as there was no development in
projects after winning election in UP.
For II- It has no direct contact with the theme which was mentioned above.
For III- It is out of the context of theme which was mentioned above.
Purchased by roshnipanwar025@gmail.com
Q20.
Study the following information carefully and answer the given questions. A word and number
arrangement machine when given an input line of words and numbers rearranges them following
a particular rule in each step. The following is an illustration of input and rearrangement.
Input: 14 use 65 robot 27 apple 12 train 25 20 lack melody 15 acute.
Step I: 12 zuse 14 65 robot 27 apple train 25 20 lack melody 15 acute
Step II: 14 atrain 12 zuse 65 robot 27 apple 25 20 lack melody 15 acute
Step III: 15 arobot 14 atrain 12 zuse 65 27 apple 25 20 lack melody acute
Step IV: 20 amelody 15 arobot 14 atrain 12 zuse 65 27 apple 25 lack acute
Step V: 20 amelody 15 arobot 14 atrain 12 zuse 65 27 apple 25 lack acute
Step VI: 25 alack 20 amelody 15 arobot 14 atrain 12 zuse 65 27 apple acute
Step VII: 27 zapple 25 alack 20 amelody 15 arobot 14 atrain 12 zuse 65 acute
Step VIII: 65 zacute 27 zapple 25 alack 20 amelody 15 arobot 14 atrain 12 zuse.
Step VIII is the last step of the above arrangement.
As per the rules followed in the steps given above, nd out in each of the following questions the
appropriate step for the given input.
Input: troy 25 beyond intense harm 16 upper 33 admit 23 10 20 shoot 11 out 8.
In which step are the elements ‘aharm zout abeyond zupper 11 33’ found in the trend order?
(Marks 1)
A. Fifth
B. Seventh
C. Fourth
D. Sixth
E. There is no such step for more join
Your Answer: Not Attempted
https://t.me/currentAffairscLuB
Correct Answer: E. There is no such step <br>

Solution
Number and letter both are arrange from left hand side together.
Pick a number in ascending order and word in descending order.
Number take as it is and if words rst letter is vowel than add ‘z’ before word and if word started
https://store.adda247.com/#!/myTestAnalysis/printsolution/mappingId=25273/packageId=359/lang=ENGLISH 19/109
12/31/2017 Adda247 Store | Adda247 Store
with consonant that add ‘a’. And so on.
Input: troy 25 beyond intense harm 16 upper 33 admit 23 10 20 shoot 11 out 8.
Step I: 8 zupper troy 25 beyond intense harm 16 33 admit 23 10 20 shoot 11 out
Step II: 10 atroy 8 zupper 25 beyond intense harm 16 33 admit 23 20 shoot 11 out
Step III: 11 ashoot 10 atroy 8 zupper 25 beyond intense harm 16 33 admit 23 20 out
Step IV: 16 zout 11 ashoot 10 atroy 8 zupper 25 beyond intense harm 33 admit 23 20
Step V: 20 zintense 16 zout 11 ashoot 10 atroy 8 zupper 25 beyond harm 33 admit 23
Step VI: 23 aharm 20 zintense 16 zout 11 ashoot 10 atroy 8 zupper 25 beyond 33 admit
Step VII: 25 abeyond 23 aharm 20 zintense 16 zout 11 ashoot 10 atroy 8 zupper 33 admit
Step VIII: 33 zadmit 25 abeyond 23 aharm 20 zintense 16 zout 11 ashoot 10 atroy 8 zupper
Step VIII is the last step of the above arrangement.
Purchased by roshnipanwar025@gmail.com
Q21.
Study the following information carefully and answer the given questions. A word and number
arrangement machine when given an input line of words and numbers rearranges them following
a particular rule in each step. The following is an illustration of input and rearrangement.
Input: 14 use 65 robot 27 apple 12 train 25 20 lack melody 15 acute.
Step I: 12 zuse 14 65 robot 27 apple train 25 20 lack melody 15 acute
Step II: 14 atrain 12 zuse 65 robot 27 apple 25 20 lack melody 15 acute
Step III: 15 arobot 14 atrain 12 zuse 65 27 apple 25 20 lack melody acute
Step IV: 20 amelody 15 arobot 14 atrain 12 zuse 65 27 apple 25 lack acute
Step V: 20 amelody 15 arobot 14 atrain 12 zuse 65 27 apple 25 lack acute
Step VI: 25 alack 20 amelody 15 arobot 14 atrain 12 zuse 65 27 apple acute
Step VII: 27 zapple 25 alack 20 amelody 15 arobot 14 atrain 12 zuse 65 acute
Step VIII: 65 zacute 27 zapple 25 alack 20 amelody 15 arobot 14 atrain 12 zuse.
Step VIII is the last step of the above arrangement.
As per the rules followed in the steps given above, nd out in each of the following questions the
appropriate step for the given input.
Input: troy 25 beyond intense harm 16 upper 33 admit 23 10 20 shoot 11 out 8.
How many words or numbers are there between ‘intense’ and ‘33’ as they appear in the third step
of the output? (Marks 1)
A. Six for more join
B. Three
C. Four https://t.me/currentAffairscLuB
D. Two
E. Seven

Your Answer: Not Attempted

Correct Answer: D. Two <br>


https://store.adda247.com/#!/myTestAnalysis/printsolution/mappingId=25273/packageId=359/lang=ENGLISH 20/109
12/31/2017 Adda247 Store | Adda247 Store

Solution
Number and letter both are arrange from left hand side together.
Pick a number in ascending order and word in descending order.
Number take as it is and if words rst letter is vowel than add ‘z’ before word and if word started
with consonant that add ‘a’. And so on.
Input: troy 25 beyond intense harm 16 upper 33 admit 23 10 20 shoot 11 out 8.
Step I: 8 zupper troy 25 beyond intense harm 16 33 admit 23 10 20 shoot 11 out
Step II: 10 atroy 8 zupper 25 beyond intense harm 16 33 admit 23 20 shoot 11 out
Step III: 11 ashoot 10 atroy 8 zupper 25 beyond intense harm 16 33 admit 23 20 out
Step IV: 16 zout 11 ashoot 10 atroy 8 zupper 25 beyond intense harm 33 admit 23 20
Step V: 20 zintense 16 zout 11 ashoot 10 atroy 8 zupper 25 beyond harm 33 admit 23
Step VI: 23 aharm 20 zintense 16 zout 11 ashoot 10 atroy 8 zupper 25 beyond 33 admit
Step VII: 25 abeyond 23 aharm 20 zintense 16 zout 11 ashoot 10 atroy 8 zupper 33 admit
Step VIII: 33 zadmit 25 abeyond 23 aharm 20 zintense 16 zout 11 ashoot 10 atroy 8 zupper
Step VIII is the last step of the above arrangement.
Purchased by roshnipanwar025@gmail.com
Q22.
Study the following information carefully and answer the given questions. A word and number
arrangement machine when given an input line of words and numbers rearranges them following
a particular rule in each step. The following is an illustration of input and rearrangement.
Input: 14 use 65 robot 27 apple 12 train 25 20 lack melody 15 acute.
Step I: 12 zuse 14 65 robot 27 apple train 25 20 lack melody 15 acute
Step II: 14 atrain 12 zuse 65 robot 27 apple 25 20 lack melody 15 acute
Step III: 15 arobot 14 atrain 12 zuse 65 27 apple 25 20 lack melody acute
Step IV: 20 amelody 15 arobot 14 atrain 12 zuse 65 27 apple 25 lack acute
Step V: 20 amelody 15 arobot 14 atrain 12 zuse 65 27 apple 25 lack acute
Step VI: 25 alack 20 amelody 15 arobot 14 atrain 12 zuse 65 27 apple acute
Step VII: 27 zapple 25 alack 20 amelody 15 arobot 14 atrain 12 zuse 65 acute
Step VIII: 65 zacute 27 zapple 25 alack 20 amelody 15 arobot 14 atrain 12 zuse.
Step VIII is the last step of the above arrangement.
As per the rules followed in the steps given above, nd out in each of the following questions the
for more join
appropriate step for the given input.
Input: troy 25 beyond intense harm 16 https://t.me/currentAffairscLuB
upper 33 admit 23 10 20 shoot 11 out 8.
How many steps will be required to complete the above input? (Marks 1)
A. Eight
B. Six
C. Five
D. Seven
E. None of these
https://store.adda247.com/#!/myTestAnalysis/printsolution/mappingId=25273/packageId=359/lang=ENGLISH 21/109
12/31/2017 Adda247 Store | Adda247 Store
Your Answer: Not Attempted

Correct Answer: A. Eight <br>

Solution
Number and letter both are arrange from left hand side together.
Pick a number in ascending order and word in descending order.
Number take as it is and if words rst letter is vowel than add ‘z’ before word and if word started
with consonant that add ‘a’. And so on.
Input: troy 25 beyond intense harm 16 upper 33 admit 23 10 20 shoot 11 out 8.
Step I: 8 zupper troy 25 beyond intense harm 16 33 admit 23 10 20 shoot 11 out
Step II: 10 atroy 8 zupper 25 beyond intense harm 16 33 admit 23 20 shoot 11 out
Step III: 11 ashoot 10 atroy 8 zupper 25 beyond intense harm 16 33 admit 23 20 out
Step IV: 16 zout 11 ashoot 10 atroy 8 zupper 25 beyond intense harm 33 admit 23 20
Step V: 20 zintense 16 zout 11 ashoot 10 atroy 8 zupper 25 beyond harm 33 admit 23
Step VI: 23 aharm 20 zintense 16 zout 11 ashoot 10 atroy 8 zupper 25 beyond 33 admit
Step VII: 25 abeyond 23 aharm 20 zintense 16 zout 11 ashoot 10 atroy 8 zupper 33 admit
Step VIII: 33 zadmit 25 abeyond 23 aharm 20 zintense 16 zout 11 ashoot 10 atroy 8 zupper
Step VIII is the last step of the above arrangement.
Purchased by roshnipanwar025@gmail.com
Q23.
Study the following information carefully and answer the given questions. A word and number
arrangement machine when given an input line of words and numbers rearranges them following
a particular rule in each step. The following is an illustration of input and rearrangement.
Input: 14 use 65 robot 27 apple 12 train 25 20 lack melody 15 acute.
Step I: 12 zuse 14 65 robot 27 apple train 25 20 lack melody 15 acute
Step II: 14 atrain 12 zuse 65 robot 27 apple 25 20 lack melody 15 acute
Step III: 15 arobot 14 atrain 12 zuse 65 27 apple 25 20 lack melody acute
Step IV: 20 amelody 15 arobot 14 atrain 12 zuse 65 27 apple 25 lack acute
Step V: 20 amelody 15 arobot 14 atrain 12 zuse 65 27 apple 25 lack acute
Step VI: 25 alack 20 amelody 15 arobot 14 atrain 12 zuse 65 27 apple acute
Step VII: 27 zapple 25 alack 20 amelody 15 arobot 14 atrain 12 zuse 65 acute
for more join
Step VIII: 65 zacute 27 zapple 25 alack 20 amelody 15 arobot 14 atrain 12 zuse.
https://t.me/currentAffairscLuB
Step VIII is the last step of the above arrangement.
As per the rules followed in the steps given above, nd out in each of the following questions the
appropriate step for the given input.
Input: troy 25 beyond intense harm 16 upper 33 admit 23 10 20 shoot 11 out 8.
Which of the following represents the position of ‘8’ in the last but one step? (Marks 1)
A. Ninth from the left end
B. Fourth from the right end
https://store.adda247.com/#!/myTestAnalysis/printsolution/mappingId=25273/packageId=359/lang=ENGLISH 22/109
12/31/2017 Adda247 Store | Adda247 Store
C. Thirteenth from the left end
D. Sixth from the right end
E. Both (B) and (C)
Your Answer: Not Attempted

Correct Answer: E. Both (B) and (C) <br>

Solution
Number and letter both are arrange from left hand side together.
Pick a number in ascending order and word in descending order.
Number take as it is and if words rst letter is vowel than add ‘z’ before word and if word started
with consonant that add ‘a’. And so on.
Input: troy 25 beyond intense harm 16 upper 33 admit 23 10 20 shoot 11 out 8.
Step I: 8 zupper troy 25 beyond intense harm 16 33 admit 23 10 20 shoot 11 out
Step II: 10 atroy 8 zupper 25 beyond intense harm 16 33 admit 23 20 shoot 11 out
Step III: 11 ashoot 10 atroy 8 zupper 25 beyond intense harm 16 33 admit 23 20 out
Step IV: 16 zout 11 ashoot 10 atroy 8 zupper 25 beyond intense harm 33 admit 23 20
Step V: 20 zintense 16 zout 11 ashoot 10 atroy 8 zupper 25 beyond harm 33 admit 23
Step VI: 23 aharm 20 zintense 16 zout 11 ashoot 10 atroy 8 zupper 25 beyond 33 admit
Step VII: 25 abeyond 23 aharm 20 zintense 16 zout 11 ashoot 10 atroy 8 zupper 33 admit
Step VIII: 33 zadmit 25 abeyond 23 aharm 20 zintense 16 zout 11 ashoot 10 atroy 8 zupper
Step VIII is the last step of the above arrangement.
Purchased by roshnipanwar025@gmail.com
Q24.
Study the following information carefully and answer the given questions. A word and number
arrangement machine when given an input line of words and numbers rearranges them following
a particular rule in each step. The following is an illustration of input and rearrangement.
Input: 14 use 65 robot 27 apple 12 train 25 20 lack melody 15 acute.
Step I: 12 zuse 14 65 robot 27 apple train 25 20 lack melody 15 acute
Step II: 14 atrain 12 zuse 65 robot 27 apple 25 20 lack melody 15 acute
Step III: 15 arobot 14 atrain 12 zuse 65 27 apple 25 20 lack melody acute
Step IV: 20 amelody 15 arobot 14 atrain 12 zuse 65 27 apple 25 lack acute
for more join
Step V: 20 amelody 15 arobot 14 atrain 12 zuse 65 27 apple 25 lack acute
Step VI: 25 alack 20 amelody 15 arobot https://t.me/currentAffairscLuB
14 atrain 12 zuse 65 27 apple acute
Step VII: 27 zapple 25 alack 20 amelody 15 arobot 14 atrain 12 zuse 65 acute
Step VIII: 65 zacute 27 zapple 25 alack 20 amelody 15 arobot 14 atrain 12 zuse.
Step VIII is the last step of the above arrangement.
As per the rules followed in the steps given above, nd out in each of the following questions the
appropriate step for the given input.
Input: troy 25 beyond intense harm 16 upper 33 admit 23 10 20 shoot 11 out 8.
https://store.adda247.com/#!/myTestAnalysis/printsolution/mappingId=25273/packageId=359/lang=ENGLISH 23/109
12/31/2017 Adda247 Store | Adda247 Store
In the fth step of the rearrangement, ‘23’ is related to ‘20’and ‘admit’ is related to ‘zintense’ in a
certain way. Which of the following would ‘11’ be related to, following the same pattern? (Marks 1)
A. upper
B. 50
C. out
D. 28
E. None of these
Your Answer: Not Attempted

Correct Answer: E. None of these <br>

Solution
Number and letter both are arrange from left hand side together.
Pick a number in ascending order and word in descending order.
Number take as it is and if words rst letter is vowel than add ‘z’ before word and if word started
with consonant that add ‘a’. And so on.
Input: troy 25 beyond intense harm 16 upper 33 admit 23 10 20 shoot 11 out 8.
Step I: 8 zupper troy 25 beyond intense harm 16 33 admit 23 10 20 shoot 11 out
Step II: 10 atroy 8 zupper 25 beyond intense harm 16 33 admit 23 20 shoot 11 out
Step III: 11 ashoot 10 atroy 8 zupper 25 beyond intense harm 16 33 admit 23 20 out
Step IV: 16 zout 11 ashoot 10 atroy 8 zupper 25 beyond intense harm 33 admit 23 20
Step V: 20 zintense 16 zout 11 ashoot 10 atroy 8 zupper 25 beyond harm 33 admit 23
Step VI: 23 aharm 20 zintense 16 zout 11 ashoot 10 atroy 8 zupper 25 beyond 33 admit
Step VII: 25 abeyond 23 aharm 20 zintense 16 zout 11 ashoot 10 atroy 8 zupper 33 admit
Step VIII: 33 zadmit 25 abeyond 23 aharm 20 zintense 16 zout 11 ashoot 10 atroy 8 zupper
Step VIII is the last step of the above arrangement.
Purchased by roshnipanwar025@gmail.com
Q25.
The current higher-levels of antibiotic-resistant bacteria, a threat to the very existence of mankind,
are attributed to the extensive use of antibiotics. Antibiotic resistance occurs when an antibiotic
has lost its ability to e ectively control or kill bacterial growth. Genetically, antibiotic resistance
spreads through bacteria populations both “vertically,” when new generations inherit antibiotic
for more join
resistance genes, and “horizontally,” when bacteria share or exchange sections of genetic material
with other bacteria. https://t.me/currentAffairscLuB
From the passage, it can be inferred that the e ectiveness of the extensive use of antibiotics can
be extended by doing which of the following, assuming that each is a realistic possibility? (Marks 2)
A. Gradually increasing the quantities of antibiotic used
B. Leaving mild infections untreated
C. Periodically switching the type of antibiotic used
D. Using only strong antibiotics
https://store.adda247.com/#!/myTestAnalysis/printsolution/mappingId=25273/packageId=359/lang=ENGLISH 24/109
12/31/2017 Adda247 Store | Adda247 Store
E. None of these
Your Answer: Not Attempted

Correct Answer: C. Periodically switching the type of antibiotic used <br>

Solution
Extensive use of antibiotics causes antibiotic resistance when an antibiotic loses its ability to kill or
control bacterial growth. This resistance spreads through bacteria population both vertically and
horizontally. What can be done to prolong the e ectiveness of antibiotics? It can be inferred that
the ongoing use of a 'particular antibiotic will not be e ective against future generations of
bacteria which have an innate resistance to that antibiotic. What would be e ective against them?
If a di erent antibiotic is used, bacteria resistant to one type of antibiotic could be killed. This can
continue as the antibiotics are changed. Option (a) will not help because the bacteria are already
resistant. Option (b) is irrelevant since use of antibiotics is a part of the argument, so leaving
infections untreated is not relevant to the argument. Option (c) can extend the e ectiveness of
antibiotics. Since developing resistance is applicable, to both strong and mild antibiotics, option (d)
is irrelevant.
Purchased by roshnipanwar025@gmail.com
Q26.
Study the information given below and answer the questions that follow:
Following are the criteria for promotion in company from Manager to President. The employee
must-
(i) Not be more than 45 years of age as on 10.02.2018.
(ii) Age must be greater than 32 years as on 10 February 2018.
(iii) Have at least 10 years of service in the same company out of which at least four years should
be Manager.
(iv) Not have any adverse remark in Con dential Report (C.R).
(v) Should have post-graduation with at least 55% of marks.
Based on the above criteria and the information given in each of the following questions, you have
to decide on the promotability of each case.
Aniket joined the Company 12 years back as Team leader. After 5 years, he was promoted
manager with two increments and was 30 years old as on 10.02.2014. He has no adverse remark
for more join
in his C.R. He has done MBA with 67% of marks. (Marks 2)
A. To be promoted https://t.me/currentAffairscLuB
B. Not to be promoted
C. Found CR adverse
D. Data inadequate
E. None of these
Your Answer: Not Attempted

https://store.adda247.com/#!/myTestAnalysis/printsolution/mappingId=25273/packageId=359/lang=ENGLISH 25/109
12/31/2017 Adda247 Store | Adda247 Store
Correct Answer: A. To be promoted <br>

Solution
All conditions are satis ed.
Purchased by roshnipanwar025@gmail.com
Q27.
Read the following information carefully and answer the questions given below:
Eight persons L, M, N, O, P, Q, R, S are working on an eight storey building where ground oor is
numbered as 1 and top oor is numbered as 8. Each person works on a di erent oor and also
ordered di erent snacks from the restaurant viz. Aloo tikki, Masala Vada, Noodles, Pizza, Pakoda,
Pav bhaji, Sweet corn, Samosa. They also drink di erent varieties of drink from CCD machine viz.
Tea, Co ee, Lemon Tea, and Soup. 3 persons drink tea, 2 persons drink co ee and soup each and
1 person drinks Lemon tea.
There are three oors in between the one who ordered Masala Vada and Q and both of them lives
on an odd numbered oor. L lives immediate above O. S drinks Tea. The one who ordered Pakoda
also drinks Soup. Both the persons, who drinks Co ee, works on two consecutive oors. N
ordered Sweet corn and lives below the one who ordered Samosa. P drinks neither Tea nor Co ee
and works on any oor below Q. The one who works on lowermost oor ordered Pav Bhaji. Only
the persons who work on even number oor drinks Tea. The one who works on 4th oor ordered
Pizza but did not drink Tea. Neither O nor L ordered Pizza. M drinks Soup and both the persons
who work on a oor which is immediately above and immediately below him drinks Tea. Samosa is
ordered with Co ee but not order by Q. Only one person lives between the one who ordered
Noodles and L.
Who among the following ordered Noodles? (Marks 2)
A. L
B. S
C. P
D. R
E. Q
Your Answer: Not Attempted

Correct Answer: B. S <br> for more join


Solution https://t.me/currentAffairscLuB
There are three oors in between the one who ordered Masala Vada and Q and both of them lives
on an odd numbered oor. The one who works on lowermost oor order Pav Bhaji. The one who
works on 4th oor ordered Pizza but did not drink Tea. Only the persons who works on even
number oor drinks Tea. Now, it is clear that the persons live on 2nd, 6th and 8th oor drinks Tea.
So, there can be three possible cases-

https://store.adda247.com/#!/myTestAnalysis/printsolution/mappingId=25273/packageId=359/lang=ENGLISH 26/109
12/31/2017 Adda247 Store | Adda247 Store

Now, P neither drink Tea nor Co ee and works on any oor below Q. So, Case: 3 will be eliminated
as P works on oor no.1. M drinks Soup and both the persons who work on a oor which is
immediately above and immediately below him drinks Tea. From this Case:2 will be eliminated.
Now, L lives immediate above O. Neither O nor L ordered Pizza. So, only possible condition will be
as shown below-

Now, Samosa is ordered with Co ee but not order by Q. So, it will be the person who works on 5th
oor who ordered Samosa. Both the persons, who drinks Co ee, works on two consecutive oors.
N ordered Sweet corn and lives below the one who ordered Samosa. So, N works on 2nd oor. S
drinks Tea so S works on 8th oor and remaining R will work on 4th oor. The one who ordered
Pakoda also drinks Soup.

So, rest P will drink lemon tea. Only one person lives between the one who ordered Noodles and
L. So, the nal arrangement will be-----

Purchased by roshnipanwar025@gmail.com
Q28.
Read the following information carefully and answer the questions given below:
Eight persons L, M, N, O, P, Q, R, S are working on an eight storey building where ground oor is
numbered as 1 and top oor is numbered as 8. Each person works on a di erent oor and also
ordered di erent snacks from the restaurant viz. Aloo tikki, Masala Vada, Noodles, Pizza, Pakoda,
Pav bhaji, Sweet corn, Samosa. They also drink di erent varieties of drink from CCD machine viz.
Tea, Co ee, Lemon Tea, and Soup. 3 persons drink tea, 2 persons drink co ee and soup each and
1 person drinks Lemon tea.
There are three oors in between the one who ordered Masala Vada and Q and both of them lives
on an odd numbered oor. L lives immediate above O. S drinks Tea. The one who ordered Pakoda
also drinks Soup. Both the persons, who drinks Co ee, works on two consecutive oors. N
ordered Sweet corn and lives below the one who ordered Samosa. P drinks neither Tea nor Co ee
and works on any oor below Q. The one who works on lowermost oor ordered Pav Bhaji. Only
the persons who work on even number oor drinks Tea. The one who works on 4th oor ordered
Pizza but did not drink Tea. Neither O nor L ordered Pizza. M drinks Soup and both the persons
for more join
who work on a oor which is immediately above and immediately below him drinks Tea. Samosa is
ordered with Co ee but not order by Q.https://t.me/currentAffairscLuB
Only one person lives between the one who ordered
Noodles and L.
Which of the following drink is taken with Aloo Tikki? (Marks 2)
A. Lemon Tea
B. Tea
C. Either Co ee or Lemon Tea
D. Soup
https://store.adda247.com/#!/myTestAnalysis/printsolution/mappingId=25273/packageId=359/lang=ENGLISH 27/109
12/31/2017 Adda247 Store | Adda247 Store
E. Data inadequate
Your Answer: Not Attempted

Correct Answer: B. Tea <br>

Solution
There are three oors in between the one who ordered Masala Vada and Q and both of them lives
on an odd numbered oor. The one who works on lowermost oor order Pav Bhaji. The one who
works on 4th oor ordered Pizza but did not drink Tea. Only the persons who works on even
number oor drinks Tea. Now, it is clear that the persons live on 2nd, 6th and 8th oor drinks Tea.
So, there can be three possible cases-

Now, P neither drink Tea nor Co ee and works on any oor below Q. So, Case: 3 will be eliminated
as P works on oor no.1. M drinks Soup and both the persons who work on a oor which is
immediately above and immediately below him drinks Tea. From this Case:2 will be eliminated.
Now, L lives immediate above O. Neither O nor L ordered Pizza. So, only possible condition will be
as shown below-

Now, Samosa is ordered with Co ee but not order by Q. So, it will be the person who works on 5th
oor who ordered Samosa. Both the persons, who drinks Co ee, works on two consecutive oors.
N ordered Sweet corn and lives below the one who ordered Samosa. So, N works on 2nd oor. S
drinks Tea so S works on 8th oor and remaining R will work on 4th oor. The one who ordered
Pakoda also drinks Soup.

So, rest P will drink lemon tea. Only one person lives between the one who ordered Noodles and
L. So, the nal arrangement will be-----

Purchased by roshnipanwar025@gmail.com
Q29.
Read the following information carefully and answer the questions given below:
Eight persons L, M, N, O, P, Q, R, S are working on an eight storey building where ground oor is
for more join
numbered as 1 and top oor is numbered as 8. Each person works on a di erent oor and also
https://t.me/currentAffairscLuB
ordered di erent snacks from the restaurant viz. Aloo tikki, Masala Vada, Noodles, Pizza, Pakoda,
Pav bhaji, Sweet corn, Samosa. They also drink di erent varieties of drink from CCD machine viz.
Tea, Co ee, Lemon Tea, and Soup. 3 persons drink tea, 2 persons drink co ee and soup each and
1 person drinks Lemon tea.
There are three oors in between the one who ordered Masala Vada and Q and both of them lives
on an odd numbered oor. L lives immediate above O. S drinks Tea. The one who ordered Pakoda
also drinks Soup. Both the persons, who drinks Co ee, works on two consecutive oors. N
https://store.adda247.com/#!/myTestAnalysis/printsolution/mappingId=25273/packageId=359/lang=ENGLISH 28/109
12/31/2017 Adda247 Store | Adda247 Store
ordered Sweet corn and lives below the one who ordered Samosa. P drinks neither Tea nor Co ee
and works on any oor below Q. The one who works on lowermost oor ordered Pav Bhaji. Only
the persons who work on even number oor drinks Tea. The one who works on 4th oor ordered
Pizza but did not drink Tea. Neither O nor L ordered Pizza. M drinks Soup and both the persons
who work on a oor which is immediately above and immediately below him drinks Tea. Samosa is
ordered with Co ee but not order by Q. Only one person lives between the one who ordered
Noodles and L.
R ordered which snack? (Marks 2)
A. Pakoda
B. Pav Bhaji
C. Noodles
D. Samosa
E. Pizza
Your Answer: Not Attempted

Correct Answer: E. Pizza <br>

Solution
There are three oors in between the one who ordered Masala Vada and Q and both of them lives
on an odd numbered oor. The one who works on lowermost oor order Pav Bhaji. The one who
works on 4th oor ordered Pizza but did not drink Tea. Only the persons who works on even
number oor drinks Tea. Now, it is clear that the persons live on 2nd, 6th and 8th oor drinks Tea.
So, there can be three possible cases-

Now, P neither drink Tea nor Co ee and works on any oor below Q. So, Case: 3 will be eliminated
as P works on oor no.1. M drinks Soup and both the persons who work on a oor which is
immediately above and immediately below him drinks Tea. From this Case:2 will be eliminated.
Now, L lives immediate above O. Neither O nor L ordered Pizza. So, only possible condition will be
as shown below-

for more join


Now, Samosa is ordered with Co ee but not order by Q. So, it will be the person who works on 5th
oor who ordered Samosa. Both the persons, who drinks Co ee, works on two consecutive oors.
https://t.me/currentAffairscLuB
N ordered Sweet corn and lives below the one who ordered Samosa. So, N works on 2nd oor. S
drinks Tea so S works on 8th oor and remaining R will work on 4th oor. The one who ordered
Pakoda also drinks Soup.

https://store.adda247.com/#!/myTestAnalysis/printsolution/mappingId=25273/packageId=359/lang=ENGLISH 29/109
12/31/2017 Adda247 Store | Adda247 Store
So, rest P will drink lemon tea. Only one person lives between the one who ordered Noodles and
L. So, the nal arrangement will be-----

Purchased by roshnipanwar025@gmail.com
Q30.
Read the following information carefully and answer the questions given below:
Eight persons L, M, N, O, P, Q, R, S are working on an eight storey building where ground oor is
numbered as 1 and top oor is numbered as 8. Each person works on a di erent oor and also
ordered di erent snacks from the restaurant viz. Aloo tikki, Masala Vada, Noodles, Pizza, Pakoda,
Pav bhaji, Sweet corn, Samosa. They also drink di erent varieties of drink from CCD machine viz.
Tea, Co ee, Lemon Tea, and Soup. 3 persons drink tea, 2 persons drink co ee and soup each and
1 person drinks Lemon tea.
There are three oors in between the one who ordered Masala Vada and Q and both of them lives
on an odd numbered oor. L lives immediate above O. S drinks Tea. The one who ordered Pakoda
also drinks Soup. Both the persons, who drinks Co ee, works on two consecutive oors. N
ordered Sweet corn and lives below the one who ordered Samosa. P drinks neither Tea nor Co ee
and works on any oor below Q. The one who works on lowermost oor ordered Pav Bhaji. Only
the persons who work on even number oor drinks Tea. The one who works on 4th oor ordered
Pizza but did not drink Tea. Neither O nor L ordered Pizza. M drinks Soup and both the persons
who work on a oor which is immediately above and immediately below him drinks Tea. Samosa is
ordered with Co ee but not order by Q. Only one person lives between the one who ordered
Noodles and L.
On which of the following oor does O work? (Marks 2)
A. 7th
B. 4th
C. 3rd
D. 5th
E. 2nd
Your Answer: Not Attempted

Correct Answer: D. 5th <br>


for more join
Solution
There are three https://t.me/currentAffairscLuB
oors in between the one who ordered Masala Vada and Q and both of them lives
on an odd numbered oor. The one who works on lowermost oor order Pav Bhaji. The one who
works on 4th oor ordered Pizza but did not drink Tea. Only the persons who works on even
number oor drinks Tea. Now, it is clear that the persons live on 2nd, 6th and 8th oor drinks Tea.
So, there can be three possible cases-

https://store.adda247.com/#!/myTestAnalysis/printsolution/mappingId=25273/packageId=359/lang=ENGLISH 30/109
12/31/2017 Adda247 Store | Adda247 Store
Now, P neither drink Tea nor Co ee and works on any oor below Q. So, Case: 3 will be eliminated
as P works on oor no.1. M drinks Soup and both the persons who work on a oor which is
immediately above and immediately below him drinks Tea. From this Case:2 will be eliminated.
Now, L lives immediate above O. Neither O nor L ordered Pizza. So, only possible condition will be
as shown below-

Now, Samosa is ordered with Co ee but not order by Q. So, it will be the person who works on 5th
oor who ordered Samosa. Both the persons, who drinks Co ee, works on two consecutive oors.
N ordered Sweet corn and lives below the one who ordered Samosa. So, N works on 2nd oor. S
drinks Tea so S works on 8th oor and remaining R will work on 4th oor. The one who ordered
Pakoda also drinks Soup.

So, rest P will drink lemon tea. Only one person lives between the one who ordered Noodles and
L. So, the nal arrangement will be-----

Purchased by roshnipanwar025@gmail.com
Q31.
Read the following information carefully and answer the questions given below:
Eight persons L, M, N, O, P, Q, R, S are working on an eight storey building where ground oor is
numbered as 1 and top oor is numbered as 8. Each person works on a di erent oor and also
ordered di erent snacks from the restaurant viz. Aloo tikki, Masala Vada, Noodles, Pizza, Pakoda,
Pav bhaji, Sweet corn, Samosa. They also drink di erent varieties of drink from CCD machine viz.
Tea, Co ee, Lemon Tea, and Soup. 3 persons drink tea, 2 persons drink co ee and soup each and
1 person drinks Lemon tea.
There are three oors in between the one who ordered Masala Vada and Q and both of them lives
on an odd numbered oor. L lives immediate above O. S drinks Tea. The one who ordered Pakoda
also drinks Soup. Both the persons, who drinks Co ee, works on two consecutive oors. N
ordered Sweet corn and lives below the one who ordered Samosa. P drinks neither Tea nor Co ee
and works on any oor below Q. The one who works on lowermost oor ordered Pav Bhaji. Only
the persons who work on even number oor drinks Tea. The one who works on 4th oor ordered
Pizza but did not drink Tea. Neither O nor L ordered Pizza. M drinks Soup and both the persons
who work on a oor which is immediately above and immediately below him drinks Tea. Samosa is
for more join
ordered with Co ee but not order by Q. Only one person lives between the one who ordered
Noodles and L. https://t.me/currentAffairscLuB
How many persons lives between the one who ordered Pakoda and the one who ordered Sweet
corn? (Marks 2)
A. Five
B. Six
C. Two
D. Three
https://store.adda247.com/#!/myTestAnalysis/printsolution/mappingId=25273/packageId=359/lang=ENGLISH 31/109
12/31/2017 Adda247 Store | Adda247 Store
E. None
Your Answer: Not Attempted

Correct Answer: E. None <br>

Solution
There are three oors in between the one who ordered Masala Vada and Q and both of them lives
on an odd numbered oor. The one who works on lowermost oor order Pav Bhaji. The one who
works on 4th oor ordered Pizza but did not drink Tea. Only the persons who works on even
number oor drinks Tea. Now, it is clear that the persons live on 2nd, 6th and 8th oor drinks Tea.
So, there can be three possible cases-

Now, P neither drink Tea nor Co ee and works on any oor below Q. So, Case: 3 will be eliminated
as P works on oor no.1. M drinks Soup and both the persons who work on a oor which is
immediately above and immediately below him drinks Tea. From this Case:2 will be eliminated.
Now, L lives immediate above O. Neither O nor L ordered Pizza. So, only possible condition will be
as shown below-

Now, Samosa is ordered with Co ee but not order by Q. So, it will be the person who works on 5th
oor who ordered Samosa. Both the persons, who drinks Co ee, works on two consecutive oors.
N ordered Sweet corn and lives below the one who ordered Samosa. So, N works on 2nd oor. S
drinks Tea so S works on 8th oor and remaining R will work on 4th oor. The one who ordered
Pakoda also drinks Soup.

So, rest P will drink lemon tea. Only one person lives between the one who ordered Noodles and
L. So, the nal arrangement will be-----

Purchased by roshnipanwar025@gmail.com
Q32.
Read the following passage carefully to answer the given questions:
Prime Minister Narendra Modi on Sunday inaugurated the rst phase of the Rs 615 crore ‘roll-on-
for more join
roll-o ’ (Ro-Ro) ferry service between Ghogha in Saurashtra and Dahej in south Gujarat. “This is
https://t.me/currentAffairscLuB
the rst of its kind project not only in India but also in South-East Asia,” Mr. Modi, a veteran leader
of BJP said at the rally which comes ahead of the Assembly polls in the State. “Imagine how much
time and petrol this ferry service will save. This ferry service connects Saurashtra and South
Gujarat… people from these regions frequently travels to and fro,” Mr. Modi said. He further
reiterated "This ferry service will not be restricted to this one route. We are planning to link other
places also through ferries.
What may be the repercussion of the Ferry service which was launched by PM Modi before the
https://store.adda247.com/#!/myTestAnalysis/printsolution/mappingId=25273/packageId=359/lang=ENGLISH 32/109
12/31/2017 Adda247 Store | Adda247 Store
state election?
(I) Ferry Service may give reward to BJP party in form of more seats.
(II) Opposition may incite public by referring ferry service as Election Lollypop.
(III) Opposition may declare something di erent in their election manifesto to entice public. (Marks
1)
A. Only I and III
B. Only II and III
C. Only II
D. Only I
E. All of these
Your Answer: Not Attempted

Correct Answer: E. All of these <br>

Solution
Description: For I- BJP may get bene t in number of seats as public will get bene tted from Ferry
service.
For II- Opposition may tell people, as it is only an election propaganda.
For III- Opposition will try something di erent to attract public towards their party.
Purchased by roshnipanwar025@gmail.com
Q33.
In each group of questions below are two conclusions followed by ve set of statements. You have
to choose the correct set of statements that logically satis es given conclusions. Assume the given
statements to be true even if they seem to be at variance from commonly known facts. Mark your
answer as,
Conclusions:
Some pen are not cat.
No Book is cat. A
ll the pen can be mobile.
All the bottle can be mobile.
Statements: (Marks 1)
A. Some pen are bottle. No bottle is cat. No cat is Book. All the Book are mobile. All the cat are
for more join
mobile.
https://t.me/currentAffairscLuB
B. All the Book are mobile. All the cat are mobile. Some pen are bottle. No bottle is cat. All cat are
Book.
C. No bottle is cat. No cat is Book. All the Book are mobile. All the cat are mobile. All bottle are pen.
No pen is mobile.
D. Some pen are bottle. No bottle is cat. Some cat are Book. All the Book are mobile. All the cat are
mobile.
E. No cat is Book. All the Book are mobile. All the cat are mobile. All pen are cat. All book are bottle.
https://store.adda247.com/#!/myTestAnalysis/printsolution/mappingId=25273/packageId=359/lang=ENGLISH 33/109
12/31/2017 Adda247 Store | Adda247 Store
Your Answer: Not Attempted

Correct Answer: A. Some pen are bottle. No bottle is cat. No cat is Book. All the Book are mobile.
All the cat are mobile. <br>

Solution

Purchased by roshnipanwar025@gmail.com
Q34.
In each group of questions below are two conclusions followed by ve set of statements. You have
to choose the correct set of statements that logically satis es given conclusions. Assume the given
statements to be true even if they seem to be at variance from commonly known facts. Mark your
answer as,
Conclusions:
No Cat is a Dog
Some pig is Dogs.
Statements:
Statements – 1: No Cat is cow. No cow is a Dog. Some Dogs are pig.
Statements – 2: All Cats are cows. All cows are Dog. No Dog is pig.
Statements – 3: All Cats are cows. No cow is a Dog. Some Dogs are pig.
Statements – 4: All Cats are cows. No cow is a Dog. No Dog is pig.
Statements – 5: All Cats are cows. All cows are Dog. Some Dogs are pig. (Marks 1)
A. Only Statements – 1
B. Only Statements – 2
C. Only Statements – 3
D. Only Statements – 4
E. Only Statements – 5
Your Answer: Not Attempted

Correct Answer: C. Only Statements – 3 <br>

Solution for more join


https://t.me/currentAffairscLuB
Purchased by roshnipanwar025@gmail.com
Q35.
The Election Commission’s decision to deploy the Voter Veri able Paper Audit Trail system for all
the constituencies in the Gujarat Assembly elections is questionable. This will be the rst time
VVPAT will be used on a State-wide basis. A State-wide voter paper trail may silence the EVM’s
critics, but is a regressive step.
Which of the following will be the repercussion of the given statement?
https://store.adda247.com/#!/myTestAnalysis/printsolution/mappingId=25273/packageId=359/lang=ENGLISH 34/109
12/31/2017 Adda247 Store | Adda247 Store
(I)The implementation of VVPAT was to have been undertaken by the EC in a phased manner, but
this blanket use appears to have been expedited after a series of unwarranted attacks on EVMs by
some political parties and scaremongers.
(II) A costly but useful complement to the Electronic Voting Machine, VVPAT will allow the voter to
verify her vote after registering it on the EVM, and the paper trail will allow for an audit of the
election results by the EC in a select and randomized number of constituencies.
(III)It would be wise for the EC to rapidly transit to third-generation, tamper-proof machines, which
must be thoroughly tested and vetted by experts before deployment. (Marks 1)
A. Only (III)
B. Only (II)
C. Only (I)
D. Both (II) and (III)
E. None of these
Your Answer: Not Attempted

Correct Answer: B. Only (II) <br>

Solution
For I- This statement could not be the repercussion of the given statement as it states about the
cause of using VVPAT instead of EVM.
For II-- This statement could be the repercussion of the given statement as it states about the
additional features given in VVPAT and its usefulness in a transparent voting mechanism.
For III- This statement could not be the repercussion of the given statement as it states about a
peace of advice related to the deployment of VVPAT.
Purchased by roshnipanwar025@gmail.com
Q36.
Each of the questions below, consist a question and two statements numbered I and II. You have
to decide whether the data provided in the statements are su cient to answer the question. Read
the three statements and Give answer-
Twelve persons named A to L are placed at a di erent hour division of a clock. What is the angle
between E and H?
I. B is at 7th hour division. E is opposite to K. L is at 60° from A. K is to the immediate left of H.
for more join
II. H is at 90° from C which is 60° from D. F is at 11th hour division. F is adjacent to K and J. G is at
30° from I. (Marks 1) https://t.me/currentAffairscLuB
A. The data in both statements I and II together are necessary to answer the question.
B. The data in statement II alone are su cient to answer the question while the data in statement
I alone are not su cient to answer the question.
C. The data even in both statements I and II together are not su cient to answer the question.
D. The data either in statement I alone or in statement II alone are su cient to answer the
question.
https://store.adda247.com/#!/myTestAnalysis/printsolution/mappingId=25273/packageId=359/lang=ENGLISH 35/109
12/31/2017 Adda247 Store | Adda247 Store
E. The data in statement I alone are su cient to answer the question, while the data in statement
II alone are not su cient to answer the question.
Your Answer: Not Attempted

Correct Answer: A. The data in both statements I and II together are necessary to answer the
question. <b> </b> <br>

Solution

Purchased by roshnipanwar025@gmail.com
Q37.
On the basis of the information given below, answer the following questions.
A, B, C, D, E and F are six persons, who belong to six di erent countries i.e. India, China, Germany,
France, Russia and Japan but not necessarily in the same order. Also each person likes di erent
subjects i.e. Physics, Chemistry, Economics, Medicine, Peace and Literature but not necessarily in
the same order.
They are sitting around a circular table facing the centre. The persons who like Medicine,
Economics and Peace are neither from Japan nor from India.
The persons who belong to France and China likes neither Medicine nor Literature. A is neither
from Japan nor sits on the immediate left of the person who likes Chemistry. The only person, who
is between E and F, likes Physics.
The one who likes Medicine is sitting opposite the person, who belongs to Russia. While the
person, who belongs to China sits on the immediate left of the person, who likes peace.
The person who is on the left side of the person from Japan does not like Literature. D likes
Chemistry and belongs to China and he is facing the person who likes Physics.
The one who likes Literature sits on the immediate right of the person, who belongs to Japan but
on the left of the person who likes Economics. C does not like Economics and F does not like
Medicine.
Who likes Chemistry? (Marks 1)
A. The person who is sitting to the immediate left of B
B. The person who is sitting to the immediate right of A
C. The person who is sitting opposite to the person, who likes Physics.
for more join
D. The person, who belongs to Japan.
E. Both (C) and (D) https://t.me/currentAffairscLuB
Your Answer: Not Attempted

Correct Answer: C. The person who is sitting opposite to the person, who likes Physics. <br>

https://store.adda247.com/#!/myTestAnalysis/printsolution/mappingId=25273/packageId=359/lang=ENGLISH 36/109
12/31/2017 Adda247 Store | Adda247 Store
Solution
From the given question, start from the point, the only person between E and F, has like Physics. It
is also clear that D likes Chemistry and he belongs to China and is facing the person who likes
Physics. It is also given that the person, who belongs to China, sits on the immediate left of the
person who likes peace.

Now, move on the next point – The one who likes Literature sits on the immediate right of the
person, who belongs to Japan but on the left of the person who likes Economics.
And it is also given that one who likes Medicine is sitting opposite the person, who belongs to
Russia. F does not like Medicine. So E will be on the immediate left of the one who likes Physics
and F will be on his immediate right.

Now, The next given point is that A is neither from Japan nor sits on the immediate left of the
person who likes Chemistry. So, A likes peace.
It is also given in the question,

And persons from France and China, does not like Medicine and Literature.
From the above two points, it is clear that the person, who belongs to Germany likes Medicine. The
person, who belongs to France, likes peace & the person, who belongs to India, likes literature.
From the given point, C does not like Economics.
So the nal arrangements are-

Purchased by roshnipanwar025@gmail.com
Q38.
On the basis of the information given below, answer the following questions.
A, B, C, D, E and F are six persons, who belong to six di erent countries i.e. India, China, Germany,
France, Russia and Japan but not necessarily in the same order. Also each person likes di erent
subjects i.e. Physics, Chemistry, Economics, Medicine, Peace and Literature but not necessarily in
the same order.
They are sitting around a circular table facing the centre. The persons who like Medicine,
Economics and Peace are neither from Japan nor from India.
for more join
The persons who belong to France and China likes neither Medicine nor Literature. A is neither
https://t.me/currentAffairscLuB
from Japan nor sits on the immediate left of the person who likes Chemistry. The only person, who
is between E and F, likes Physics.
The one who likes Medicine is sitting opposite the person, who belongs to Russia. While the
person, who belongs to China sits on the immediate left of the person, who likes peace.
The person who is on the left side of the person from Japan does not like Literature. D likes
Chemistry and belongs to China and he is facing the person who likes Physics.
The one who likes Literature sits on the immediate right of the person, who belongs to Japan but
https://store.adda247.com/#!/myTestAnalysis/printsolution/mappingId=25273/packageId=359/lang=ENGLISH 37/109
12/31/2017 Adda247 Store | Adda247 Store
on the left of the person who likes Economics. C does not like Economics and F does not like
Medicine.
The person who sits between E and D is? (Marks 1)
A. Second to the left of the person, who belongs to India.
B. Second to the right of the person, who belongs to Japan.
C. Fourth to the left of the person, who belongs to Russia.
D. Second to the right of the person, who likes literature.
E. None of these
Your Answer: Not Attempted

Correct Answer: C. Fourth to the left of the person, who belongs to Russia. <br>

Solution
From the given question, start from the point, the only person between E and F, has like Physics. It
is also clear that D likes Chemistry and he belongs to China and is facing the person who likes
Physics. It is also given that the person, who belongs to China, sits on the immediate left of the
person who likes peace.

Now, move on the next point – The one who likes Literature sits on the immediate right of the
person, who belongs to Japan but on the left of the person who likes Economics.
And it is also given that one who likes Medicine is sitting opposite the person, who belongs to
Russia. F does not like Medicine. So E will be on the immediate left of the one who likes Physics
and F will be on his immediate right.

Now, The next given point is that A is neither from Japan nor sits on the immediate left of the
person who likes Chemistry. So, A likes peace.
It is also given in the question,

And persons from France and China, does not like Medicine and Literature.
From the above two points, it is clear that the person, who belongs to Germany likes Medicine. The
for more join
person, who belongs to France, likes peace & the person, who belongs to India, likes literature.
From the given point, C does not like Economics.
https://t.me/currentAffairscLuB
So the nal arrangements are-

Purchased by roshnipanwar025@gmail.com
Q39.
On the basis of the information given below, answer the following questions.
A, B, C, D, E and F are six persons, who belong to six di erent countries i.e. India, China, Germany,
France, Russia and Japan but not necessarily in the same order. Also each person likes di erent
https://store.adda247.com/#!/myTestAnalysis/printsolution/mappingId=25273/packageId=359/lang=ENGLISH 38/109
12/31/2017 Adda247 Store | Adda247 Store
subjects i.e. Physics, Chemistry, Economics, Medicine, Peace and Literature but not necessarily in
the same order.
They are sitting around a circular table facing the centre. The persons who like Medicine,
Economics and Peace are neither from Japan nor from India.
The persons who belong to France and China likes neither Medicine nor Literature. A is neither
from Japan nor sits on the immediate left of the person who likes Chemistry. The only person, who
is between E and F, likes Physics.
The one who likes Medicine is sitting opposite the person, who belongs to Russia. While the
person, who belongs to China sits on the immediate left of the person, who likes peace.
The person who is on the left side of the person from Japan does not like Literature. D likes
Chemistry and belongs to China and he is facing the person who likes Physics.
The one who likes Literature sits on the immediate right of the person, who belongs to Japan but
on the left of the person who likes Economics. C does not like Economics and F does not like
Medicine.
Who is sitting on the immediate right of the one, who likes Literature? (Marks 1)
A. D
B. E
C. B
D. F
E. None of these
Your Answer: Not Attempted

Correct Answer: C. B <br>

Solution
From the given question, start from the point, the only person between E and F, has like Physics. It
is also clear that D likes Chemistry and he belongs to China and is facing the person who likes
Physics. It is also given that the person, who belongs to China, sits on the immediate left of the
person who likes peace.

for more join


Now, move on the next point – The one who likes Literature sits on the immediate right of the
person, who belongs to Japan but on the left of the person who likes Economics.
https://t.me/currentAffairscLuB
And it is also given that one who likes Medicine is sitting opposite the person, who belongs to
Russia. F does not like Medicine. So E will be on the immediate left of the one who likes Physics
and F will be on his immediate right.

Now, The next given point is that A is neither from Japan nor sits on the immediate left of the
person who likes Chemistry. So, A likes peace.

https://store.adda247.com/#!/myTestAnalysis/printsolution/mappingId=25273/packageId=359/lang=ENGLISH 39/109
12/31/2017 Adda247 Store | Adda247 Store
It is also given in the question,

And persons from France and China, does not like Medicine and Literature.
From the above two points, it is clear that the person, who belongs to Germany likes Medicine. The
person, who belongs to France, likes peace & the person, who belongs to India, likes literature.
From the given point, C does not like Economics.
So the nal arrangements are-

Purchased by roshnipanwar025@gmail.com
Q40.
On the basis of the information given below, answer the following questions.
A, B, C, D, E and F are six persons, who belong to six di erent countries i.e. India, China, Germany,
France, Russia and Japan but not necessarily in the same order. Also each person likes di erent
subjects i.e. Physics, Chemistry, Economics, Medicine, Peace and Literature but not necessarily in
the same order.
They are sitting around a circular table facing the centre. The persons who like Medicine,
Economics and Peace are neither from Japan nor from India.
The persons who belong to France and China likes neither Medicine nor Literature. A is neither
from Japan nor sits on the immediate left of the person who likes Chemistry. The only person, who
is between E and F, likes Physics.
The one who likes Medicine is sitting opposite the person, who belongs to Russia. While the
person, who belongs to China sits on the immediate left of the person, who likes peace.
The person who is on the left side of the person from Japan does not like Literature. D likes
Chemistry and belongs to China and he is facing the person who likes Physics.
The one who likes Literature sits on the immediate right of the person, who belongs to Japan but
on the left of the person who likes Economics. C does not like Economics and F does not like
Medicine.
The one who like Medicine sits? (Marks 1)
A. On the immediate left of the person, who belongs to France.
B. On the immediate right of the person, who belongs to Japan.
C. On the opposite of the person, who likes Economics.
D. On the opposite of the person, who belongs to Russia. for more join
E. Both (C) and (D)
https://t.me/currentAffairscLuB
Your Answer: Not Attempted

Correct Answer: E. Both (C) and (D) <br> <br>

https://store.adda247.com/#!/myTestAnalysis/printsolution/mappingId=25273/packageId=359/lang=ENGLISH 40/109
12/31/2017 Adda247 Store | Adda247 Store
Solution
From the given question, start from the point, the only person between E and F, has like Physics. It
is also clear that D likes Chemistry and he belongs to China and is facing the person who likes
Physics. It is also given that the person, who belongs to China, sits on the immediate left of the
person who likes peace.

Now, move on the next point – The one who likes Literature sits on the immediate right of the
person, who belongs to Japan but on the left of the person who likes Economics.
And it is also given that one who likes Medicine is sitting opposite the person, who belongs to
Russia. F does not like Medicine. So E will be on the immediate left of the one who likes Physics
and F will be on his immediate right.

Now, The next given point is that A is neither from Japan nor sits on the immediate left of the
person who likes Chemistry. So, A likes peace.
It is also given in the question,

And persons from France and China, does not like Medicine and Literature.
From the above two points, it is clear that the person, who belongs to Germany likes Medicine. The
person, who belongs to France, likes peace & the person, who belongs to India, likes literature.
From the given point, C does not like Economics.
So the nal arrangements are-

Purchased by roshnipanwar025@gmail.com
Q41.
On the basis of the information given below, answer the following questions.
A, B, C, D, E and F are six persons, who belong to six di erent countries i.e. India, China, Germany,
France, Russia and Japan but not necessarily in the same order. Also each person likes di erent
subjects i.e. Physics, Chemistry, Economics, Medicine, Peace and Literature but not necessarily in
the same order.
They are sitting around a circular table facing the centre. The persons who like Medicine,
Economics and Peace are neither from Japan nor from India.
for more join
The persons who belong to France and China likes neither Medicine nor Literature. A is neither
https://t.me/currentAffairscLuB
from Japan nor sits on the immediate left of the person who likes Chemistry. The only person, who
is between E and F, likes Physics.
The one who likes Medicine is sitting opposite the person, who belongs to Russia. While the
person, who belongs to China sits on the immediate left of the person, who likes peace.
The person who is on the left side of the person from Japan does not like Literature. D likes
Chemistry and belongs to China and he is facing the person who likes Physics.
The one who likes Literature sits on the immediate right of the person, who belongs to Japan but
https://store.adda247.com/#!/myTestAnalysis/printsolution/mappingId=25273/packageId=359/lang=ENGLISH 41/109
12/31/2017 Adda247 Store | Adda247 Store
on the left of the person who likes Economics. C does not like Economics and F does not like
Medicine.
Who is sitting on third to the left of second to the right of the person, who belongs to France?
(Marks 1)
A. B
B. A
C. C
D. E
E. None of these
Your Answer: Not Attempted

Correct Answer: E. None of these <br>

Solution
From the given question, start from the point, the only person between E and F, has like Physics. It
is also clear that D likes Chemistry and he belongs to China and is facing the person who likes
Physics. It is also given that the person, who belongs to China, sits on the immediate left of the
person who likes peace.

Now, move on the next point – The one who likes Literature sits on the immediate right of the
person, who belongs to Japan but on the left of the person who likes Economics.
And it is also given that one who likes Medicine is sitting opposite the person, who belongs to
Russia. F does not like Medicine. So E will be on the immediate left of the one who likes Physics
and F will be on his immediate right.

Now, The next given point is that A is neither from Japan nor sits on the immediate left of the
person who likes Chemistry. So, A likes peace.
It is also given in the question,

And persons from France and China, does not like Medicine and Literature.
for more join
From the above two points, it is clear that the person, who belongs to Germany likes Medicine. The
person, who belongs to France, likes peace & the person, who belongs to India, likes literature.
https://t.me/currentAffairscLuB
From the given point, C does not like Economics.
So the nal arrangements are-

Purchased by roshnipanwar025@gmail.com
Q42.
In the following questions, the symbols and % are used with the following meaning as illustrated
below.
https://store.adda247.com/#!/myTestAnalysis/printsolution/mappingId=25273/packageId=359/lang=ENGLISH 42/109
12/31/2017 Adda247 Store | Adda247 Store

In which of the following expression lead to, being de nitely true? (Marks 1)
A.
B.
C.
D.
E.
Your Answer: Not Attempted

Correct Answer: B. <img


src="https://d37ohbyy0w8xn8.cloudfront.net/1510661074737image5.png"> <br>

Solution
Only in this case de nitely true.
Purchased by roshnipanwar025@gmail.com
Q43.
The piece of hardware that converts your computer's digital signal to an analog signal that can
travel over Telephone lines is called a– (Marks 1)
A. Red wire
B. Blue cord
C. Tower
D. Modem
E. None of these
Your Answer: Not Attempted

Correct Answer: D. Modem <br>

Solution
Purchased by roshnipanwar025@gmail.com
Q44.
Study the information carefully and answer the questions given below.
1. Not be less than 24 years and more than 29 years as on 10th October, 2017.
for more join
2. Should have work experience of at least two years as an o cer.
3. Have secured at least 40 % marks in https://t.me/currentAffairscLuB
the written test.
4. Ful lls all other criteria except at (2) above but has a diploma in Sales his/her case is to be
referred to General Manager, sales.
5. Be a post-graduate with at least 50 % marks.
6. Ful lls all other criteria except at (3) above but has worked as sales O cer at least for three
years, his/her case is to be referred to Director, Marketing.
(A) Mrs. Swati was born on 5th June, 1990.
https://store.adda247.com/#!/myTestAnalysis/printsolution/mappingId=25273/packageId=359/lang=ENGLISH 43/109
12/31/2017 Adda247 Store | Adda247 Store
(B) Done her post-graduation in sales with rst class.
(C) She has secured 35 % marks in the written Test.
(D) She has been working in an organization as a sales O cer for the last four years.
If Swati ful lls all these criteria then from which of the following point it can be concluded that she
is not selected for the Job? (Marks 1)
A. Only A
B. Only B
C. Only C
D. Only D
E. None of these
Your Answer: Not Attempted

Correct Answer: C. Only C <br>

Solution
Only (c) as she should have secured at least 40 % marks in the written test but she has secured 35
% marks in the written Test.
Purchased by roshnipanwar025@gmail.com
Q45.
Patents refer to the protection of the idea behind an innovation gets whereas copyright refers to
the protection the expression of the idea gets. Computer programmes are unique in the sense
that they get both these protections. In order to get both the protections it is necessary that the
idea behind an innovation and its expression are distinguishable.
Which of the following can be inferred from the statements above? (Marks 1)
A. A patent is only as di cult to obtain as a copyright.
B. A computer programme writer is the inventor of the idea of the programme.
C. The idea behind a computer programme and its expression are distinguishable.
D. In products other than computer programmes, the idea behind them and the expression of the
idea are indistinguishable.
E. None of these
Your Answer: Not Attempted
for more join
Correct Answer: C. The idea behind a computer programme and its expression are
distinguishable. <br> https://t.me/currentAffairscLuB
Solution
As computer programmes are said to have both patent and copyright protection, (c) is the
appropriate choice. (a) is not an inference from the statement (b) is what is known generally and
not an inference. While (c) must be true, the converse (d) is not a de nite inference from the
statement.
https://store.adda247.com/#!/myTestAnalysis/printsolution/mappingId=25273/packageId=359/lang=ENGLISH 44/109
12/31/2017 Adda247 Store | Adda247 Store
Purchased by roshnipanwar025@gmail.com
Q46.
Raju, Rancho and Farhan enter into a business plan with their investments in the ratio 5 : 7 : 9
respectively. After 3 months, Raju withdraws his whole amount, Rancho withdraws half of his
amount and Farhan triples his amount. After another 6 months, Farhan withdraws 8/9 of his
amount, Rancho doubles his amount and Raju re-enters into business with an amount equal to his
initial amount.
The pro t at the end of year is Rs 11737. If pro t is shared among all of them in ratio of their
investments, then the pro t of Raju is how much less than that of Rancho? (Marks 2)
A. Rs 2861
B. Rs 984
C. Rs 1331
D. Rs 4187
E. None of these
Your Answer: Not Attempted

Correct Answer: C. Rs 1331 <br>

Solution

Purchased by roshnipanwar025@gmail.com
Q47.
Raju, Rancho and Farhan enter into a business plan with their investments in the ratio 5 : 7 : 9
respectively. After 3 months, Raju withdraws his whole amount, Rancho withdraws half of his
amount and Farhan triples his amount. After another 6 months, Farhan withdraws 8/9 of his
amount, Rancho doubles his amount and Raju re-enters into business with an amount equal to his
initial amount.
All of them maintain their amounts at the end of 1st year for another 12 months. What is the
share of Rancho if the overall pro t for only 2nd year is Rs. 22500? (Marks 1)
A. 10000
B. 11250
C. 9725
D. 12050
for more join
E. None of these https://t.me/currentAffairscLuB
Your Answer: Not Attempted

Correct Answer: E. None of these <br>

Solution

https://store.adda247.com/#!/myTestAnalysis/printsolution/mappingId=25273/packageId=359/lang=ENGLISH 45/109
12/31/2017 Adda247 Store | Adda247 Store
Purchased by roshnipanwar025@gmail.com
Q48.
To answer the following questions, which of the information given in the Statements (A), (B), (C)
and (D) below is/are necessary/ su cient?
How much minimum marks is required to pass an examination?
A. Student A secured 38% marks in the examination and failed by 8 marks. Student B secured 42%
marks in the same examination and got 12 more than the minimum pass marks.
B. Student A secured 35% of the total marks in the examination and failed by 23 marks. If he had
secured 25 more marks his percentage of marks would have been 40%.
C. A student will be declared passed if he secures 39.60% of the total marks. (Marks 1)
A. Any two of them
B. Only A
C. C and either A or B
D. Either A or B only
E. Any of these
Your Answer: Not Attempted

Correct Answer: D. Either A or B only <br>

Solution

Purchased by roshnipanwar025@gmail.com
Q49.
To answer the following questions, which of the information given in the Statements (A), (B), (C)
and (D) below is/are necessary/ su cient?
A company has two kinds of employees—supervisors and clerks. The total monthly salary of the
employees is Rs 285000. What is the total number of employees in that company?
A. The ratio of the no. of supervisors to that of clerks in the company is 4 : 5.
B. The total monthly salary of all the supervisors is 28% more than that of clerks.
C. 20% of the clerks’ monthly salary is Rs 25000. (Marks 1)
A. Only A and B together
B. Only A and C together
C. Only C
for more join
D. All statements are required https://t.me/currentAffairscLuB
E. Question can’t be answered even after using all the information
Your Answer: Not Attempted

Correct Answer: E. Question can’t be answered even after using all the information <br>

https://store.adda247.com/#!/myTestAnalysis/printsolution/mappingId=25273/packageId=359/lang=ENGLISH 46/109
12/31/2017 Adda247 Store | Adda247 Store
Solution

Purchased by roshnipanwar025@gmail.com
Q50.
To answer the following questions, which of the information given in the Statements (A), (B), (C)
and (D) below is/are necessary/ su cient?
Ashok earned a pro t of Rs 10,000 by investing Rs 1,50,000 in a business for one year. If the pro t
earned in the business is proportional to the investment and the period of investment, what
would have been the share of Saket in the total pro t at the end of one year, if Saket and Sudhir
also joined Ashok in the business?
A. Sudhir’s investment was for 9 months.
B. Saket’s investment was thrice that of Ashok.
C. Total pro t earned by the three at the end of one year was Rs 55,000.
D. Sudhir’s investment was two-thirds of that of Ashok. (Marks 1)
A. A, B and D together are su cient
B. A, C and D together are su cient
C. All A, B, C and D together are su cient
D. B, C and D together are su cient
E. All A, B, C and D together are not su cient
Your Answer: Not Attempted

Correct Answer: C. All A, B, C and D together are su cient <br>

Solution

Purchased by roshnipanwar025@gmail.com
Q51.
To answer the following questions, which of the information given in the Statements (A), (B), (C)
and (D) below is/are necessary/ su cient?
Rajnish buys 30 books and 65 pens. If price of each book is more than price of each pen then what
money does he have to pay for this?
A. At a pro t of 20% he sells all the objects for Rs 3828.
B. The CP of one book and one pen is Rs 90.
for more join
C. The di erence of sum and di erencehttps://t.me/currentAffairscLuB
of buying price of one pen and one book is Rs 28. (Marks
1)
A. Only A alone is su cient
B. B and C together are su cient
C. A alone or B and C together are su cient
D. All together are necessary
E. All even together are not su cient
https://store.adda247.com/#!/myTestAnalysis/printsolution/mappingId=25273/packageId=359/lang=ENGLISH 47/109
12/31/2017 Adda247 Store | Adda247 Store
Your Answer: Not Attempted

Correct Answer: C. A alone or B and C together are su cient <br>

Solution

Purchased by roshnipanwar025@gmail.com
Q52.
To answer the following questions, which of the information given in the Statements (A), (B), (C)
and (D) below is/are necessary/ su cient?
X, Y and Z secured 45%, 50% and 60% marks respectively in Biology. W’s marks in Biology is 12.5
more than X’s marks and 4 less than Z’s marks. Find out the individual marks of four students.
A. For the students total marks obtained for Biology is 311.5.
B. Total of W’s and X’s marks in Biology is 147.5. (Marks 1)
A. A and B together
B. Only C
C. A and either B or C
D. All together
E. None of the above
Your Answer: Not Attempted

Correct Answer: E. None of the above <br>

Solution

Purchased by roshnipanwar025@gmail.com
Q53.
Study the graph and table given below and answer the following questions. The line graph shows
the listed price per kg of various items in a wholesale store.

The table given below shows the amount of items bought by a retailer from the wholesale store.
The table also shows the discount % o ered by the wholesaler on the list price and total cost
incurred by the retailer.
for more join
https://t.me/currentAffairscLuB
Calculate the pro t earned by retailer on selling 20 kgs of wheat purchased by him to a customer
at a discount of 5% on the listed price ? (Marks 2)
A. Rs. 25
B. Rs. 45
C. Rs. 75
D. Rs. 50
https://store.adda247.com/#!/myTestAnalysis/printsolution/mappingId=25273/packageId=359/lang=ENGLISH 48/109
12/31/2017 Adda247 Store | Adda247 Store
E. None of these
Your Answer: Not Attempted

Correct Answer: A. Rs. 25 <br>

Solution

Purchased by roshnipanwar025@gmail.com
Q54.
Study the graph and table given below and answer the following questions. The line graph shows
the listed price per kg of various items in a wholesale store.

The table given below shows the amount of items bought by a retailer from the wholesale store.
The table also shows the discount % o ered by the wholesaler on the list price and total cost
incurred by the retailer.

The retailer sold all the cashew bought by him to a customer at a price 25% more than the listed
price. Calculate his overall pro t percent. (Marks 2)
A. 33.33%
B. 66.66%
C. 55.55%
D. 42.64%
E. 77.77%
Your Answer: Not Attempted

Correct Answer: B. 66.66% <br>

Solution

Purchased by roshnipanwar025@gmail.com
Q55.
Study the graph and table given below and answer the following questions. The line graph shows
for more join
the listed price per kg of various items in a wholesale store.
https://t.me/currentAffairscLuB
The table given below shows the amount of items bought by a retailer from the wholesale store.
The table also shows the discount % o ered by the wholesaler on the list price and total cost
incurred by the retailer.

If 50% of the rice bought by the retailer got spoiled, then at what price/kg must he sell the
remaining amount of rice to be at a situation of no loss-no gain ? (Marks 2)
https://store.adda247.com/#!/myTestAnalysis/printsolution/mappingId=25273/packageId=359/lang=ENGLISH 49/109
12/31/2017 Adda247 Store | Adda247 Store
A. Rs. 40
B. Rs. 19
C. Rs. 27
D. Rs. 22
E. None of these
Your Answer: Not Attempted

Correct Answer: C. Rs. 27 <br>

Solution

Cost price of Rice for Retailer = 20×15×90/100


= Rs. 270
To be in a situation of no loss –no gain, he must sell remaining 50% at Rs. 270.
Price per kg = 270/10= Rs. 27
Purchased by roshnipanwar025@gmail.com
Q56.
Study the graph and table given below and answer the following questions. The line graph shows
the listed price per kg of various items in a wholesale store.

The table given below shows the amount of items bought by a retailer from the wholesale store.
The table also shows the discount % o ered by the wholesaler on the list price and total cost
incurred by the retailer.

The retailer sold all the pulses he bought at a price that is 30% more than the listed price and
o ered 2 kgs of Almond free with it. Find overall pro t% of the retailer in this bargain ?
(approximate) (Marks 2)
A. 50%
B. 40%
C. 35%
D. 61%
E. 45% for more join
Your Answer: Not Attempted
https://t.me/currentAffairscLuB
Correct Answer: D. 61% <br>

Solution

Purchased by roshnipanwar025@gmail.com
Q57.
https://store.adda247.com/#!/myTestAnalysis/printsolution/mappingId=25273/packageId=359/lang=ENGLISH 50/109
12/31/2017 Adda247 Store | Adda247 Store
Study the graph and table given below and answer the following questions. The line graph shows
the listed price per kg of various items in a wholesale store.

The table given below shows the amount of items bought by a retailer from the wholesale store.
The table also shows the discount % o ered by the wholesaler on the list price and total cost
incurred by the retailer.

The retailer mixed 6 kgs. of impurity (free of cost) with all the sugar he had and sold the mixture at
a discount which is 25% less than that discount (in percentage) o ered by the wholesaler. Find the
pro t % on the sale of all of the amount of this mixture? (Marks 2)
A. 52.50%
B. 46.15%
C. 48.75%
D. 57.50%
E. None of these
Your Answer: Not Attempted

Correct Answer: C. 48.75% <br>

Solution

Purchased by roshnipanwar025@gmail.com
Q58.
Study the folling case and answer the following questions based on it.

George has recently acquired shares of four companies, namely Asian Paintz (AZ), BMZ, ChaeWoo
(CW) and Dataman (DT). The nancial results of these companies for the nancial year ended
2002-03 revealed these interesting facts: Pro ts of AZ were 10 per cent of its sales, while the
pro ts of BMZ were 20 per cent of its sales. While the pro ts of CW and DT were the same, the
sales of CW were the same as those of BMZ. The total expenses of CW were 400 per cent more
than its pro ts while the sales of DT were 200 per cent more than its pro ts. The total expenses of
CW were Rs. 10 million and the total expenses of CW were 11.11 per cent more than those of AZ.
for more join
If pro ts of DT and BMZ were to be exchanged, what would be the ratio of the pro t percentage
(expressed as a percentage of sales) forhttps://t.me/currentAffairscLuB
these two companies ? (Marks 2)
A. 7 : 13
B. 5 : 12
C. 12 : 5
D. 5 : 13
E. None of these
Your Answer: Not Attempted
https://store.adda247.com/#!/myTestAnalysis/printsolution/mappingId=25273/packageId=359/lang=ENGLISH 51/109
12/31/2017 Adda247 Store | Adda247 Store

Correct Answer: C. 12 : 5 <br>

Solution

following data table can be formed from the information provided.

The respective pro t percentages would have been 40% and 16.66% — a ratio of 12 : 5.
Purchased by roshnipanwar025@gmail.com
Q59.
Study the folling case and answer the following questions based on it.

George has recently acquired shares of four companies, namely Asian Paintz (AZ), BMZ, ChaeWoo
(CW) and Dataman (DT). The nancial results of these companies for the nancial year ended
2002-03 revealed these interesting facts: Pro ts of AZ were 10 per cent of its sales, while the
pro ts of BMZ were 20 per cent of its sales. While the pro ts of CW and DT were the same, the
sales of CW were the same as those of BMZ. The total expenses of CW were 400 per cent more
than its pro ts while the sales of DT were 200 per cent more than its pro ts. The total expenses of
CW were Rs. 10 million and the total expenses of CW were 11.11 per cent more than those of AZ.
If next year, the pro ts of AZ were to equal CW’s current pro t, then with an increase of 12.5 per
cent in sales, what would be the pro t of AZ expressed as a percentage of sales next year ? (Marks
2)
A. 17.77 per cent
B. 22.22 per cent
C. 18.88 per cent
D. None
E. None of these
Your Answer: Not Attempted

Correct Answer: A. 17.77 per cent <br>

Solution for more join


https://t.me/currentAffairscLuB
following data table can be formed from the information provided.

2/11.25 = 17.77%
Purchased by roshnipanwar025@gmail.com
Q60.
Study the folling case and answer the following questions based on it.

https://store.adda247.com/#!/myTestAnalysis/printsolution/mappingId=25273/packageId=359/lang=ENGLISH 52/109
12/31/2017 Adda247 Store | Adda247 Store
George has recently acquired shares of four companies, namely Asian Paintz (AZ), BMZ, ChaeWoo
(CW) and Dataman (DT). The nancial results of these companies for the nancial year ended
2002-03 revealed these interesting facts: Pro ts of AZ were 10 per cent of its sales, while the
pro ts of BMZ were 20 per cent of its sales. While the pro ts of CW and DT were the same, the
sales of CW were the same as those of BMZ. The total expenses of CW were 400 per cent more
than its pro ts while the sales of DT were 200 per cent more than its pro ts. The total expenses of
CW were Rs. 10 million and the total expenses of CW were 11.11 per cent more than those of AZ.
Which company had the lowest total expenses ? (Marks 2)
A. AZ
B. BMZ
C. CW
D. DT
E. None of these
Your Answer: Not Attempted

Correct Answer: D. DT <br>

Solution

following data table can be formed from the information provided.

Lowest total expense = 4 million for DT.


Purchased by roshnipanwar025@gmail.com
Q61.
Study the folling case and answer the following questions based on it.

George has recently acquired shares of four companies, namely Asian Paintz (AZ), BMZ, ChaeWoo
(CW) and Dataman (DT). The nancial results of these companies for the nancial year ended
2002-03 revealed these interesting facts: Pro ts of AZ were 10 per cent of its sales, while the
pro ts of BMZ were 20 per cent of its sales. While the pro ts of CW and DT were the same, the
sales of CW were the same as those of BMZ. The total expenses of CW were 400 per cent more
than its pro ts while the sales of DT were 200 per cent more than its pro ts. The total expenses of
for more join
CW were Rs. 10 million and the total expenses of CW were 11.11 per cent more than those of AZ.
Which company had the lowest pro ts https://t.me/currentAffairscLuB
? (Marks 2)
A. AZ
B. BMZ
C. CW
D. DT
E. None of these
Your Answer: Not Attempted
https://store.adda247.com/#!/myTestAnalysis/printsolution/mappingId=25273/packageId=359/lang=ENGLISH 53/109
12/31/2017 Adda247 Store | Adda247 Store

Correct Answer: A. AZ <br>

Solution

following data table can be formed from the information provided.

Lowest pro ts = 1 million for AZ.


Purchased by roshnipanwar025@gmail.com
Q62.
Study the folling case and answer the following questions based on it.

George has recently acquired shares of four companies, namely Asian Paintz (AZ), BMZ, ChaeWoo
(CW) and Dataman (DT). The nancial results of these companies for the nancial year ended
2002-03 revealed these interesting facts: Pro ts of AZ were 10 per cent of its sales, while the
pro ts of BMZ were 20 per cent of its sales. While the pro ts of CW and DT were the same, the
sales of CW were the same as those of BMZ. The total expenses of CW were 400 per cent more
than its pro ts while the sales of DT were 200 per cent more than its pro ts. The total expenses of
CW were Rs. 10 million and the total expenses of CW were 11.11 per cent more than those of AZ.
What is the ratio of the lowest and highest pro t ? (Marks 2)
A. 12 : 5
B. 7 : 18
C. 5 : 12
D. 12 : 23
E. None of these
Your Answer: Not Attempted

Correct Answer: C. 5 : 12 <br>

Solution

following data table can be formed from the information provided.


for more join
The required ratio is 1 : 2.4 = 5 : 12. https://t.me/currentAffairscLuB
Purchased by roshnipanwar025@gmail.com
Q63.
Ramu and Shyamu decide to sell their cars each at Rs.
36,000. While Ramu decided to give a discount of 8% on the rst Rs. 8000, 5% on next Rs. 12000
and 3% on the rest to buyer Shashi. Shyamu decided to give a discount of 7% on the rst 12000,
6% on next 8000 and 5% on the rest to buyer Rajesh. These discounts were, however, subject to
https://store.adda247.com/#!/myTestAnalysis/printsolution/mappingId=25273/packageId=359/lang=ENGLISH 54/109
12/31/2017 Adda247 Store | Adda247 Store
the buyers making the payment on time failing which the discount gets reduced by 1% for every
delay of a week. In each case, the selling price of 36,000 was arrived at by increasing the cost price
by 25%.
If each of them got the payments on time, what is the approximate percentage pro t of the
person getting the higher pro t ? (Marks 2)
A. 19%
B. 21%
C. 25%
D. 17%
E. 20%
Your Answer: Not Attempted

Correct Answer: A. 19% <br>

Solution

Purchased by roshnipanwar025@gmail.com
Q64.
Ramu and Shyamu decide to sell their cars each at Rs.
36,000. While Ramu decided to give a discount of 8% on the rst Rs. 8000, 5% on next Rs. 12000
and 3% on the rest to buyer Shashi. Shyamu decided to give a discount of 7% on the rst 12000,
6% on next 8000 and 5% on the rest to buyer Rajesh. These discounts were, however, subject to
the buyers making the payment on time failing which the discount gets reduced by 1% for every
delay of a week. In each case, the selling price of 36,000 was arrived at by increasing the cost price
by 25%.
If Shashi defaults by 1 and 2 weeks in the second and third payments respectively, what would be
the pro t of Ramu in the sale of the car ? (Marks 2)
A. Rs. 5920
B. Rs. 6240
C. Rs. 5860 for more join
D. Rs. 5980
E. None of these https://t.me/currentAffairscLuB
Your Answer: Not Attempted

Correct Answer: A. Rs. 5920 <br>

https://store.adda247.com/#!/myTestAnalysis/printsolution/mappingId=25273/packageId=359/lang=ENGLISH 55/109
12/31/2017 Adda247 Store | Adda247 Store
Solution

Purchased by roshnipanwar025@gmail.com
Q65.
Amit intended to travel a certain distance at a certain uniform speed. But after one hour, he
increased his speed by 25%. As a result, in the remaining part of the time that he originally
planned for the journey, he could now cover as much distance as he initially thought he would be
able to cover.
What is the total time taken for the journey ? (Marks 1)
A. 4 hrs.
B. 5 hrs
C. 6 hrs
D. Can’t be determined
E. None of these
Your Answer: Not Attempted

Correct Answer: B. 5 hrs <br>

Solution

Purchased by roshnipanwar025@gmail.com
Q66.
Amit intended to travel a certain distance at a certain uniform speed. But after one hour, he
increased his speed by 25%. As a result, in the remaining part of the time that he originally
planned for the journey, he could now cover as much distance as he initially thought he would be
able to cover.
After Amit increased his speed, if he decided to terminate his journey after covering the distance
he initially intended to cover and not cover the extra distance as given in the data, what is the total
time taken for the journey ? (Marks 1)
A. 4 hr 12 min.
B. 5 hr 24 min.
C. 3 hr 36 min.
for more join
D. 4 hr 36 min. https://t.me/currentAffairscLuB
E. None of these
Your Answer: Not Attempted

Correct Answer: A. 4 hr 12 min. <br>

https://store.adda247.com/#!/myTestAnalysis/printsolution/mappingId=25273/packageId=359/lang=ENGLISH 56/109
12/31/2017 Adda247 Store | Adda247 Store
Solution

Purchased by roshnipanwar025@gmail.com
Q67.
Quantity 1: Time taken by Manoj and Shubham to complete the work together. When Manoj works
alone, he takes 4.8 hrs more than the time taken by Manoj and Shubham to complete the work
together. When Shubham works alone, he takes 10.8 hrs more than the time taken by both of
them to complete the work together.
Quantity 2: 7.4 hrs. (Marks 2)
A. Quantity I > Quantity II
B. Quantity I < Quantity II
C. Quantity I ≥ Quantity II
D. Quantity I ≤ Quantity II
E. Quantity I = Quantity II or No relation

Your Answer: Not Attempted

Correct Answer: B. Quantity I &lt; Quantity II <br>

Solution

Let, Manoj and Shubham take ‘x’ hrs. to complete the work working together.
Let, Manoj does ‘m’ units per hour.

Purchased by roshnipanwar025@gmail.com
Q68.
Quantity 1: Pro t percent for a shopkeeper. The shopkeeper uses a faulty scale while buying cloth
from wholesaler. His scale measures 10% extra than the actual measurement. While selling, the
shopkeeper gives 10% less and o ers a discount of 10% to the customer.
Quantity 2: Pro tpercent for a man who calculates his pro t on selling price. He marks up the cost
of his item by 25% and then sells the item at a discount of 10%. (Marks 2)
A. Quantity I > Quantity II
B. Quantity I < Quantity II
for more join
C. Quantity I ≥ Quantity II https://t.me/currentAffairscLuB
D. Quantity I ≤ Quantity II
E. Quantity I = Quantity II or No relation

Your Answer: Not Attempted

Correct Answer: B. Quantity I &lt; Quantity II <br>


https://store.adda247.com/#!/myTestAnalysis/printsolution/mappingId=25273/packageId=359/lang=ENGLISH 57/109
12/31/2017 Adda247 Store | Adda247 Store

Solution

Purchased by roshnipanwar025@gmail.com
Q69.
Quantity 1: Original duration of ight. In a ight of 3000 km an aircraft was slowed down by bad
weather. Its average speed for the trip was reduced by 100 km/hr. and the time increased by one
hour.
Quantity 2: Usual time of a man who, when walks at 3/4 th of his usual pace, reaches his o ce 20
minutes late. (Marks 2)
A. Quantity I > Quantity II
B. Quantity I < Quantity II
C. Quantity I ≥ Quantity II
D. Quantity I ≤ Quantity II
E. Quantity I = Quantity II or No relation

Your Answer: Not Attempted

Correct Answer: A. Quantity I &gt; Quantity II <br>

Solution

Purchased by roshnipanwar025@gmail.com
Q70.
Wheels of diameters 7 cm and 14 cm start rolling simultaneously from X and Y, which are 1990.50
cm apart, towards each other in opposite directions. Both of them make same no. of revolutions
per second. Both of them meet after 10s.
Quantity 1: speed of smaller wheel.
Quantity 2: 21π cm/s (Marks 2)
A. Quantity I > Quantity II
B. Quantity I < Quantity II
for more join
C. Quantity I ≥ Quantity II https://t.me/currentAffairscLuB
D. Quantity I ≤ Quantity II
E. Quantity I = Quantity II or No relation

Your Answer: Not Attempted

Correct Answer: E. Quantity I = Quantity II or No relation <br> <br>


https://store.adda247.com/#!/myTestAnalysis/printsolution/mappingId=25273/packageId=359/lang=ENGLISH 58/109
12/31/2017 Adda247 Store | Adda247 Store

Solution

Purchased by roshnipanwar025@gmail.com
Q71.
There are two bags, one of which contains 3 black and 4 white balls, while the other contains 4
black and 3 white balls. A dice is cast, if the face 1 or 3 turns up, a ball is taken from the rst bag
and if any other face turns up, a ball is chosen from the second bag.
The probability of choosing a black ball is: (Marks 2)
A. 11/24
B. 11/21
C. 11/27
D. 11/29
E. None of these
Your Answer: Not Attempted

Correct Answer: B. 11/21 <br>

Solution

Purchased by roshnipanwar025@gmail.com
Q72.
There are two bags, one of which contains 3 black and 4 white balls, while the other contains 4
black and 3 white balls. A dice is cast, if the face 1 or 3 turns up, a ball is taken from the rst bag
and if any other face turns up, a ball is chosen from the second bag.
The probability of choosing a white ball is: (Marks 2)
A. 11/21
B. 11/29
C. 10/21
D. 14/23
E. None of these
Your Answer: Not Attempted
for more join
Correct Answer: C. 10/21 <br> https://t.me/currentAffairscLuB
Solution

Purchased by roshnipanwar025@gmail.com
Q73.

https://store.adda247.com/#!/myTestAnalysis/printsolution/mappingId=25273/packageId=359/lang=ENGLISH 59/109
12/31/2017 Adda247 Store | Adda247 Store
Read the following line graphs carefully and answer the following questions :
A Scooterist and a motorist cover a distance of 120 km. They both start their journey at 9 : 00 AM.
In the rst line graph the distance time-slots of 30 minutes is given for the scooterist and the same
is given in second graph for the motorist.
Note: Both scooterist and mototrist run with a constant speed in the given intervals of 30 minutes.

Average speed of the scooterist to cover rst 110 km distance is what percent more/less than the
average speed of the motorist to cover rst 100 km distance ? (Marks 2)
A. 5%
B. 10%
C. 15%
D. 20%
E. None of these
Your Answer: Not Attempted

Correct Answer: B. 10% <br>

Solution

Purchased by roshnipanwar025@gmail.com
Q74.
Read the following line graphs carefully and answer the following questions :
A Scooterist and a motorist cover a distance of 120 km. They both start their journey at 9 : 00 AM.
In the rst line graph the distance time-slots of 30 minutes is given for the scooterist and the same
is given in second graph for the motorist.
Note: Both scooterist and mototrist run with a constant speed in the given intervals of 30 minutes.

At 10 : 30 am, a car starts chasing the scooterist with the speed of 140 km/hr from the point where
scooterist started his journey , then nd the time at which the car will caught the scooterist ?
(Marks 2) for more join
A. 11 : 18 am
B. 11 : 08 am https://t.me/currentAffairscLuB
C. 11 : 15 am
D. 11 : 32 am
E. Can’t be determined
Your Answer: Not Attempted

Correct Answer: A. 11 : 18 am
https://store.adda247.com/#!/myTestAnalysis/printsolution/mappingId=25273/packageId=359/lang=ENGLISH 60/109
12/31/2017 Adda247 Store | Adda247 Store

Solution

Purchased by roshnipanwar025@gmail.com
Q75.
Read the following line graphs carefully and answer the following questions :
A Scooterist and a motorist cover a distance of 120 km. They both start their journey at 9 : 00 AM.
In the rst line graph the distance time-slots of 30 minutes is given for the scooterist and the same
is given in second graph for the motorist.
Note: Both scooterist and mototrist run with a constant speed in the given intervals of 30 minutes.

If the speed of the scooterist and the motorist is increased by 10% and 20% respectively
throughout the journey then nd the di erence in time taken by the scooterist and motorist to
cover the given total distance ? (Marks 2)
A. 11/23 hr
B. 7/22 hr
C. 4/11 hr
D. 25/132 hr
E. None of these
Your Answer: Not Attempted

Correct Answer: D. 25/132 hr <br>

Solution

Time taken by scooterist to cover the total distance with increased speed=
Time taken by motorist to cover the total distance with increased speed=
Purchased by roshnipanwar025@gmail.com
Q76.
Read the following line graphs carefully and answer the following questions :
A Scooterist and a motorist cover a distance of 120 km. They both start their journey at 9 : 00 AM.
for more join
In the rst line graph the distance time-slots of 30 minutes is given for the scooterist and the same
https://t.me/currentAffairscLuB
is given in second graph for the motorist.
Note: Both scooterist and mototrist run with a constant speed in the given intervals of 30 minutes.

At what time they will be 30 km apart from each other if they choose the same road to cover 120
km of distance ? (Marks 2)
A. 10 : 20 am
https://store.adda247.com/#!/myTestAnalysis/printsolution/mappingId=25273/packageId=359/lang=ENGLISH 61/109
12/31/2017 Adda247 Store | Adda247 Store
B. 11 : 15 am
C. 11 : 09 am
D. 10 : 00 am
E. None of these
Your Answer: Not Attempted

Correct Answer: E. None of these <br>

Solution
There will be more than one possibility to be 30 km apart.

Purchased by roshnipanwar025@gmail.com
Q77.
In the gure given below, ACB is a right angled triangle. CD is the altitude. Circles are inscribed
within the triangles ACD, BCD. P and Q are the centres of the circles. The distance PQ is
(Marks 2)
A. 5
B. √50
C. 7
D. 8
E. None of these
Your Answer: Not Attempted

Correct Answer: C. 7 <br>

Solution

Purchased by roshnipanwar025@gmail.com
Q78.
The average earning of a group of persons is Rs. 50 per day. The di erence between the highest
earning and lowest earning of any two persons of the group is Rs. 45. If these two people are
excluded the average earning of the group decreased by Rs. 1. If the minimum earning of the
for more join
person in the group lies between 42 and 47 and the number of persons initially in the group was
equal to a prime number, with both its https://t.me/currentAffairscLuB
digits prime. The number of persons in the group initially
was: (Marks 2)
A. 29
B. 53
C. 31
D. 25
E. None of these
https://store.adda247.com/#!/myTestAnalysis/printsolution/mappingId=25273/packageId=359/lang=ENGLISH 62/109
12/31/2017 Adda247 Store | Adda247 Store
Your Answer: Not Attempted

Correct Answer: E. None of these <br>

Solution

Let there be n people (initially) in the group, then the total earning of the group = n × 50
Again n × 50 = (n – 2) × 49 + (2x + 45)
⇒ n = 2x – 53; where x is the lowest earning of any person.
Now, since 42 < x < 47 an n ∈ prime numbers
Then the only possible value of n = 37 for x = 45.
Purchased by roshnipanwar025@gmail.com
Q79.
There are 500 rooms in a multi- oored hotel. However, due to a change in rule, the hotel has to
decrease the number of oors by 5. However, the management is able to put 5 more rooms in
each oor. Over all, the number of rooms in the hotel decreases by 10%.
What is the number of oors and the number of rooms/ oor the hotel originally had? (Marks 1)
A. 10 oors 50 rooms
B. 20 oors 20 rooms
C. 20 oors 25 rooms
D. 50 oors 10 rooms
E. none of these
Your Answer: Not Attempted

Correct Answer: C. 20 oors 25 rooms <br>

Solution

Purchased by roshnipanwar025@gmail.com
Q80.
There are 500 rooms in a multi- oored hotel. However, due to a change in rule, the hotel has to
decrease the number of oors by 5. However, the management is able to put 5 more rooms in
for more join
each oor. Over all, the number of rooms in the hotel decreases by 10%.
If the hotel has to increase the total no.https://t.me/currentAffairscLuB
of rooms to 600, then what is the additional number of
rooms the hotel must put in each oor ? (Marks 1)
A. 20
B. 15
C. 10
D. 25
E. None of these
https://store.adda247.com/#!/myTestAnalysis/printsolution/mappingId=25273/packageId=359/lang=ENGLISH 63/109
12/31/2017 Adda247 Store | Adda247 Store
Your Answer: Not Attempted

Correct Answer: C. 10 <br>

Solution

Purchased by roshnipanwar025@gmail.com
Q81.
Read the following passage carefully and answer the questions that follow.
Shaw’s defense of a theater of ideas brought him up against both his great bugbears—
commercialized art on the one hand and Art for Art’s Sake on the other. His teaching is that beauty
is a by-product of other activity; that the artist writes out of moral passion (in forms varying from
political conviction to religious zeal), not out of love of art; that the pursuit of art for its own sake is
a form of self-indulgence as bad as any other sort of sensuality. In the end, the errors of “pure” art
and of commercialized art are identical: they both appeal primarily to the senses. True art, on the
other hand, is not merely a matter of pleasure. It may be unpleasant. A favorite Shavian metaphor
for the function of the arts is that of tooth-pulling. Even if the patient is under laughing gas, the
tooth is still pulled.
The history of aesthetics a ords more examples of a didactic than of a hedonist view. But Shaw’s
didacticism takes an unusual turn in its application to the history of arts. If, as Shaw holds, ideas
are a most important part of a work of art, and if, as he also holds, ideas go out of date, it follows
that even the best works of art go out of date in some important respects and that the generally
held view that great works are in all respects eternal is not shared by Shaw. In the preface to Three
Plays for Puritans, he maintains that renewal in the arts means renewal in philosophy, that the
rst great artist who comes along after a renewal gives to the new philosophy full and nal form,
that subsequent artists, though even more gifted, can do nothing but re ne upon the master
without matching him. Shaw, whose essential modesty is as disarming as his pose of vanity is
disconcerting, assigns to himself the role, not of the master, but of the pioneer, the role of a
Marlowe rather than of a Shakespeare. “The whirligig of time will soon bring my audiences to my
own point of view,” he writes, “and then the next Shakespeare that comes along will turn these
petty tentatives of mine into masterpieces nal for their epoch.”
“Final for their epoch”—even Shakespearean masterpieces are not nal beyond that. No one, says
for more join
Shaw, will ever write a better tragedy than Lear or a better opera than Don Giovanni or a better
https://t.me/currentAffairscLuB
music drama than Der Ring des Nibelungen; but just as essential to a play as this aesthetic merit is
moral relevance which, if we take a naturalistic and historical view of morals, it loses, or partly
loses, in time. Shaw, who has the courage of his historicism, consistently withstands the view that
moral problems do not change, and argues therefore that for us modern literature and music
form a Bible surpassing in signi cance the Hebrew Bible. That is Shaw’s anticipatory challenge to
the neo-orthodoxy of today.

https://store.adda247.com/#!/myTestAnalysis/printsolution/mappingId=25273/packageId=359/lang=ENGLISH 64/109
12/31/2017 Adda247 Store | Adda247 Store
It can be inferred from the passage that Shaw would probably agree with all of the following
statements about Shakespeare EXCEPT: (Marks 1.5)
A. He wrote out of a moral passion.
B. All of his plays are out of date in some important respect.
C. He was the most profound and original thinker of his epoch.
D. He was a greater artist than Marlowe.
E. His Lear gives full and nal form to the philosophy of his age.
Your Answer: Not Attempted

Correct Answer: C. He was the most profound and original thinker of his epoch. <br>

Solution
Refer the last sentence of the second paragraph, “…and then the next Shakespeare that comes
along will turn these petty tentatives of mine into masterpieces nal for their epoch…” and the rst
sentence of the third paragraph, “Final for their epoch—even Shakespearean masterpieces are not
nal beyond that.” Thus in Shaw’s point of view, the adjective mentioned in the option (c) would be
applicable for Marlowe. Hence (c) is the correct choice.
Purchased by roshnipanwar025@gmail.com
Q82.
Read the following passage carefully and answer the questions that follow.
Shaw’s defense of a theater of ideas brought him up against both his great bugbears—
commercialized art on the one hand and Art for Art’s Sake on the other. His teaching is that beauty
is a by-product of other activity; that the artist writes out of moral passion (in forms varying from
political conviction to religious zeal), not out of love of art; that the pursuit of art for its own sake is
a form of self-indulgence as bad as any other sort of sensuality. In the end, the errors of “pure” art
and of commercialized art are identical: they both appeal primarily to the senses. True art, on the
other hand, is not merely a matter of pleasure. It may be unpleasant. A favorite Shavian metaphor
for the function of the arts is that of tooth-pulling. Even if the patient is under laughing gas, the
tooth is still pulled.
The history of aesthetics a ords more examples of a didactic than of a hedonist view. But Shaw’s
didacticism takes an unusual turn in its application to the history of arts. If, as Shaw holds, ideas
are a most important part of a work of art, and if, as he also holds, ideas go out of date, it follows
for more join
that even the best works of art go out of date in some important respects and that the generally
https://t.me/currentAffairscLuB
held view that great works are in all respects eternal is not shared by Shaw. In the preface to Three
Plays for Puritans, he maintains that renewal in the arts means renewal in philosophy, that the
rst great artist who comes along after a renewal gives to the new philosophy full and nal form,
that subsequent artists, though even more gifted, can do nothing but re ne upon the master
without matching him. Shaw, whose essential modesty is as disarming as his pose of vanity is
disconcerting, assigns to himself the role, not of the master, but of the pioneer, the role of a
Marlowe rather than of a Shakespeare. “The whirligig of time will soon bring my audiences to my
https://store.adda247.com/#!/myTestAnalysis/printsolution/mappingId=25273/packageId=359/lang=ENGLISH 65/109
12/31/2017 Adda247 Store | Adda247 Store
own point of view,” he writes, “and then the next Shakespeare that comes along will turn these
petty tentatives of mine into masterpieces nal for their epoch.”
“Final for their epoch”—even Shakespearean masterpieces are not nal beyond that. No one, says
Shaw, will ever write a better tragedy than Lear or a better opera than Don Giovanni or a better
music drama than Der Ring des Nibelungen; but just as essential to a play as this aesthetic merit is
moral relevance which, if we take a naturalistic and historical view of morals, it loses, or partly
loses, in time. Shaw, who has the courage of his historicism, consistently withstands the view that
moral problems do not change, and argues therefore that for us modern literature and music
form a Bible surpassing in signi cance the Hebrew Bible. That is Shaw’s anticipatory challenge to
the neo-orthodoxy of today.
Which of the following does the author cite as a contradiction in Shaw? (Marks 1.5)
A. Whereas he pretended to be vain, he was actually modest.
B. He questioned the signi cance of the Hebrew Bible, and yet he believed that a great artist could
be motivated by religious zeal.
C. Although he insisted that true art springs from moral passion, he rejected the notion that
morals do not change.
D. He considered himself to be the pioneer of a new philosophy, but he hoped his audiences
would eventually adopt his point of view.
E. On the one hand, he held that ideas are a most important part of a work of art; on the other
hand, he believed that ideas go out of date.
Your Answer: Not Attempted

Correct Answer: A. Whereas he pretended to be vain, he was actually modest. <br>

Solution
Refer the second paragraph of the passage, “Shaw, whose essential modesty is as disarming as his
pose of vanity is disconcerting, assigns to himself the role, not of the master, but of the pioneer,
the role of a Marlowe rather than of a Shakespeare.” Thus option (a) is the correct choice.
Purchased by roshnipanwar025@gmail.com
Q83.
Read the following passage carefully and answer the questions that follow.
Shaw’s defense of a theater of ideas brought him up against both his great bugbears—
for more join
commercialized art on the one hand and Art for Art’s Sake on the other. His teaching is that beauty
https://t.me/currentAffairscLuB
is a by-product of other activity; that the artist writes out of moral passion (in forms varying from
political conviction to religious zeal), not out of love of art; that the pursuit of art for its own sake is
a form of self-indulgence as bad as any other sort of sensuality. In the end, the errors of “pure” art
and of commercialized art are identical: they both appeal primarily to the senses. True art, on the
other hand, is not merely a matter of pleasure. It may be unpleasant. A favorite Shavian metaphor
for the function of the arts is that of tooth-pulling. Even if the patient is under laughing gas, the
tooth is still pulled.
https://store.adda247.com/#!/myTestAnalysis/printsolution/mappingId=25273/packageId=359/lang=ENGLISH 66/109
12/31/2017 Adda247 Store | Adda247 Store
The history of aesthetics a ords more examples of a didactic than of a hedonist view. But Shaw’s
didacticism takes an unusual turn in its application to the history of arts. If, as Shaw holds, ideas
are a most important part of a work of art, and if, as he also holds, ideas go out of date, it follows
that even the best works of art go out of date in some important respects and that the generally
held view that great works are in all respects eternal is not shared by Shaw. In the preface to Three
Plays for Puritans, he maintains that renewal in the arts means renewal in philosophy, that the
rst great artist who comes along after a renewal gives to the new philosophy full and nal form,
that subsequent artists, though even more gifted, can do nothing but re ne upon the master
without matching him. Shaw, whose essential modesty is as disarming as his pose of vanity is
disconcerting, assigns to himself the role, not of the master, but of the pioneer, the role of a
Marlowe rather than of a Shakespeare. “The whirligig of time will soon bring my audiences to my
own point of view,” he writes, “and then the next Shakespeare that comes along will turn these
petty tentatives of mine into masterpieces nal for their epoch.”
“Final for their epoch”—even Shakespearean masterpieces are not nal beyond that. No one, says
Shaw, will ever write a better tragedy than Lear or a better opera than Don Giovanni or a better
music drama than Der Ring des Nibelungen; but just as essential to a play as this aesthetic merit is
moral relevance which, if we take a naturalistic and historical view of morals, it loses, or partly
loses, in time. Shaw, who has the courage of his historicism, consistently withstands the view that
moral problems do not change, and argues therefore that for us modern literature and music
form a Bible surpassing in signi cance the Hebrew Bible. That is Shaw’s anticipatory challenge to
the neo-orthodoxy of today.
The ideas attributed to Shaw in the passage suggest that he would most likely agree with which of
the following statements? (Marks 1.5)
A. Every great poet digs down to a level where human nature is always and everywhere alike.
B. A play cannot be comprehended fully without some knowledge and imaginative understanding
of its context.
C. A great music drama like Der Ring des Nibelungen springs from a love of beauty, not from a
love of art.
D. Morality is immutable; it is not something to be discussed and worked out.
E. Don Giovanni is a masterpiece because it is as relevant today as it was when it was created.
Your Answer: Not Attempted
for more join
Correct Answer: C. A great music drama like Der Ring des Nibelungen springs from a love of
https://t.me/currentAffairscLuB
beauty, not from a love of art. <br>

Solution
Refer the latter half of the rst paragraph, “True art, on the other hand, is not merely a matter of
pleasure. It may be unpleasant. A favorite Shavian metaphor for the function of the arts is…” It can
be inferred from the paragraph that the option (c) is the correct choice for the answer.
Purchased by roshnipanwar025@gmail.com
https://store.adda247.com/#!/myTestAnalysis/printsolution/mappingId=25273/packageId=359/lang=ENGLISH 67/109
12/31/2017 Adda247 Store | Adda247 Store
Q84.
Read the following passage carefully and answer the questions that follow.
Shaw’s defense of a theater of ideas brought him up against both his great bugbears—
commercialized art on the one hand and Art for Art’s Sake on the other. His teaching is that beauty
is a by-product of other activity; that the artist writes out of moral passion (in forms varying from
political conviction to religious zeal), not out of love of art; that the pursuit of art for its own sake is
a form of self-indulgence as bad as any other sort of sensuality. In the end, the errors of “pure” art
and of commercialized art are identical: they both appeal primarily to the senses. True art, on the
other hand, is not merely a matter of pleasure. It may be unpleasant. A favorite Shavian metaphor
for the function of the arts is that of tooth-pulling. Even if the patient is under laughing gas, the
tooth is still pulled.
The history of aesthetics a ords more examples of a didactic than of a hedonist view. But Shaw’s
didacticism takes an unusual turn in its application to the history of arts. If, as Shaw holds, ideas
are a most important part of a work of art, and if, as he also holds, ideas go out of date, it follows
that even the best works of art go out of date in some important respects and that the generally
held view that great works are in all respects eternal is not shared by Shaw. In the preface to Three
Plays for Puritans, he maintains that renewal in the arts means renewal in philosophy, that the
rst great artist who comes along after a renewal gives to the new philosophy full and nal form,
that subsequent artists, though even more gifted, can do nothing but re ne upon the master
without matching him. Shaw, whose essential modesty is as disarming as his pose of vanity is
disconcerting, assigns to himself the role, not of the master, but of the pioneer, the role of a
Marlowe rather than of a Shakespeare. “The whirligig of time will soon bring my audiences to my
own point of view,” he writes, “and then the next Shakespeare that comes along will turn these
petty tentatives of mine into masterpieces nal for their epoch.”
“Final for their epoch”—even Shakespearean masterpieces are not nal beyond that. No one, says
Shaw, will ever write a better tragedy than Lear or a better opera than Don Giovanni or a better
music drama than Der Ring des Nibelungen; but just as essential to a play as this aesthetic merit is
moral relevance which, if we take a naturalistic and historical view of morals, it loses, or partly
loses, in time. Shaw, who has the courage of his historicism, consistently withstands the view that
moral problems do not change, and argues therefore that for us modern literature and music
form a Bible surpassing in signi cance the Hebrew Bible. That is Shaw’s anticipatory challenge to
the neo-orthodoxy of today. for more join
The passage contains information that answers which of the following questions?
https://t.me/currentAffairscLuB
I. According to Shaw, what is the most important part of a work of art?
II. In Shaw’s view, what does the Hebrew Bible have in common with Don Giovanni?
III. According to the author, what was Shaw’s assessment of himself as a playwright? (Marks 1.5)
A. I only
B. III only
C. I and II only

https://store.adda247.com/#!/myTestAnalysis/printsolution/mappingId=25273/packageId=359/lang=ENGLISH 68/109
12/31/2017 Adda247 Store | Adda247 Store
D. II and III only
E. I, II, and III
Your Answer: Not Attempted

Correct Answer: D. II and III only <br>

Solution
Option (II) is valid because both the works despite being masterpieces were surpassed later on
and the option (III) is valid because he regards himself as a pioneer, not a master. In the passage,
Shaw’s opinion regarding art’s transition and relevance is discussed and not the constituents of a
creation. Hence option (d) is the correct choice.
Purchased by roshnipanwar025@gmail.com
Q85.
Read the following passage carefully and answer the questions that follow.
Shaw’s defense of a theater of ideas brought him up against both his great bugbears—
commercialized art on the one hand and Art for Art’s Sake on the other. His teaching is that beauty
is a by-product of other activity; that the artist writes out of moral passion (in forms varying from
political conviction to religious zeal), not out of love of art; that the pursuit of art for its own sake is
a form of self-indulgence as bad as any other sort of sensuality. In the end, the errors of “pure” art
and of commercialized art are identical: they both appeal primarily to the senses. True art, on the
other hand, is not merely a matter of pleasure. It may be unpleasant. A favorite Shavian metaphor
for the function of the arts is that of tooth-pulling. Even if the patient is under laughing gas, the
tooth is still pulled.
The history of aesthetics a ords more examples of a didactic than of a hedonist view. But Shaw’s
didacticism takes an unusual turn in its application to the history of arts. If, as Shaw holds, ideas
are a most important part of a work of art, and if, as he also holds, ideas go out of date, it follows
that even the best works of art go out of date in some important respects and that the generally
held view that great works are in all respects eternal is not shared by Shaw. In the preface to Three
Plays for Puritans, he maintains that renewal in the arts means renewal in philosophy, that the
rst great artist who comes along after a renewal gives to the new philosophy full and nal form,
that subsequent artists, though even more gifted, can do nothing but re ne upon the master
without matching him. Shaw, whose essential modesty is as disarming as his pose of vanity is
for more join
disconcerting, assigns to himself the role, not of the master, but of the pioneer, the role of a
Marlowe rather than of a Shakespeare.https://t.me/currentAffairscLuB
“The whirligig of time will soon bring my audiences to my
own point of view,” he writes, “and then the next Shakespeare that comes along will turn these
petty tentatives of mine into masterpieces nal for their epoch.”
“Final for their epoch”—even Shakespearean masterpieces are not nal beyond that. No one, says
Shaw, will ever write a better tragedy than Lear or a better opera than Don Giovanni or a better
music drama than Der Ring des Nibelungen; but just as essential to a play as this aesthetic merit is
moral relevance which, if we take a naturalistic and historical view of morals, it loses, or partly
https://store.adda247.com/#!/myTestAnalysis/printsolution/mappingId=25273/packageId=359/lang=ENGLISH 69/109
12/31/2017 Adda247 Store | Adda247 Store
loses, in time. Shaw, who has the courage of his historicism, consistently withstands the view that
moral problems do not change, and argues therefore that for us modern literature and music
form a Bible surpassing in signi cance the Hebrew Bible. That is Shaw’s anticipatory challenge to
the neo-orthodoxy of today.
According to the author, Shaw’s didacticism was unusual in that it was characterized by (Marks 1.5)
A. idealism
B. historicism
C. hedonism
D. morality
E. religious zeal
Your Answer: Not Attempted

Correct Answer: B. historicism <br>

Solution
According to the author, Shaw’s didacticism was unusual in that it was characterized by
historicism. Hence (b) is the correct choice.
Purchased by roshnipanwar025@gmail.com
Q86.
Read the following passage carefully and answer the questions that follow. Certain words are
given in bold to help you locate them while answering some of the questions.
The distinguishing feature of life is that here the relational modes are of a dynamic nature, such
that reality or existence of any particular mode is dependent on other modes of a di erent order
and vice versa. We have here a situation in which a particular relation - mode or function does not
exist per se but through others, and there is thus a mutual dependence of such a nature that it is
impossible to start with any one of them as being prior to the other. We have here a circle of
revolutions in which any point can be regarded either as the rst or as the last. Yet the rst is in
the last and the last is in the rst. If this relationship is such in life, it is still more so with regard to
the owing activity of the mind-complex, which absolutely unrepresentable by any terms of
physical notation, which behaves as an integrated growing whole and yet keeps its co-variant
relations with life-processes, the body and the environment. Another point is that even the cellular
membrane has a special selective action which attains its highest evolution and development in
for more join
instinct and human intelligence. The selective action in the case of all animals beginning with the
https://t.me/currentAffairscLuB
uni-cellular is to be found in the peculiar phenomenon called behaviour, which is the registration
in an unknown manner of past experiences. This peculiar phenomenon of behaviour serves to
destroy the barrier of time and makes the past, present and the future coalesce at any given
instant, and thus starts the history of the individual as personality. In the lower grades of life
where the behaviour of the individual animal is largely under the control of the body-complex, the
term personality may not suitably be used. But as the mind emerges out of the body and begins to
assert itself in its spontaneous existence though carrying with it the peculiar body-emergents as
https://store.adda247.com/#!/myTestAnalysis/printsolution/mappingId=25273/packageId=359/lang=ENGLISH 70/109
12/31/2017 Adda247 Store | Adda247 Store
appetitive functions, begins to show itself as a true individual, the integrated history of which,
having risen above the appetitive functions, begins to reveal itself in accordance with a selective
purpose, which is its own emergent as value. The appetitive functions here do not lose their
existence but have a transmuted modi cation in consonance with the value-sense. Here the
biological tendencies are not destroyed but their potency, and indeed the potency of the whale
life-history, converges towards the achievement of the self-emergent purpose, the value. There -is
thus here a new ordering of the old existent states of previous history producing by their
harmony, contentment, and blissfulness associated with the progressive march of the higher man.
In the lower order the con icts between the animal and the environment are annulled by the life-
process itself in a very naturalistic manner. With the evolution of mind, mental con icts of
di erent orders arise through our intercourse with other minds. Such con icts are natural and
obvious; and the life-process instead of annulling them often increases them. But as a new
selective purpose as value emerges in man, he sets his house in order. The integrated history
behaves as a person and the con icts are annulled and the whole history becomes a history of
self-realisation in the light of the value. Where the emergent value cannot exert itself as the real
and constant selective purpose of the man but is in con ict with the biological selective purpose
and only inconstantly shows its supremacy from time to time, we have the picture of the ordinary
struggling man.
The factor that most distinguishingly discriminates a lower-being with a higher-being is that (Marks
1.5)
A. the former misses the special selective action
B. the physical purpose dominates the value-purpose
C. the former lacks harmony, contentment and blissfulness
D. the former lacks the dynamic nature of relation
E. All of the above
Your Answer: Not Attempted

Correct Answer: B. the physical purpose dominates the value-purpose <br>

Solution
Refer the last sentence of the passage, “Where the emergent value cannot exert itself as the real
for more join
and constant selective purpose of the man but is in con ict with the biological selective purpose
and only inconstantly shows its supremacy from time to time, we have the picture of the ordinary
https://t.me/currentAffairscLuB
struggling man.” Hence (b) is the correct choice in the context of the passage.
Purchased by roshnipanwar025@gmail.com
Q87.
Read the following passage carefully and answer the questions that follow. Certain words are
given in bold to help you locate them while answering some of the questions.
The distinguishing feature of life is that here the relational modes are of a dynamic nature, such
that reality or existence of any particular mode is dependent on other modes of a di erent order
https://store.adda247.com/#!/myTestAnalysis/printsolution/mappingId=25273/packageId=359/lang=ENGLISH 71/109
12/31/2017 Adda247 Store | Adda247 Store
and vice versa. We have here a situation in which a particular relation - mode or function does not
exist per se but through others, and there is thus a mutual dependence of such a nature that it is
impossible to start with any one of them as being prior to the other. We have here a circle of
revolutions in which any point can be regarded either as the rst or as the last. Yet the rst is in
the last and the last is in the rst. If this relationship is such in life, it is still more so with regard to
the owing activity of the mind-complex, which absolutely unrepresentable by any terms of
physical notation, which behaves as an integrated growing whole and yet keeps its co-variant
relations with life-processes, the body and the environment. Another point is that even the cellular
membrane has a special selective action which attains its highest evolution and development in
instinct and human intelligence. The selective action in the case of all animals beginning with the
uni-cellular is to be found in the peculiar phenomenon called behaviour, which is the registration
in an unknown manner of past experiences. This peculiar phenomenon of behaviour serves to
destroy the barrier of time and makes the past, present and the future coalesce at any given
instant, and thus starts the history of the individual as personality. In the lower grades of life
where the behaviour of the individual animal is largely under the control of the body-complex, the
term personality may not suitably be used. But as the mind emerges out of the body and begins to
assert itself in its spontaneous existence though carrying with it the peculiar body-emergents as
appetitive functions, begins to show itself as a true individual, the integrated history of which,
having risen above the appetitive functions, begins to reveal itself in accordance with a selective
purpose, which is its own emergent as value. The appetitive functions here do not lose their
existence but have a transmuted modi cation in consonance with the value-sense. Here the
biological tendencies are not destroyed but their potency, and indeed the potency of the whale
life-history, converges towards the achievement of the self-emergent purpose, the value. There -is
thus here a new ordering of the old existent states of previous history producing by their
harmony, contentment, and blissfulness associated with the progressive march of the higher man.
In the lower order the con icts between the animal and the environment are annulled by the life-
process itself in a very naturalistic manner. With the evolution of mind, mental con icts of
di erent orders arise through our intercourse with other minds. Such con icts are natural and
obvious; and the life-process instead of annulling them often increases them. But as a new
selective purpose as value emerges in man, he sets his house in order. The integrated history
behaves as a person and the con icts are annulled and the whole history becomes a history of
for more join
self-realisation in the light of the value. Where the emergent value cannot exert itself as the real
and constant selective purpose of the man but is in con ict with the biological selective purpose
https://t.me/currentAffairscLuB
and only inconstantly shows its supremacy from time to time, we have the picture of the ordinary
struggling man.
Which one of the aspects of behaviour is not supported by the passage? (Marks 1.5)
A. Every organism is blessed with it.
B. It passes through the phases of evolution.
C. It transcends temporal barrier.

https://store.adda247.com/#!/myTestAnalysis/printsolution/mappingId=25273/packageId=359/lang=ENGLISH 72/109
12/31/2017 Adda247 Store | Adda247 Store
D. Gradually the physical aspect of behaviour pales into insigni cance.
E. None of the above
Your Answer: Not Attempted

Correct Answer: E. None of the above <br>

Solution
Refer the statement, “Here the biological tendencies are not destroyed but their potency, and
indeed the potency of the whale life-history, converges towards the achievement of the self-
emergent purpose, the value.” Thus it does not become insigni cant, rather it is directed towards
ful llment of values. Hence (e) is the correct choice.
Purchased by roshnipanwar025@gmail.com
Q88.
Read the following passage carefully and answer the questions that follow. Certain words are
given in bold to help you locate them while answering some of the questions.
The distinguishing feature of life is that here the relational modes are of a dynamic nature, such
that reality or existence of any particular mode is dependent on other modes of a di erent order
and vice versa. We have here a situation in which a particular relation - mode or function does not
exist per se but through others, and there is thus a mutual dependence of such a nature that it is
impossible to start with any one of them as being prior to the other. We have here a circle of
revolutions in which any point can be regarded either as the rst or as the last. Yet the rst is in
the last and the last is in the rst. If this relationship is such in life, it is still more so with regard to
the owing activity of the mind-complex, which absolutely unrepresentable by any terms of
physical notation, which behaves as an integrated growing whole and yet keeps its co-variant
relations with life-processes, the body and the environment. Another point is that even the cellular
membrane has a special selective action which attains its highest evolution and development in
instinct and human intelligence. The selective action in the case of all animals beginning with the
uni-cellular is to be found in the peculiar phenomenon called behaviour, which is the registration
in an unknown manner of past experiences. This peculiar phenomenon of behaviour serves to
destroy the barrier of time and makes the past, present and the future coalesce at any given
instant, and thus starts the history of the individual as personality. In the lower grades of life
where the behaviour of the individual animal is largely under the control of the body-complex, the
for more join
term personality may not suitably be used. But as the mind emerges out of the body and begins to
https://t.me/currentAffairscLuB
assert itself in its spontaneous existence though carrying with it the peculiar body-emergents as
appetitive functions, begins to show itself as a true individual, the integrated history of which,
having risen above the appetitive functions, begins to reveal itself in accordance with a selective
purpose, which is its own emergent as value. The appetitive functions here do not lose their
existence but have a transmuted modi cation in consonance with the value-sense. Here the
biological tendencies are not destroyed but their potency, and indeed the potency of the whale
life-history, converges towards the achievement of the self-emergent purpose, the value. There -is
https://store.adda247.com/#!/myTestAnalysis/printsolution/mappingId=25273/packageId=359/lang=ENGLISH 73/109
12/31/2017 Adda247 Store | Adda247 Store
thus here a new ordering of the old existent states of previous history producing by their
harmony, contentment, and blissfulness associated with the progressive march of the higher man.
In the lower order the con icts between the animal and the environment are annulled by the life-
process itself in a very naturalistic manner. With the evolution of mind, mental con icts of
di erent orders arise through our intercourse with other minds. Such con icts are natural and
obvious; and the life-process instead of annulling them often increases them. But as a new
selective purpose as value emerges in man, he sets his house in order. The integrated history
behaves as a person and the con icts are annulled and the whole history becomes a history of
self-realisation in the light of the value. Where the emergent value cannot exert itself as the real
and constant selective purpose of the man but is in con ict with the biological selective purpose
and only inconstantly shows its supremacy from time to time, we have the picture of the ordinary
struggling man.
The phrase “ registration in an unknown number of past experiences” signi es that (Marks 1.5)
A. behavioral tenets from the past superimpose one over another.
B. personality is an exhibition of all the three aspects of time.
C. experiences when accumulated transcend the barrier of time.
D. one is not discriminating while learning lessons from the past.
E. None of the above

Your Answer: Not Attempted

Correct Answer: D. one is not discriminating while learning lessons from the past. <br>

Solution
The phrase “ resignation in an unknown number of past experiences ” signi es that one is not
discriminating while learning lessons from the past. Behaviour registers innumerable experiences
of the past. Hence (d) is the correct choice in the context of the passage.
Purchased by roshnipanwar025@gmail.com
Q89.
Read the following passage carefully and answer the questions that follow. Certain words are
given in bold to help you locate them while answering some of the questions.
The distinguishing feature of life is that here the relational modes are of a dynamic nature, such
for more join
that reality or existence of any particular mode is dependent on other modes of a di erent order
and vice versa. We have here a situation https://t.me/currentAffairscLuB
in which a particular relation - mode or function does not
exist per se but through others, and there is thus a mutual dependence of such a nature that it is
impossible to start with any one of them as being prior to the other. We have here a circle of
revolutions in which any point can be regarded either as the rst or as the last. Yet the rst is in
the last and the last is in the rst. If this relationship is such in life, it is still more so with regard to
the owing activity of the mind-complex, which absolutely unrepresentable by any terms of
physical notation, which behaves as an integrated growing whole and yet keeps its co-variant
https://store.adda247.com/#!/myTestAnalysis/printsolution/mappingId=25273/packageId=359/lang=ENGLISH 74/109
12/31/2017 Adda247 Store | Adda247 Store
relations with life-processes, the body and the environment. Another point is that even the cellular
membrane has a special selective action which attains its highest evolution and development in
instinct and human intelligence. The selective action in the case of all animals beginning with the
uni-cellular is to be found in the peculiar phenomenon called behaviour, which is the registration
in an unknown manner of past experiences. This peculiar phenomenon of behaviour serves to
destroy the barrier of time and makes the past, present and the future coalesce at any given
instant, and thus starts the history of the individual as personality. In the lower grades of life
where the behaviour of the individual animal is largely under the control of the body-complex, the
term personality may not suitably be used. But as the mind emerges out of the body and begins to
assert itself in its spontaneous existence though carrying with it the peculiar body-emergents as
appetitive functions, begins to show itself as a true individual, the integrated history of which,
having risen above the appetitive functions, begins to reveal itself in accordance with a selective
purpose, which is its own emergent as value. The appetitive functions here do not lose their
existence but have a transmuted modi cation in consonance with the value-sense. Here the
biological tendencies are not destroyed but their potency, and indeed the potency of the whale
life-history, converges towards the achievement of the self-emergent purpose, the value. There -is
thus here a new ordering of the old existent states of previous history producing by their
harmony, contentment, and blissfulness associated with the progressive march of the higher man.
In the lower order the con icts between the animal and the environment are annulled by the life-
process itself in a very naturalistic manner. With the evolution of mind, mental con icts of
di erent orders arise through our intercourse with other minds. Such con icts are natural and
obvious; and the life-process instead of annulling them often increases them. But as a new
selective purpose as value emerges in man, he sets his house in order. The integrated history
behaves as a person and the con icts are annulled and the whole history becomes a history of
self-realisation in the light of the value. Where the emergent value cannot exert itself as the real
and constant selective purpose of the man but is in con ict with the biological selective purpose
and only inconstantly shows its supremacy from time to time, we have the picture of the ordinary
struggling man.
Which one of the following represents the main theme of the passage most suitably? (Marks 1.5)
A. An ordinary man struggles because he lives physically.
B. With the evolution of cellular organism there is an evolution of behaviour.
for more join
C. There occurs a transmutation of appetitive functions in Homo-sapiens.
D. Behaviour is an evolution from physical to mental.
https://t.me/currentAffairscLuB
E. Value orientation gives precedence to mind over body.
Your Answer: Not Attempted

Correct Answer: E. Value orientation gives precedence to mind over body. <br>

https://store.adda247.com/#!/myTestAnalysis/printsolution/mappingId=25273/packageId=359/lang=ENGLISH 75/109
12/31/2017 Adda247 Store | Adda247 Store
Solution
Read the passage carefully, it can be inferred that the most suitable theme of the passage is “
Value orientation gives precedence to mind over body .” Hence option (e) is the correct choice.
Purchased by roshnipanwar025@gmail.com
Q90.
Read the following passage carefully and answer the questions that follow. Certain words are
given in bold to help you locate them while answering some of the questions.
The distinguishing feature of life is that here the relational modes are of a dynamic nature, such
that reality or existence of any particular mode is dependent on other modes of a di erent order
and vice versa. We have here a situation in which a particular relation - mode or function does not
exist per se but through others, and there is thus a mutual dependence of such a nature that it is
impossible to start with any one of them as being prior to the other. We have here a circle of
revolutions in which any point can be regarded either as the rst or as the last. Yet the rst is in
the last and the last is in the rst. If this relationship is such in life, it is still more so with regard to
the owing activity of the mind-complex, which absolutely unrepresentable by any terms of
physical notation, which behaves as an integrated growing whole and yet keeps its co-variant
relations with life-processes, the body and the environment. Another point is that even the cellular
membrane has a special selective action which attains its highest evolution and development in
instinct and human intelligence. The selective action in the case of all animals beginning with the
uni-cellular is to be found in the peculiar phenomenon called behaviour, which is the registration
in an unknown manner of past experiences. This peculiar phenomenon of behaviour serves to
destroy the barrier of time and makes the past, present and the future coalesce at any given
instant, and thus starts the history of the individual as personality. In the lower grades of life
where the behaviour of the individual animal is largely under the control of the body-complex, the
term personality may not suitably be used. But as the mind emerges out of the body and begins to
assert itself in its spontaneous existence though carrying with it the peculiar body-emergents as
appetitive functions, begins to show itself as a true individual, the integrated history of which,
having risen above the appetitive functions, begins to reveal itself in accordance with a selective
purpose, which is its own emergent as value. The appetitive functions here do not lose their
existence but have a transmuted modi cation in consonance with the value-sense. Here the
biological tendencies are not destroyed but their potency, and indeed the potency of the whale
life-history, converges towards the achievement of the self-emergent purpose, the value. There -is
for more join
thus here a new ordering of the old existent states of previous history producing by their
harmony, contentment, and blissfulness https://t.me/currentAffairscLuB
associated with the progressive march of the higher man.
In the lower order the con icts between the animal and the environment are annulled by the life-
process itself in a very naturalistic manner. With the evolution of mind, mental con icts of
di erent orders arise through our intercourse with other minds. Such con icts are natural and
obvious; and the life-process instead of annulling them often increases them. But as a new
selective purpose as value emerges in man, he sets his house in order. The integrated history
behaves as a person and the con icts are annulled and the whole history becomes a history of
https://store.adda247.com/#!/myTestAnalysis/printsolution/mappingId=25273/packageId=359/lang=ENGLISH 76/109
12/31/2017 Adda247 Store | Adda247 Store
self-realisation in the light of the value. Where the emergent value cannot exert itself as the real
and constant selective purpose of the man but is in con ict with the biological selective purpose
and only inconstantly shows its supremacy from time to time, we have the picture of the ordinary
struggling man.
Choose the word among the given options which is most similar in meaning to the word “
COALESCE” as used in the passage. (Marks 1.5)
A. Unfasten
B. Sever
C. A liate
D. Sunder
E. Lacerate
Your Answer: Not Attempted

Correct Answer: C. A liate <br>

Solution
The word “ Coalesce ” means come together to form one mass or whole. The word “ A liate ”
means o cially join or become attached to an organization. Hence both are similar in meanings.
Sever means put an end to (a connection or relationship); break o .
Sunder means split apart.
Lacerate means criticize forcefully or severely.
Purchased by roshnipanwar025@gmail.com
Q91.
In each of the following questions ve options are given, of which one word is most nearly the
same or opposite in meaning to the given word in the question. Find the correct option having
either same or opposite meaning.
EXODUS (Marks 1)
A. Eschew
B. Disparage
C. Camaraderie
D. Egression
E. Rustic for more join
Your Answer: Not Attempted
https://t.me/currentAffairscLuB
Correct Answer: D. Egression <br>

Solution
Exodus means a mass departure of people while Egress comes from a Latin word meaning "going
out." An egress is basically the same thing as an exit. Hence and exodus and egression has almost
a similar meaning and therefore option (d) is the correct choice.
https://store.adda247.com/#!/myTestAnalysis/printsolution/mappingId=25273/packageId=359/lang=ENGLISH 77/109
12/31/2017 Adda247 Store | Adda247 Store
Purchased by roshnipanwar025@gmail.com
Q92.
In each of the following questions ve options are given, of which one word is most nearly the
same or opposite in meaning to the given word in the question. Find the correct option having
either same or opposite meaning.
QUINTESSENTIAL (Marks 1)
A. Feckless
B. Nonchalant
C. archetypal
D. Coetaneous
E. Banal
Your Answer: Not Attempted

Correct Answer: C. archetypal <br>

Solution
quintessential means representing the most perfect or typical example of a quality or class.
Archetypal means very typical of a certain kind of person or thing. Hence quintessential and
archetypal have similar meaning and therefore option (c) is the correct choice.

Purchased by roshnipanwar025@gmail.com
Q93.
In each of the following questions ve options are given, of which one word is most nearly the
same or opposite in meaning to the given word in the question. Find the correct option having
either same or opposite meaning.
STYMIE (Marks 1)
A. Enmesh
B. Prepossession
C. Oblige
D. Prevaricate
E. Embellish
Your Answer: Not Attempted
for more join
Correct Answer: C. Oblige <br> https://t.me/currentAffairscLuB
Solution
Stymie means prevent or hinder the progress of. Oblige means do as (someone) asks or desires in
order to help or please them. Hence stymie and oblige are opposite in meaning and therefore
option (c) is the correct choice.
Purchased by roshnipanwar025@gmail.com

https://store.adda247.com/#!/myTestAnalysis/printsolution/mappingId=25273/packageId=359/lang=ENGLISH 78/109
12/31/2017 Adda247 Store | Adda247 Store
Q94.
In each of the following questions ve options are given, of which one word is most nearly the
same or opposite in meaning to the given word in the question. Find the correct option having
either same or opposite meaning.
CLAMOUR (Marks 1)
A. Malaise
B. Prima donna
C. Autarky
D. Reticence
E. Incursion
Your Answer: Not Attempted

Correct Answer: D. Reticence <br>

Solution
Clamour means a loud and confused noise, especially that of people shouting.
Reticence means not revealing one's thoughts or feelings readily; reserve.
Hence clamour and reticence are opposite in meaning.
Purchased by roshnipanwar025@gmail.com
Q95.
In each of the following questions ve options are given, of which one word is most nearly the
same or opposite in meaning to the given word in the question. Find the correct option having
either same or opposite meaning.
DEPRAVED (Marks 1)
A. Nefarious
B. Despotic
C. Inveterate
D. Ominous
E. Ingenuity

Your Answer: Not Attempted


for more join
Correct Answer: A. Nefarious <br>
https://t.me/currentAffairscLuB
Solution
Depraved means morally corrupt; wicked. Nefarious means (typically of an action or activity)
wicked or criminal. Hence depraved and nefarious are similar in meaning.
Purchased by roshnipanwar025@gmail.com
Q96.

https://store.adda247.com/#!/myTestAnalysis/printsolution/mappingId=25273/packageId=359/lang=ENGLISH 79/109
12/31/2017 Adda247 Store | Adda247 Store
Each of the following questions has a paragraph from which one sentence has been deleted. From
the given options, choose the one that completes the paragraph in the most appropriate way.
As the former Deputy Governor of the RBI, K.C. Chakraborty, had pointed out then, the RBI’s
failure to ensure transparency in its currency distribution operations signi cantly aggravated the
situation. _________________________________________. Indeed, as representatives of bank unions have
claimed, the information asymmetry that followed meant that some banks, mostly private ones,
were favoured over public sector banks in the matter of supply of adequate currency. The severe
shortage of fresh currency was aggravated by the fact that the RBI failed to ensure a more
equitable and fair distribution of notes. (Marks 1)
A. It quickly became evident that unprecedented bungling at the very highest levels was
responsible for the crisis
B. Bank customers had no way of knowing where they could access their money.
C. At the height of the crisis, it was reduced to running a lottery for those using digital payment
systems.
D. Meanwhile, a year after the gigantic asco, the RBI claims it is still “scrutinising” the old notes.
E. If the intention was to replace cash by digital modes of payment, the latter would have to assure
the same level of certainty, predictability and acceptance.
Your Answer: Not Attempted

Correct Answer: B. Bank customers had no way of knowing where they could access their money.
<br>

Solution
The given paragraph is about the aftermath of the demonitisation. Among the given options, there
is only sentence (b) that ts perfectly into the blank space. The sentence adds meaning to the
paragraph and at the same time it correctly follows the rst sentence of the paragraph. Other
options are irrelevant in the context of the paragraph. Hence (b) is the correct choice.

Purchased by roshnipanwar025@gmail.com
Q97.
Each of the following questions has a paragraph from which one sentence has been deleted. From
the given options, choose the one that completes the paragraph in the most appropriate way.
for more join
The decision of the Yogi Adityanath government to introduce NCERT books in madrasas without
disrupting the religious curriculum is inhttps://t.me/currentAffairscLuB
keeping with the spirit of Prime Minister Modi’s mission to
educate and empower the Muslim youth by modernising madrasas. The Nai Manzil scheme,
launched soon after PM Modi came to power, has been promoting modern education and
developing scienti c temperament in madrasas with the introduction of science, computers,
mathematics and English. __________________________________________________ (Marks 1)
A. The e ort also includes developing multiple skills which will come in handy for Muslim students
when they get into the job market.
https://store.adda247.com/#!/myTestAnalysis/printsolution/mappingId=25273/packageId=359/lang=ENGLISH 80/109
12/31/2017 Adda247 Store | Adda247 Store
B. The idea is to encourage transparency and isolate the y-by-night operators.
C. We should not give the impression to the world that we have not moved on from the time we
collectively opposed the introduction of English and abstained from learning it for decades,
arguing it was the language of the in dels.
D. The “saviours of Muslims” in the community need to stand up and be counted.
E. The madrasas need to catch up with the national curriculum.
Your Answer: Not Attempted

Correct Answer: A. The e ort also includes developing multiple skills which will come in handy for
Muslim students when they get into the job market. <br>

Solution
The given paragraph is about the decision by central and state governments to introduce NCERT
books in madrasas without disrupting the religious curriculum following the Nai Manzil scheme
launched to educate and empower the Muslim youth by modernizing madrasas. Among the given
options, only statement (a) ts perfectly into the paragraph as it continues on the same theme as
the paragraph. Other options are irrelevant in the context of the paragraph. Hence (a) is the
correct choice.
Purchased by roshnipanwar025@gmail.com
Q98.
Each of the following questions has a paragraph from which one sentence has been deleted. From
the given options, choose the one that completes the paragraph in the most appropriate way.
Has any decision-maker in the capital understood the full consequences of declaring its air a
“national emergency”? Visitors — whether on business or diplomats — will think three times
before visiting Delhi this winter. One has only to recall that it was estimated that when President
Obama visited for the Republic Day parade in 2015 he may have lost six hours of his life by
spending three days in the capital. The US Embassy imported 1,800 air puri ers for his entourage.
_______________________________________________ (Marks 1)
A. What’s more, it’s virtually a no-cost solution.
B. Certain causes, like the burning of farm residue require a carrot and stick approach to
encourage farmers to recycle crop waste rather than burn it.
C. Beijing, which was previously the black sheep of the world’s urban air contamination, recorded
for more join
53 micrograms.
https://t.me/currentAffairscLuB
D. Children can’t attend school or play outside, and this has made Delhi the air pollution pariah of
the world.
E. Indeed, a rule of thumb for any transport infrastructure scheme, whether in cities or outside
them, should be that they can be permitted only if half the users constitute the public.

Your Answer: Not Attempted

https://store.adda247.com/#!/myTestAnalysis/printsolution/mappingId=25273/packageId=359/lang=ENGLISH 81/109
12/31/2017 Adda247 Store | Adda247 Store
Correct Answer: D. Children can’t attend school or play outside, and this has made Delhi the air
pollution pariah of the world. <br>

Solution
The given paragraph is about the poor condition of air-quality in Delhi which is getting worse.
Among the given options, there is only sentence (d) which nds some connection with the
paragraph and at the same time it concludes the paragraph in the best manner, adding meaning
to it. Other options are not feasible enough to make the paragraph complete and conclusive.
Hence (d) is the correct choice.
Purchased by roshnipanwar025@gmail.com
Q99.
Each of the following questions has a paragraph from which one sentence has been deleted. From
the given options, choose the one that completes the paragraph in the most appropriate way.
Ms. Patel was a prominent face of the British-India relationship — being awarded the Pravasi
Bharatiya Samman earlier this year and often speaking publicly in support of the Indian
government’s policies, such as demonetisation. But her departure is unlikely to have a major
impact on things, given the broad-based nature of the engagement across departments.
_________________________________________. They include Foreign and Commonwealth O ce Minister
Mark Field, whose e orts to encourage the BJP to join the International Democratic Union, a global
alliance of centre-right parties, predated his time as minister for Asia. (Marks 1)
A. The fact that she was allowed to resign rather than be red is signi cant too
B. Others have suggested it represented part of a far more widely backed but behind the scenes
shift in British foreign policy.
C. It is notable that her replacement as Development Minister, Penny Mordaunt, was also a strong
Leave campaigner.
D. Meanwhile, the First Secretary of State, Damian Green, is facing a parliamentary inquiry over
conduct allegations.
E. Others within the Conservative Party have also been championing close relations with India, and
the Bharatiya Janata Party (BJP) in particular.
Your Answer: Not Attempted

Correct Answer: E. Others within the Conservative Party have also been championing close
for more join
relations with India, and the Bharatiya Janata Party (BJP) in particular. <br>
https://t.me/currentAffairscLuB
Solution
The paragraph is about the departure of Ms. Patel and its impact on India’s ruling party. Among
the given options, only sentence (e) nds an appropriate alternative to t into the sentence
perfectly. Other options are not in the context of the paragraph’s actual subject. Hence (e) is the
correct choice.
Purchased by roshnipanwar025@gmail.com
https://store.adda247.com/#!/myTestAnalysis/printsolution/mappingId=25273/packageId=359/lang=ENGLISH 82/109
12/31/2017 Adda247 Store | Adda247 Store
Q100.
Each of the following questions has a paragraph from which one sentence has been deleted. From
the given options, choose the one that completes the paragraph in the most appropriate way.
Though India is not on the president’s Asia itinerary, the nomenclature alone—Indo-Paci c rather
than Asia-Paci c—suggests that New Delhi stands rightly to play a central part in the Trump
administration’s larger Asia strategy. With long-standing allies like Japan, South Korea and
Australia, India o ers democratic and economic ballast to deal with the rise of China’s power.
_______________________________________. For the Trump team to succeed with the ambition of
building a network of Asian partners which share our values, including India, the White House will
need to corral its economic policies to match its strategic pursuits. (Marks 1)
A. Sadly, US economic policy appears disconnected from the administration’s broader strategic
goal.
B. To elevate India’s role, make it a full partner in our Asian network, and enhance Washington’s
relations with New Delhi, the administration should help India gain a seat at the tables from which
it is absent.
C. Asia’s third largest economy deserves to have a seat at the table, and it will help India to be
more embedded in the premier regime focused on free and open trade in Asia.
D. At a time when China has emerged as the most powerful economic partner to virtually every
country in Asia, including South Asia, we must have a stable strategic and non-contentious
relationship with India.
E. The Commerce Department’s technical discussion with India on standards marks a great step in
that direction.
Your Answer: Not Attempted

Correct Answer: A. Sadly, US economic policy appears disconnected from the administration’s
broader strategic goal. <br>

Solution
The given paragraph is about the US policies toward India and China. Among the given options,
only sentence ts perfectly into the blank space as it follows the sentence prior to the space as
well as the one following it. It brings the continuity into the meaning of the paragraph. Other
for more join
options are not relevant enough to bring about a similar meaning to the paragraph. Hence (a) is
the correct choice.
https://t.me/currentAffairscLuB
Purchased by roshnipanwar025@gmail.com
Q101.
There are four sentences given in the following question. Find the sentence(s) which is/are
grammatically correct and mark your answer choosing the best possible alternative among the
ve options given below each question. If all sentences are correct, choose (E) as your answer.

https://store.adda247.com/#!/myTestAnalysis/printsolution/mappingId=25273/packageId=359/lang=ENGLISH 83/109
12/31/2017 Adda247 Store | Adda247 Store
(I)Having trimmed China’s ideological sails to the winds of change, Xi Jinping has rather calculatedly
deviated from Mao Zedong’s thesis that “a revolution is not a dinner party”.
(II)While the US government has formally given notice of its intention to withdraw from the PA,
they are still in the UNFCCC and their withdrawal from the PA will take two years to come into
force.
(III)The evolution of resistant strains is a natural phenomenon and this is accelerated by the
selective pressure exerted by widespread, indiscriminate and irrational uses of antibacterial drugs.
(IV)Air puri er manufacturers are witnessing a spike in sales in the Capital and adjoining areas as
consumers resort to panic buying with air pollution in Delhi-NCR breaching the critical limit, even
as doctors doubt e cacy of such products. (Marks 1)
A. Only (I) is correct
B. Only (IV) is correct
C. Both (I) and (II) are correct
D. Only (I), (III) and (IV) are correct
E. All are correct
Your Answer: Not Attempted

Correct Answer: E. All are correct <br>

Solution
All the given sentences are grammatically correct and hence they don’t require any correction.
Purchased by roshnipanwar025@gmail.com
Q102.
The following question consists of a sentence which is divided into three parts which contain
grammatical errors in one or more than one part of the sentence, as speci ed in bold in each part.
If there is an error in any part of the sentence, nd the correct alternatives to replace those parts
from the three options given below each question to make the sentence grammatically correct. If
the given sentence is grammatically correct or does not require any correction, choose (E), i.e., “No
correction required” as your answer.
Air quality in Delhi-NCR is at the seasons’ worst for the last couple of days (I)/as a combined e ect
of smoke from stubble burning and moisture is turning the region (II)/into a “gas chamber” to gasp
the people. (III) for more join
(I)has been at the seasons’ worst
(II)turned the region https://t.me/currentAffairscLuB
(III)leaving people gasping (Marks 1)
A. Only (I)
B. Only (III)
C. Both (I) and (III)
D. All (I), (II) and (III)
E. No correction required
https://store.adda247.com/#!/myTestAnalysis/printsolution/mappingId=25273/packageId=359/lang=ENGLISH 84/109
12/31/2017 Adda247 Store | Adda247 Store
Your Answer: Not Attempted

Correct Answer: D. All (I), (II) and (III) <br>

Solution
In the rst part of the sentence, the phrase “ is at the seasons’ worst ” should be replaced by “ has
been at the seasons’ worst ” as the use of “ for the last couple of days” indicates that the sentence
is in Present Perfect Continuous form. In the second part, the present form of the phrase “ is
turning the region ” should be replaced by its past form “ turned the region ” as it is clearly
mentioned that the sentence is depicting the action of the past [ for the last couple of days]. In the
third part of the sentence the phrase “ to gasp the people ” is totally irrelevant and meaningless; it
should be replaced by the correct phrase “ leaving people gasping ” to make the sentence
grammatically correct. Thus all the three parts of the sentence require corrections in their bold
parts. Hence (d) is the correct choice.

Purchased by roshnipanwar025@gmail.com
Q103.
In the question given below, there are four sentences. Choose the sentence which is grammatically
incorrect as your answer. If all the given sentences are grammatically correct and do not require
any correction, choose (E) i.e. “All are correct” as your answer. (Marks 1)
A. The essence of the problems is that microbial drugs themselves fail to discriminate between
harmful bacteria and bene cial bacteria.
B. Non-therapeutic uses of antibiotics act as growth promoters as they kill microbes in the
intestine and thus help absorb feed nutrients better, resulting in weight gain.
C. Although Article 19 of the Constitution provides to all citizens freedom of speech and
expression, nothing explicitly guarantees freedom of the press and not much has changed since
colonial rule.
D. We know from contemporary experience as much as from history that crime and punishment
may be associative concepts but is not necessarily in a cause and e ect relationship.
E. All are correct.
Your Answer: Not Attempted
for more join
Correct Answer: D. We know from contemporary experience as much as from history that crime
https://t.me/currentAffairscLuB
and punishment may be associative concepts but is not necessarily in a cause and e ect
relationship. <br>

Solution
There is a grammatical error in the sentence (d), the singular verb “ is ” should be replaced by its
plural form “ are ” as it refers to more than one subject i.e. “ crime and punishment.” The other
sentences are grammatically correct. Hence (d) is the correct choice.
https://store.adda247.com/#!/myTestAnalysis/printsolution/mappingId=25273/packageId=359/lang=ENGLISH 85/109
12/31/2017 Adda247 Store | Adda247 Store
Purchased by roshnipanwar025@gmail.com
Q104.
There are three sentences given in the following question. Find the sentence(s) which is/are
grammatically correct and mark your answer choosing the best possible alternative among the
ve options given below each question. If all the sentences are correct, choose (E) as your answer.
(I)Logistics is the mainstay of any economy and a vital driver of economic progress as it
encompasses the management of ow of goods from the place of initiation to the place of
consumption.
(II)Although air is the fastest transportation mode, it continues to have a diminutive share in the
transportation pie and has insu cient connectivity and lack of cargo terminals.
(III)For improving storing facilities, decreasing transportation overheads and boosting e cacy of
complete logistics set-up, multimodal logistic parks should be developed and uni ed throughout
the country with focus on last mile connectivity. (Marks 1)
A. Only (II) is correct
B. Only (III) is correct
C. Both (I) and (II) are correct
D. Both (I) and (III) are correct
E. All are correct
Your Answer: Not Attempted

Correct Answer: E. All are correct <br>

Solution
All the given sentences are grammatically correct. Hence (e) is the correct choice.
Purchased by roshnipanwar025@gmail.com
Q105.
Rearrange the following sentences to form a meaningful paragraph and then answer the
questions that follow.
If the sentence (A), “In nance, what is 'pull to par'?” is the rst sentence of the paragraph, then
what is the sequence of other sentences after rearrangement?
(A)In nance, what is 'pull to par'?
(B)If the buyer had purchased the bond at a discount, the price of the bond begins to increase
for more join
towards its face value as the time left for its maturity decreases.
(C)Pull to par happens because any https://t.me/currentAffairscLuB
nancial security is priced based on the potential future cash
ow that it can provide its owner combined with the amount of time that the investor must wait
before receiving the cash.
(D)This refers to the tendency of the price of a bond in nancial markets to converge towards its
face value as the bond nears its maturity.
(E)In the case of a bond bought at a premium, its price begins to decrease towards its face value as
its time to maturity decreases every day. (Marks 1)
https://store.adda247.com/#!/myTestAnalysis/printsolution/mappingId=25273/packageId=359/lang=ENGLISH 86/109
12/31/2017 Adda247 Store | Adda247 Store
A. BEDC
B. EBDC
C. DCBE
D. DCEB
E. DBCE
Your Answer: Not Attempted

Correct Answer: C. DCBE <br>

Solution
If (A) is the rst sentence, the correct sequence of other sentences after rearrangement should be
DCBE. As the rst sentence is interrogative, the second statement should answer the same which
could be found in the sentence (D). The other sentences in the sequence of (C), (B) and (E) follow
one another to make a complete paragraph. Thus sentences in the sequence of ADCBE form a
coherent paragraph which is about the expression and de nition of the term “ pull to par .” Hence
(c) is the correct choice.
Purchased by roshnipanwar025@gmail.com
Q106.
Five statements are given below, labelled a, b, c, d and e. Among these, four statements are in
logical order and form a coherent paragraph. From the given options, choose the option that does
not t into the theme of the paragraph. (Marks 1)
A. The broad mechanism is akin to China’s own build-up of the BRI initiative.
B. China spent over a decade building capacities independently in its areas of interest .
C. For instance, it built a series of dual-use facilities across the Indian Ocean surrounding India
which is popularly referred to as its ‘string of pearls’ strategy.
D. All such e orts have now been consolidated under its grandiose idea of BRI.
E. The broader attempt is to institutionalize this into a structured trilateral format, and possibly a
quadrilateral one at a later stage.
Your Answer: Not Attempted

Correct Answer: E. The broader attempt is to institutionalize this into a structured trilateral format,
and possibly a quadrilateral one at a later stage. <br> for more join
Solution https://t.me/currentAffairscLuB
Option (e) is the correct choice as it is not a part of the coherent paragraph. Other statements are
forming a coherent paragraph with their sequence being abcd. The hint is the word ‘trilateral’ as
there isn’t any speci c mention of the 3rd country. The other reason is that it doesn’t t into the
sequence and even after being a part of the same article; it is prohibited from being a part of the
paragraph due to its context.
Purchased by roshnipanwar025@gmail.com
https://store.adda247.com/#!/myTestAnalysis/printsolution/mappingId=25273/packageId=359/lang=ENGLISH 87/109
12/31/2017 Adda247 Store | Adda247 Store
Q107.
Five statements are given below, labelled a, b, c, d and e. Among these, four statements are in
logical order and form a coherent paragraph. From the given options, choose the option that does
not t into the theme of the paragraph. (Marks 1)
A. This seems to be a criticism that verges on curbing creative freedom.
B. While there is far more freedom on social media than in print, there is also considerable danger
for a careless cartoonist, and rightfully so.
C. The comments and retorts, criticism both constructive and mindless, are immediate.
D. Posting cartoons directly on social media is a di erent, and new kettle of sh.
E. The followers need not necessarily be kind and appreciative. It is a free-for-all situation. No
holds barred.
Your Answer: Not Attempted

Correct Answer: A. This seems to be a criticism that verges on curbing creative freedom. <br>

Solution
The coherent paragraph is the observation of an expert where he explained the ills of posting
something directly on social media without going through the processes of editorial judgment as a
vital lesson to young angry activists who have embraced cyberspace not only to vent their anger
but also to galvanise people
Option (a) is the correct choice as it is not a part of the coherent paragraph as it is an opinion of
the other expert and is contradictory to the theme of the coherent paragraph as according to him,
Cartoons need breathing space and leeway to retain a sense of humour while delivering a political
opinion and option (a) is one of the sentences which conforms to his view. Other statements are a
part of the coherent paragraph with their sequence being dceb.
Purchased by roshnipanwar025@gmail.com
Q108.
Five statements are given below, labelled a, b, c, d and e. Among these, four statements are in
logical order and form a coherent paragraph. From the given options, choose the option that does
not t into the theme of the paragraph. (Marks 1)
A. Perhaps it is best to begin by making a distinction between toleration as an attitude of
individuals and toleration as a social practice.
for more join
B. Toleration exists in a society where it is a prized personal attribute, a virtue.
https://t.me/currentAffairscLuB
C. Toleration is an attitude of forbearance preceded by psychological turbulence and anxiety
D. It is increasingly felt by many that we are moving towards a ercely con ict-ridden world in
which toleration is needed badly. But what does toleration mean?
E. But it is also present where persecution, violent confrontation or bloodshed have somehow
been kept at bay.
Your Answer: Not Attempted

https://store.adda247.com/#!/myTestAnalysis/printsolution/mappingId=25273/packageId=359/lang=ENGLISH 88/109
12/31/2017 Adda247 Store | Adda247 Store
Correct Answer: C. Toleration is an attitude of forbearance preceded by psychological turbulence
and anxiety <br>

Solution

Option (c) is not the part of the coherent paragraph. By looking at all the sentences we can easily
gure out that option (d) is the 1st sentence of the paragraph. The next can be either (c) or (a) but
by looking at the options (b) and (e) we can infer that these are the statements which must come
next after (a) as both the statements are mentioning the subjects which are being talked about in
option (a). Option (c) feels abrupt and can’t t into the paragraph and hence is the correct choice.
Purchased by roshnipanwar025@gmail.com
Q109.
Five statements are given below, labelled a, b, c, d and e. Among these, four statements are in
logical order and form a coherent paragraph. From the given options, choose the option that does
not t into the theme of the paragraph. (Marks 1)
A. Initially wary of accommodating regional demands, primarily owing to the bitter experience of
Partition,
B. A multi-ethnic polity sharply polarised along linguistic, religious and caste lines, India is no
stranger to subnationalist impulses.
C. Telugu-speaking areas coalesced into Andhra Pradesh, Malayali-speaking areas into Kerala,
Kannada-speaking areas into Karnataka, etc. in the 1950s.
D. One only needs to ip the pages of history to realise how nationalism, by its very nature, has
the idea of exclusion built into it.
E. India’s central leadership embarked on an audacious project to recon gure the country’s
political map based on linguistic criteria:
Your Answer: Not Attempted

Correct Answer: D. One only needs to ip the pages of history to realise how nationalism, by its
very nature, has the idea of exclusion built into it. <br>

Solution
Option (d) is the correct choice for the given question. Other options make a coherent paragraph
for more join
with their sequence being baec. The theme of the paragraph is the country’s political map based
on linguistic criteria but option (d) is nothttps://t.me/currentAffairscLuB
in harmony with the theme of the passage.
Purchased by roshnipanwar025@gmail.com
Q110.
Five statements are given below, labelled a, b, c, d and e. Among these, four statements are in
logical order and form a coherent paragraph. From the given options, choose the option that does
not t into the theme of the paragraph. (Marks 1)
A. Once a transaction is entered in the blockchain, it cannot be erased or modi ed.
https://store.adda247.com/#!/myTestAnalysis/printsolution/mappingId=25273/packageId=359/lang=ENGLISH 89/109
12/31/2017 Adda247 Store | Adda247 Store
B. Simply put, it is a digital public ledger that records every transaction.
C. Blockchain removes the need for using a trusted third party such as a bank to make a
transaction by directly connecting the customers and suppliers.
D. Blockchain is the backbone technology on which bitcoins run.
E. Blockchain is not a panacea, but a system of e ciency that is faster than siloed technologies like
customer relationship management.
Your Answer: Not Attempted

Correct Answer: E. Blockchain is not a panacea, but a system of e ciency that is faster than siloed
technologies like customer relationship management. <br>

Solution
Option (e) is the correct choice. The hint is the use of the word ‘ panacea’ which means a solution
or remedy for all di culties or diseases. The paragraph is introducing us with the concept of
blockchain but option (e) is not in agreement with the theme of paragraph and is introducing new
topics. The other options make a coherent paragraph with their sequence being dbac.
Purchased by roshnipanwar025@gmail.com
Q111.
In each of the following sentence there are three blank spaces. Below each sentence there are ve
options and each option consists of three words which can be lled up in the blanks in the
sentence to make the sentence grammatically correct.
The time has come for the Bench and Bar to ________________ as a common man has the highest
faith in the _________________ called the judiciary. He has nowhere else to go to ________________ his
legitimate rights. (Marks 1)
A. meditate, foundation, guard
B. introspect, institution, vindicate
C. re ect, association, contend
D. modify, corporation, relieve
E. think, setup, manifest
Your Answer: Not Attempted

Correct Answer: B. introspect, institution, vindicate <br> for more join


Solution https://t.me/currentAffairscLuB
“introspect, institution, vindicate” is the correct set of words that ll perfectly into the blanks
provided in the sentence. Hence (b) is the correct choice.
Introspect means examine one's own thoughts or feelings.
Vindicate means show or prove to be right, reasonable, or justi ed.
Contend means struggle to surmount (a di culty).
Manifest means clear or obvious to the eye or mind.
https://store.adda247.com/#!/myTestAnalysis/printsolution/mappingId=25273/packageId=359/lang=ENGLISH 90/109
12/31/2017 Adda247 Store | Adda247 Store
Purchased by roshnipanwar025@gmail.com
Q112.
In each of the following sentence there are three blank spaces. Below each sentence there are ve
options and each option consists of three words which can be lled up in the blanks in the
sentence to make the sentence grammatically correct.
In the last few decades, the ___________________ and _________________ use gave rise to mutant
pathogens that are rapidly ________________ the e ectiveness of the limited arsenal of life-saving
anti-microbials. (Marks 1)
A. indiscriminate, overzealous, eroding
B. speci c, edgy, xing
C. systematic, emotional, crumbling
D. frivolous, passive, enabling
E. varied, peevish, preserving
Your Answer: Not Attempted

Correct Answer: A. indiscriminate, overzealous, eroding <br>

Solution
“indiscriminate, overzealous, eroding” is the correct set of words that ll perfectly into the blanks
provided in the sentence. Hence (a) is the correct choice.
Indiscriminate means done at random or without careful judgment.
Overzealous means too zealous in one's attitude or behaviour.
Edgy means tense, nervous, or irritable.
Frivolous means not having any serious purpose or value.
Peevish means having or showing an irritable disposition.
Purchased by roshnipanwar025@gmail.com
Q113.
In each of the following sentence there are three blank spaces. Below each sentence there are ve
options and each option consists of three words which can be lled up in the blanks in the
sentence to make the sentence grammatically correct.
Finance Minister Arun Jaitley announced ________________ Rs 6.92 lakh crore infrastructure
spending recently. A major area of focus will be the banking sector as banks are _______________
for more join
with bad debts and a sum of Rs 2.11 lakh crore for recapitalisation has been ________________.
(Marks 1) https://t.me/currentAffairscLuB
A. huge, aiding, allocated
B. enormous, loosening, signi cant
C. grand, tackling, wasted
D. massive, choking, earmarked
E. hefty, binding, shelled out
Your Answer: Not Attempted
https://store.adda247.com/#!/myTestAnalysis/printsolution/mappingId=25273/packageId=359/lang=ENGLISH 91/109
12/31/2017 Adda247 Store | Adda247 Store

Correct Answer: D. massive, choking, earmarked <br>

Solution
“massive, choking, earmarked” is the correct set of words that ll perfectly into the blanks
provided in the sentence. Hence (d) is the correct choice.
Earmark means designate (funds or resources) for a particular purpose.
Tackle means make determined e orts to deal with (a problem or di cult task).
Shell out means to give.
Hefty means large and heavy.
Purchased by roshnipanwar025@gmail.com
Q114.
In each of the following sentence there are three blank spaces. Below each sentence there are ve
options and each option consists of three words which can be lled up in the blanks in the
sentence to make the sentence grammatically correct.
The _____________ that there are only 193 member states of the United Nations gives political
scientists a problem. Their research into the way nation states work is highly ________________ by
the small number of cases they ______________ to study. (Marks 1)
A. case, subject, will have
B. matter, blamed, may have
C. fact, constrained, have
D. attainment, impelled, might have
E. point, inhibited, would have
Your Answer: Not Attempted

Correct Answer: C. fact, constrained, have <br>

Solution
“fact, constrained, have” is the correct set of words that ll perfectly into the blanks provided in the
sentence. Hence (c) is the correct choice.
Constrain means compel or force (someone) to follow a particular course of action.
Impel means drive, force, or urge (someone) to do something.
for more join
Inhibit means prevent or prohibit (someone) from doing something.
https://t.me/currentAffairscLuB
Purchased by roshnipanwar025@gmail.com
Q115.
In each of the following sentence there are three blank spaces. Below each sentence there are ve
options and each option consists of three words which can be lled up in the blanks in the
sentence to make the sentence grammatically correct.

https://store.adda247.com/#!/myTestAnalysis/printsolution/mappingId=25273/packageId=359/lang=ENGLISH 92/109
12/31/2017 Adda247 Store | Adda247 Store
Some have pointed to Pakistan’s diversity as a/an _______________ in its lack of democratic
development on the grounds that it is di cult to build a _______________, stable nation when there
are such sharp regional ________________. (Marks 1)
A. aspect, staunch, clash
B. cause, solid, emulation
C. element, obstinate, agreements
D. circumstance, retentive, enmity
E. factor, cohesive, rivalries
Your Answer: Not Attempted

Correct Answer: E. factor, cohesive, rivalries <br>

Solution
“factor, cohesive, rivalries” is the correct set of words that ll perfectly into the blanks provided in
the sentence. Hence (e) is the correct choice.
Cohesive means characterized by or causing cohesion.
Obstinate means stubbornly refusing to change one's opinion or chosen course of action, despite
attempts to persuade one to do so.
Retentive means (of a substance) able to absorb and hold moisture.
Enmity means a state or feeling of active opposition or hostility.
Purchased by roshnipanwar025@gmail.com
Q116.
Which among the following banks has entered into a Memorandum of Understanding (MoU) with
Life Insurance Corporation (LIC) of India for selling life insurance products of LIC?
A. Kotak Mahindra Bank
B. Axis Bank
C. ICICI Bank
D. State Bank of India
E. Karnataka Bank
Your Answer: Not Attempted

Correct Answer: E. Karnataka Bank <br> for more join


Solution https://t.me/currentAffairscLuB
Karnataka Bank Ltd has entered into a Memorandum of Understanding (MoU) with Life Insurance
Corporation (LIC) of India for selling life insurance products of LIC.
Purchased by roshnipanwar025@gmail.com
Q117.

https://store.adda247.com/#!/myTestAnalysis/printsolution/mappingId=25273/packageId=359/lang=ENGLISH 93/109
12/31/2017 Adda247 Store | Adda247 Store
RBI issued fresh directions on managing risks and code of conduct in outsourcing of nancial
services by NBFCs. Non-banking nancial companies (NBFCs) cannot outsource core management
functions like-
A. internal audit
B. strategic and compliance functions for know your customer (KYC) norms
C. sanction of loans and management of investment portfolio
D. All of the above
E. None of these
Your Answer: Not Attempted

Correct Answer: D. All of the above <br>

Solution
Non-banking nancial companies (NBFCs) cannot outsource core management functions like
internal audit, strategic and compliance functions for know your customer (KYC) norms, sanction
of loans and management of investment portfolio. In a noti cation on its website, RBI issued fresh
directions on managing risks and code of conduct in outsourcing of nancial services by NBFCs.
Purchased by roshnipanwar025@gmail.com
Q118.
Communications Minister Manoj Sinha has launched the SBG Yojana and an initiative for
expansion of clientele base of Postal Life Insurance (PLI). SBG stands for-?
A. Sampoorna Bima Grih
B. Samast Bima Gram
C. Sampoorna Bishal Gram
D. Sampoorna Bima Gram
E. Sampoorna Bima Grameen
Your Answer: Not Attempted

Correct Answer: D. Sampoorna Bima Gram <br>

Solution
Communications Minister Manoj Sinha here launched the Sampoorna Bima Gram (SBG) Yojana
for more join
and an initiative for expansion of clientele base of Postal Life Insurance.
https://t.me/currentAffairscLuB
Purchased by roshnipanwar025@gmail.com
Q119.
Which city is served by Rajiv Gandhi International Airport?
A. Ahmedabad
B. Kochi
C. Kozhikode
D. Hyderabad
https://store.adda247.com/#!/myTestAnalysis/printsolution/mappingId=25273/packageId=359/lang=ENGLISH 94/109
12/31/2017 Adda247 Store | Adda247 Store
E. Guwahati
Your Answer: Not Attempted

Correct Answer: D. Hyderabad <br>

Solution
Rajiv Gandhi International Airport is an international airport that serves Hyderabad, the largest city
in the Indian state of Telangana. It is located in Shamshabad, about 20 kilometres south of
Hyderabad.
Purchased by roshnipanwar025@gmail.com
Q120.
Match the following:
A. WTO ----- I. Provides loans to address short-term balance of payments problems
B. RBI ----- II. Multilateral trade negotiating body.
C. IMF ----- III. Facilitating lending and borrowing for reconstruction and development
D. IBRD ----- IV . Central Bank of India
A. A (II) B (IV) C (I) D (III)
B. A (III) B (IV) C (I) D (II)
C. A (I) B (IV) C (II) D (III)
D. A (IV) B (II) C (I) D (III)
E. None of these
Your Answer: Not Attempted

Correct Answer: A. A (II) B (IV) C (I) D (III) <br>

Solution
RBI-India central bank
WTO- The World Trade Organization (WTO) is the only international organization that deals with
the global rules of trade between nations.
IMF- The International Monetary Fund (IMF) is an international organization created for the
purpose of standardizing global nancial relations and exchange rates.
Purchased by roshnipanwar025@gmail.com
Q121.
for more join
Work force refers to that part of. https://t.me/currentAffairscLuB
A. Labour force which is employed.
B. Population which is unemployed.
C. Population which is forced to work.
D. Labour force which is unemployed.
E. None of these
Your Answer: Not Attempted
https://store.adda247.com/#!/myTestAnalysis/printsolution/mappingId=25273/packageId=359/lang=ENGLISH 95/109
12/31/2017 Adda247 Store | Adda247 Store

Correct Answer: A. Labour force which is employed. <br>

Solution
The total number of people employed or seeking employment in a country or region is called work
force.
Purchased by roshnipanwar025@gmail.com
Q122.
Which country has the world’s rst ‘negative emissions’ power plant?
A. Ireland
B. Finland
C. Scotland
D. Sweden
E. Iceland
Your Answer: Not Attempted

Correct Answer: E. Iceland <br>

Solution
A company in Iceland has created the world’s rst “negative emissions” power plant.
Purchased by roshnipanwar025@gmail.com
Q123.
Madikheda Dam is in which of the following state?
A. Himachal Pradesh
B. Uttar Pradesh
C. Madhya Pradesh
D. Gujarat
E. Jharkhand
Your Answer: Not Attempted

Correct Answer: C. Madhya Pradesh <br>

Solution
for more join
https://t.me/currentAffairscLuB
Madikheda Dam is a Multi-purpose Dam situated in Shivpuri district in Madhya Pradesh, India. Its
Coordinates: 25°33'20"N 77°51'10"E.
Purchased by roshnipanwar025@gmail.com
Q124.
The headquarters of the Organization of Petroleum Exporting Countries (OPEC) is situated in
A. Kuwait City, Kuwait
B. Baghdad, Iraq

https://store.adda247.com/#!/myTestAnalysis/printsolution/mappingId=25273/packageId=359/lang=ENGLISH 96/109
12/31/2017 Adda247 Store | Adda247 Store
C. Geneva, Switzerland
D. Paris, France
E. Vienna, Austria
Your Answer: Not Attempted

Correct Answer: E. Vienna, Austria <br>

Solution
Organization of the Petroleum Exporting Countries is an intergovernmental organization of 14
nations as of May 2017, founded in 1960 in Baghdad by the rst ve members (Iran, Iraq, Kuwait,
Saudi Arabia, Venezuela), and headquartered since 1965 in Vienna.
Purchased by roshnipanwar025@gmail.com
Q125.
Who has won the 2017 Shanghai Masters tennis tournament?
A. Juan Martin del Potro
B. Roger Federer
C. Dominic Thiem
D. Rafael Nadal
E. None of these
Your Answer: Not Attempted

Correct Answer: B. Roger Federer <br>

Solution
Roger Federer extended his winning streak over Rafael Nadal this year to four matches after
beating the top-ranked Spaniard 6-4, 6-3 to win the Shanghai Masters.
Purchased by roshnipanwar025@gmail.com
Q126.
Name the Yojana that Madhya Pradesh Chief Minister Shivraj Singh Chouhan has recently
inaugurated _________________ for the farmers of the state.
A. Kaushal Samvardhan Yojana
B. Krishak Udhyami Yojana Loan Scheme
C. Shala Siddhi Prohatsan Yojana
for more join
D. Bhavantar Bhugtan Yojana https://t.me/currentAffairscLuB
E. None of these
Your Answer: Not Attempted

Correct Answer: D. Bhavantar Bhugtan Yojana <br>

https://store.adda247.com/#!/myTestAnalysis/printsolution/mappingId=25273/packageId=359/lang=ENGLISH 97/109
12/31/2017 Adda247 Store | Adda247 Store
Solution
Madhya Pradesh Chief Minister Shivraj Singh Chouhan inaugurated the ambitious ‘Bhavantar
Bhugtan Yojna’ in the presence of the farmers at the Krishi Upaj Mandi in Khurai, Sagar.
Purchased by roshnipanwar025@gmail.com
Q127.
The Karera Wildlife Sanctuary (KWS) is located in which state?
A. Madhya Pradesh
B. Odisha
C. Uttar Pradesh
D. Punjab
E. Kerala
Your Answer: Not Attempted

Correct Answer: A. Madhya Pradesh <br>

Solution
Karera Wildlife Sanctuary is a wildlife Sanctuary in the Shivpuri district of Madhya Pradesh, India.
Established in 1981 to protect a population of the great Indian bustard in the region, it is now in
the process of being denoti ed due to opposition by the local people and the extinction of the bird
locally.
Purchased by roshnipanwar025@gmail.com
Q128.
What is the currency of Singapore?
A. Pound
B. Dollar
C. Rial
D. Peso
E. Ringgit
Your Answer: Not Attempted

Correct Answer: B. Dollar

Solution
for more join
Currency of Singapore- Dollar https://t.me/currentAffairscLuB
President- Halimah Yacob
Prime Minister-Lee Hsien Loong
Purchased by roshnipanwar025@gmail.com
Q129.
Name the chairman and whole time director of the SAB group, who has passed away in Mumbai,
Maharashtra.

https://store.adda247.com/#!/myTestAnalysis/printsolution/mappingId=25273/packageId=359/lang=ENGLISH 98/109
12/31/2017 Adda247 Store | Adda247 Store
A. Purushottam Lal Kaushik
B. Gautam Adhikari
C. Arjan Singh
D. Ahmed Khan
E. None of these
Your Answer: Not Attempted

Correct Answer: B. Gautam Adhikari <br>

Solution
Gautam Adhikari passed away due to a heart attack. He was the chairman and whole time director
of the SAB group.
Purchased by roshnipanwar025@gmail.com
Q130.
ICC Cricket World Cup for the year 2023 to be hosted by which country?
A. England
B. India
C. West Indies
D. South Africa
E. None of these
Your Answer: Not Attempted

Correct Answer: B. India <br>

Solution
ICC Cricket World Cup for the year 2023 to be hosted by India
Purchased by roshnipanwar025@gmail.com
Q131.
Bollywood star _______________________ has become the rst female actor ever to hoist the Indian
National Flag at the Indian Film Festival of Melbourne (IFFM), Australia’s annual celebration of lm
from India and the subcontinent.
A. Alia Bhatt
B. Sonakshi Sinha
for more join
C. Kareena Kapoor https://t.me/currentAffairscLuB
D. Aishwarya Rai Bachchan
E. Deepika Padukone
Your Answer: Not Attempted

Correct Answer: D. Aishwarya Rai Bachchan <br>

https://store.adda247.com/#!/myTestAnalysis/printsolution/mappingId=25273/packageId=359/lang=ENGLISH 99/109
12/31/2017 Adda247 Store | Adda247 Store
Solution
Bollywood star Aishwarya Rai Bachchan has become the rst female actor ever to hoist the Indian
National Flag at the Indian Film Festival of Melbourne (IFFM), Australia’s annual celebration of lm
from India and the subcontinent. The o cial Twitter handle of IFFM posted a picture of the 43-
year-old actress.
Purchased by roshnipanwar025@gmail.com
Q132.
United Nations Day is observed globally on _____________.
A. 12 October
B. 16 October
C. 18 October
D. 24 October
E. 23 October
Your Answer: Not Attempted

Correct Answer: D. 24 October <br>

Solution
24 October has been celebrated as United Nations Day since 1948. In 1971, the United Nations
General Assembly recommended that the day be observed by Member States as a public holiday.
Purchased by roshnipanwar025@gmail.com
Q133.
In the capital market, the term arbitrage is used with reference to
A. purchase of securities to cover the sale
B. sale of securities to reduce the loss on purchase
C. simultaneous purchase and sale of securities to make pro ts from price
D. variation in di erent markets
E. All of the above
Your Answer: Not Attempted

Correct Answer: C. simultaneous purchase and sale of securities to make pro ts from price <br>

Solution
for more join
Arbitrage is the simultaneous purchasehttps://t.me/currentAffairscLuB
and sale of an asset to pro t from a di erence in the price.
It is a trade that pro ts by exploiting the price di erences of identical or similar nancial
instruments on di erent markets or in di erent forms. Arbitrage exists as a result of market
ine ciencies.
Purchased by roshnipanwar025@gmail.com
Q134.

https://store.adda247.com/#!/myTestAnalysis/printsolution/mappingId=25273/packageId=359/lang=ENGLISH 100/109
12/31/2017 Adda247 Store | Adda247 Store
Name the lender that has recently launched the second edition of its Hackathon named 'Digitize
For Bank.'
A. IndusInd Bank
B. Dena Bank
C. Punjab National Bank
D. Allahabad Bank
E. State Bank of India
Your Answer: Not Attempted

Correct Answer: E. State Bank of India <br>

Solution
The State Bank of India has launched the second edition of its Hackathon named 'Digitize For
Bank.' This hackathon will be held from 1st to12th November 2017. It is presented by SBI
Collaborative Innovation Centre (CIC). This edition focuses on four ‘Cognitive’ themes namely Facial
Recognition, Signature Recognition, Voice based Authentication and Cheque Truncation Value
Enhancers.
Purchased by roshnipanwar025@gmail.com
Q135.
Name the village, which has been adopted by SBI for driving the Aadhaar-based merchant
payment initiative?
A. Rudrapur Village, Uttarakhand
B. Andhratharhi village, Jharkhand
C. Bela Village, Uttar Pradesh
D. Barsam Village, Bihar
E. Shirki village, Maharashtra
Your Answer: Not Attempted

Correct Answer: E. Shirki village, Maharashtra <br>

Solution
The State Bank of India (SBI) has adopted Shirki village in Pen taluka in Maharashtra for driving the
for more join
Aadhaar-based merchant payment initiative. Under the initiative, the merchants in the village have
https://t.me/currentAffairscLuB
been provided with simple android phones with a USB-based ngerprint capture device.The
merchant is required to have a bank account with an Aadhaar number needed.
Purchased by roshnipanwar025@gmail.com
Q136.
Mixed economy means
A. co-existence of small and large industries
B. promoting both agriculture and industries in the economy
https://store.adda247.com/#!/myTestAnalysis/printsolution/mappingId=25273/packageId=359/lang=ENGLISH 101/109
12/31/2017 Adda247 Store | Adda247 Store
C. co-existence of public and private sectors
D. co-existence of the rich and the poor
E. existence of wholesale and retail markets
Your Answer: Not Attempted

Correct Answer: C. co-existence of public and private sectors <br>

Solution
A mixed economy is de ned as an economic system consisting of a mixture of either markets and
economic planning, public ownership and private ownership, or markets and economic
interventionism.
Purchased by roshnipanwar025@gmail.com
Q137.
Expand the term FLCC.
A. Financial Literacy and Credit Counseling
B. Financial Literacy Communication Centre
C. Fiscal Literacy Communication Centre
D. Fiscal Literacy and Credit Counseling
E. None of the above
Your Answer: Not Attempted

Correct Answer: A. Financial Literacy and Credit Counseling <br>

Solution
Purchased by roshnipanwar025@gmail.com
Q138.
India Post Payments Bank (IPPB) has appointed _____________ as its Managing Director & Chief
Executive O cer.
A. Suresh Sethi
B. Mahesh Kumar Jain
C. Satpal Singh
D. Amritpal Saroya
E. Amrinder Singh
for more join
Your Answer: Not Attempted https://t.me/currentAffairscLuB
Correct Answer: A. Suresh Sethi <br>

Solution
Former MD of Vodafone M-Pesa Ltd Suresh Sethi was appointed as the Managing Director (MD)
and Chief Executive O cer (CEO) of India Post Payments Bank (IPPB).
https://store.adda247.com/#!/myTestAnalysis/printsolution/mappingId=25273/packageId=359/lang=ENGLISH 102/109
12/31/2017 Adda247 Store | Adda247 Store
Purchased by roshnipanwar025@gmail.com
Q139.
Name the rst bank in India listed in New York Stock Exchange (NYSE).
A. Union Bank of India
B. SBI Bank
C. HDFC Bank
D. ICICI Bank
E. Central Bank of India
Your Answer: Not Attempted

Correct Answer: D. ICICI Bank <br>

Solution
Purchased by roshnipanwar025@gmail.com
Q140.
In ation can de ned as
A. a persistent rise in general price level
B. a persistent fall in general price level
C. an increase purchasing power
D. increase in value of money
E. decrease in money
Your Answer: Not Attempted

Correct Answer: A. a persistent rise in general price level <br>

Solution
In economics, in ation is a sustained increase in the general price level of goods and services in an
economy over a period of time.
Purchased by roshnipanwar025@gmail.com
Q141.
World trade organization came into being on
A. 1st April, 1995
B. 1st April, 1996 for more join
C. 1st January, 1995
https://t.me/currentAffairscLuB
D. 1st April, 1994
E. None of these
Your Answer: Not Attempted

Correct Answer: C. 1st January, 1995 <br>

https://store.adda247.com/#!/myTestAnalysis/printsolution/mappingId=25273/packageId=359/lang=ENGLISH 103/109
12/31/2017 Adda247 Store | Adda247 Store
Solution
The World Trade Organization (WTO) is an intergovernmental organization that regulates
international trade. The WTO o cially commenced on 1 January 1995 under the Marrakesh
Agreement, signed by 123 nations on 15 April 1994, replacing the General Agreement on Tari s
and Trade (GATT), which commenced in 1948. It is the largest international economic organization
in the world.
Purchased by roshnipanwar025@gmail.com
Q142.
On which of the following lender the Reserve Bank of India has imposed a monetary penalty of Rs
2 crore for contravention of regulatory restrictions pertaining to loans and advances?
A. IDBI Bank
B. IndusInd Bank
C. Dena Bank
D. Dhan Laxmi Bank
E. IDFC Bank
Your Answer: Not Attempted

Correct Answer: E. IDFC Bank <br>

Solution
The Reserve Bank of India has imposed a monetary penalty of 2 crore on IDFC Bank for
contravention of regulatory restrictions pertaining to loans and advances. The central bank, in a
statement said the penalty has been imposed in exercise of powers vested in RBI under the
Banking Regulation Act, 1949, taking into account failure of the bank to adhere to certain
directions issued by RBI.
Purchased by roshnipanwar025@gmail.com
Q143.
Oriental Bank of Commerce (OBC) is an India-based bank and is one of the public sector banks in
India. Where is the corporate o ce of OBC?
A. Gurugram, Haryana
B. Kolkata, West Bengal
C. Pune, Maharashtra
D. Chennai, Tamil Nadu
for more join
E. Bengaluru, Karnataka https://t.me/currentAffairscLuB
Your Answer: Not Attempted

Correct Answer: A. Gurugram, Haryana <br>

https://store.adda247.com/#!/myTestAnalysis/printsolution/mappingId=25273/packageId=359/lang=ENGLISH 104/109
12/31/2017 Adda247 Store | Adda247 Store
Solution
Headquarters-Gurugram, India
CEO&MD-Shri Mukesh Kumar Jain
Purchased by roshnipanwar025@gmail.com
Q144.
The term MCLR refers to the new benchmark lending rate at which banks now lend to borrowers.
What does the letter ‘M’ denotes in the term MCLR?
A. Medium
B. Minimum
C. Marginal
D. Maintenance
E. Multiple
Your Answer: Not Attempted

Correct Answer: C. Marginal <br>

Solution
The marginal cost of funds based lending rate (MCLR) refers to the minimum interest rate of a
bank below which it cannot lend, except in some cases allowed by the RBI. It is an internal
benchmark or reference rate for the bank. MCLR actually describes the method by which the
minimum interest rate for loans is determined by a bank - on the basis of marginal cost or the
additional or incremental cost of arranging one more rupee to the prospective borrower.
Purchased by roshnipanwar025@gmail.com
Q145.
Name the Vice Admiral, who has taken charge as the new Vice Chief of Naval Sta (VCNS) at a
formal ceremony held in New Delhi.
A. Karambir Singh
B. Ajit Kumar P
C. P N Murugesan
D. D K Dewan
E. None of these
Your Answer: Not Attempted
for more join
Correct Answer: B. Ajit Kumar P <br> https://t.me/currentAffairscLuB
Solution
Vice Admiral Ajit Kumar P, AVSM, VSM has taken charge as the new Vice Chief of Naval Sta (VCNS)
at a formal ceremony held in New Delhi
Purchased by roshnipanwar025@gmail.com
Q146.

https://store.adda247.com/#!/myTestAnalysis/printsolution/mappingId=25273/packageId=359/lang=ENGLISH 105/109
12/31/2017 Adda247 Store | Adda247 Store
The book titled “Dreamnation: Uniting a Country with Handwritten Dreams” has been co-authored
by
A. Saji Mathew and Jubie John
B. Subroto Bagchi and Y.S. Rajan
C. Jubie John and Srijan Pal Singh
D. Y.S. Rajan and Saji Mathew
E. None of these
Your Answer: Not Attempted

Correct Answer: A. Saji Mathew and Jubie John <br>

Solution
The book titled “Dreamnation: Uniting a Country with Handwritten Dreams” has been co-authored
by Saji Mathew and Jubie John and is published by Bloomsbury.
Purchased by roshnipanwar025@gmail.com
Q147.
Who is the Governor of Tripura?
A. Padmanabha Balkrishna Acharya
B. Nirbhay Sharma
C. V. Shanmuganathan
D. Tathagata Roy
E. None of these
Your Answer: Not Attempted

Correct Answer: D. Tathagata Roy <br>

Solution
Capital of Tripura-Agartala
Chief Minister- Manik Sarkar
Governor- Tathagata Roy
Purchased by roshnipanwar025@gmail.com
Q148.
for more join
The Sariska Tiger Reserve is a national park and wildlife refuge located in the Alwar district of the
state of https://t.me/currentAffairscLuB
A. Madhya Pradesh
B. Gujarat
C. West Bengal
D. Rajasthan
E. Maharashtra
Your Answer: Not Attempted
https://store.adda247.com/#!/myTestAnalysis/printsolution/mappingId=25273/packageId=359/lang=ENGLISH 106/109
12/31/2017 Adda247 Store | Adda247 Store

Correct Answer: D. Rajasthan <br>

Solution
Sariska Tiger Reserve is a national park and tiger reserve located in the Alwar district of the state
of Rajasthan, India.
Purchased by roshnipanwar025@gmail.com
Q149.
Y Sanathoi Devi is associated with which sports?
A. Hockey
B. Football
C. Wushu
D. Badminton
E. None of these
Your Answer: Not Attempted

Correct Answer: C. Wushu <br>

Solution
Y Sanathoi Devi is associated with Wushu sports
Purchased by roshnipanwar025@gmail.com
Q150.
Which of the following Indian states has the highest onion production in the country?
A. Bihar
B. Maharashtra
C. Gujarat
D. Rajasthan
E. Karnataka
Your Answer: Not Attempted

Correct Answer: B. Maharashtra <br>

Solution
for more join
https://t.me/currentAffairscLuB
Purchased by roshnipanwar025@gmail.com
Q151.
The Index of Industrial Production (IIP) is compiled and released by
A. CII
B. ASSOCHAM
C. RBI
D. EXIM BANK

https://store.adda247.com/#!/myTestAnalysis/printsolution/mappingId=25273/packageId=359/lang=ENGLISH 107/109
12/31/2017 Adda247 Store | Adda247 Store
E. CSO
Your Answer: Not Attempted

Correct Answer: E. CSO <br>

Solution
The Index of Industrial Production (IIP) is an index for India which details out the growth of various
sectors in an economy such as mineral mining, electricity and manufacturing. The all India IIP is a
composite indicator that measures the short-term changes in the volume of production of a
basket of industrial products during a given period with respect to that in a chosen base period. It
is compiled and published monthly by the Central Statistical Organization (CSO)
Purchased by roshnipanwar025@gmail.com
Q152.
The Queen's Baton for the 2018 Commonwealth Games arrived in New Delhi, India. The 2018
Commonwealth Games will be held in __________.
A. Australia
B. New Zealand
C. England
D. Sri Lanka
E. None of these
Your Answer: Not Attempted

Correct Answer: A. Australia <br>

Solution
2018 Commonwealth Games. The 2018 Commonwealth Games, o cially known as the XXI
Commonwealth Games and commonly known as Gold Coast 2018, is an international multi-sport
event for members of the Commonwealth that will be held on the Gold Coast, Queensland,
Australia between 4 and 15 April 2018.
Purchased by roshnipanwar025@gmail.com
Q153.
Banks in India normally do not have to deal with the issues related to
A. customer complaints
for more join
B. cross complaints https://t.me/currentAffairscLuB
C. payment and settlement system
D. intellectual property system
E. cases of insolvency
Your Answer: Not Attempted

Correct Answer: D. intellectual property system <br>


https://store.adda247.com/#!/myTestAnalysis/printsolution/mappingId=25273/packageId=359/lang=ENGLISH 108/109
12/31/2017 Adda247 Store | Adda247 Store

Solution
Purchased by roshnipanwar025@gmail.com
Q154.
Central Board of Direct Taxes (CBDT) clari ed that farmers do not need to quote Permanent
Account Number (PAN) for cash sale of their produce up to Rs.____ lakh a day.
A. Rs 1 lakh
B. Rs 2 lakh
C. Rs 3 lakh
D. Rs 4 lakh
E. Rs 5 lakh
Your Answer: Not Attempted

Correct Answer: B. Rs 2 lakh <br>

Solution
The revenue department said that the farmers do not need to quote PAN for cash sale of their
produce up to Rs 2 lakh a day.
Purchased by roshnipanwar025@gmail.com
Q155.
As per the recommendations of the 7th Pay Commission, the government has raised the
maximum amount that a central government employee can borrow from the government to Rs
________________ for new construction/purchase of new house/ at.
A. 20 lakh
B. 25 lakh
C. 15 lakh
D. 30 lakh
E. 10 lakh
Your Answer: Not Attempted

Correct Answer: B. 25 lakh <br>

Solution
for more join
https://t.me/currentAffairscLuB
The total amount of advance that a central government employee can borrow from government
has been revised upwards. The employee can borrow upto 34 months of the basic pay subject to a
maximum of Rs 25 lakh, or cost of the house/ at, or the amount according to repaying capacity,
whichever is the least for new construction/purchase of new house/ at. Earlier this limit was only
Rs 7.50 lakh..

https://store.adda247.com/#!/myTestAnalysis/printsolution/mappingId=25273/packageId=359/lang=ENGLISH 109/109

You might also like